Ευκλειδης Β 59

82
59 Vx,p(x)� 3 x , p( x ) 3 χ, p(χ)� Vx, p(χ) Ε λληνική Μ α θηματ ι κή Ε τ α ιρ ε ί α

description

 

Transcript of Ευκλειδης Β 59

Page 1: Ευκλειδης Β 59

59

Vx,p(x)�3x,p(x) 3 χ, p (χ)� Vx, p (χ)

Ελληνική Μαθηματική Εταιρ ε ί α

Page 2: Ευκλειδης Β 59

ιδεuτι Βιβλία

yια μαβητέ� και εκπαιδευτικού�

ΜΑ.ΡΙΑ ΕΥΣJΑθΙΟV • ΕΛΕVθΕΡΙΟΣ ΠΡΟtΟΠΑΠΑΣ

ΜΕΘΟΔΟΛΟΓιΑ

Μεθοδολογία Άλγεβρας Α Ενιαίου Λυκείου

Μ. Ευσταθίου, Ε. Πρωτοπαπάς

rwιoιιwπι:ι.I'Wfl>llfll: mtot!'!Hia:ΚJ!fN\'f�

lila.\IO'fUII01"""-'IO'r τοrιο.....m.

I

ο

Γεωμετρία Α' Ενιαίου Λυκείου Γ. Βιδάλης, Β. Γ κιμίσης

μαθηματικά Γ' Ενιαίου Λυκείου

Μαθηματικά

Γεωμετρία Β' Ενιαίου Λυκείου

Γενικής Παιδείας Ε. Πρωτοπαπάς

*

μαθηματικά �·τόμος

ΘΕΤΙΚΗt ΚΑΙ τΕΧΝΟΛΟΓΙΚΗΣ ΚΑτΕΥΘνΝΣΗΣ

Μαθηματικά Γ' Ενιαίου Λυκείου

Ανάπτυξη εφαρμογών σε προγραμματιστικό περιβάλλον

Γ Ενιαίου Λυκείου θετικής & τεχνολογικής κατεύθυνσης

Επ. Χονδρογιάννης, Δ. Σουφλής

Γ Ενιαίου Λυκείου Γ ενικής Παιδείας Μ. Τ σιλπιρίδης

Θετικής και τεχνολογικής κατεύθυνσης Α' τόμος: Ε. Πρωτοπαπάς

ΕΚΔΟΣΕΙΣ ΠΑΤΑΚΗ www.patakis.gr

Β' τόμος: Ε. Πρωτοπαπάς, Σ. Γκούμας

σε όλο το βιβλιοπωλείο ΒΙΒΛΙΟΠΩΛΕΙΟ ΠΑΤΑΚΗ: ΑΚΑΔΗΜΙΑΣ 65. I 06 78 ΑΘΗΝΑ, ΤΗΛ. 210.38.11.850

KENTPIKH ΔΙΑΘΕΣΗ: ΕΜΜ. ΜΠΕΝΑΚΗ 16, I 06 78 Α Θ Η ΝΑ, ΤΗΛ. 210.38.31.078

ΘΕΣΣΑΛΟΝΙΚΗ: Ν. ΜΟΝΑΠΗΡΙΟΥ 122, ΤΗΛ. 2310.70.63.54-5

TRlNA SΑ:----ι .-��·.τ::" eoo' ()YD I ' ιr -0 ''"''"'"" !. so 9001

,.,.,,,,,,,," :

, .•. ι:.ι•c,-..ι,· :

--------------------------- . .......

Ccrtificaιι: Νο. 081054

Page 3: Ευκλειδης Β 59

ΕΛΛΗΝΙΚΗ ΜΑΘΗΜΑΤΙΚΗ ETAIPEIA Τεύχος 59 • Ιανουάριος • Φεβρουάριος • Μάρτιος 1006 • Ευρώ: 3,50

e-m;ιil: [email protected] www.hms.gr

ΜΑΘΗΜΑ η κο ΠΕΡΙΟΔΙΚΟ ΠΑ ΤΟ Λ ΥΚΕΙΟ

ΠΕΡΙΕΧΟΜΕΝΑ ../ Ιστορικές Μαθηματικές Αναφορές - Μαγεία των αριθμών ....... . . .. . ...... 2 ../ Στοιχεία Μαθηματικής Λογικής ........................................ 6 ../ Μαθηματικοί Διαγωνισμοί- Ευκλείδης - Αρχιμήδης - Ένα πρόβλημα .. . ..... . ..... . . . 12

Μαθηματικά Α' Τάξης ../ Συστήματα- Τριώνυμο β' βαθμού- Ανισώσεις β' βαθμού ........ . .... . .... 20 ../ Συστήματα- Συναρτήσεις- Τριώνυμο . . . . . . . . . . .......... 24 ../ Κύκλος και αναλογίες . . . ........ . ................................. 2 7

Μαθηματικά Β' Τάξης ../ Από τον ΣΕΣΑ στον Gauss . . . . . . . . . . . . . . . . . . . . . . . . . . . . . . . . . . . . . . 31 ../ Από τα κανονικά πολύγωνα στα καμπυλόγραμμα χωρία ................... 35 ../ Κωνικές τομές .............................................. 42

Μαθηματικά Γ' Τάξης ../ Ασκήσεις Ανάλυση . . . . . . . . . . . . . . . . . . . . . . . . . . . . .... 48 ../ Ασκήσεις στις παραγώγους . . . . . . . . . . . . . ............... 52 ../ Πιθανότητες . . . . . . . . . . . . . . . ................................ 58 ../ Ο Ευκλείδης προτείνει ... Ευκλείδη και • . . Διόφαντο ....................... 64

../ Στήλη του μαθητή ............................................... 72 ../ , .

δ 'ζ ..

Τα Μαθηματικα μας Διασκε α συν ......................... . ......... 77

••••••••••••••••••••••••••••••••••••••••••••••••••••••••••••••••••••••••••••••

ΕΚΔΟΣΗ ΤΗΣ ΜΑΘΗΜΑJΙΚΗΣ ΗΑΙΡΕΙΑΣ

ΠΑΝΕΠΙΠΗΜΙΟΥ 34 - 106 79 ΑΘΗΝΑ Τηλ.: 210 3617784-3616532 Fax: 2103641025 Εκδότης: Εξαρχάκος Θεόδωρος Διευθυντής: Τuρλής Ιωάννης Κωδικός ΕΛ.ΤΑ.: 2055 ISSN: 1105- 7998 Επιμέλεια 'Εκδοσης: Κυριακόπουλος Αντώνης Ευσταθίου Βαγγέλης

Εκτελεστική Γραμματεία Πρόεδρος:

Κυριακόπουλος Αντώνης

Αντιπρόεδροι:

Α': Ευσταθίου Βαγγέλης

ι·: Τοαα6nουλος Γιώργος

Γ ρομματέας:

Χριστόπουλος Παναγιώτης

Μέλη: Αργυράκης Δ.

Δρούταος π. Λουρίδας Σ. Τοnειν6ς Ν.

Συντακτική επιτροπή Αθανασόπουλος Γεώργιος Ανδρουλακάκης Νίκος Αντωνόπουλος Νίκος Αργυράκης Δημήτριος Βακαλόπουλος Κώσtας Βλάχου Αγγελική Γράψας Κων/νος Δρούτσας Παναγιώτης Ευσtαθίου Βαγγέλης Ζαχαρόπουλος Κων/νος Ζώτος Βαγγέλης Καλίκας Σταμάτης Καρακατσάνης Βασίλης Καρκάνης Βασίλης Κατσούλης Γιώργος Κερασαρίδης Γιάννης Καρδαμίτσης Σπύρος Κηπουρός Χρήστος Κόντζιας Νίκος Κοτσιφάκης Γιώργος Κυριακόπουλος Αντώνης Κυριακόπουλος Θανάσης

Κυβερνήτου �υστ. Λαζαρίδης Χρήσtος Λουρίδας Σωτήρης Μαλαφέκας Θανάσης Μεταξάς Νικόλαος Μυλωνάς Δημήτρης Μώκος Χρήσtος Ρέγκλης Δημήτρης Σαίτη Εύα Σταθόπουλος Γεώργιος Σταϊκος Κώστας Στάϊκος Παναγιώτης Στρατής Γιάννης Ταπεινός Νικόλαος Τασσόπουλος Γιώργος Τριάντος Γεώργιος Τσικαλουδάκης Γιώργος Τσιούμας Θανάσης Τυρλής Ιωάννης Χαραλαμποπούλου Λίνα Χαραλάμπους Θάνος Χρισtόπουλος Παναγιώτης

••••••••••••••••••••••••••••••••••••••••••••••••••••••••••••••••••••••••••••••

ΚΕΝΤΡΟ fΡΑΦΙΚΩΝ 1ΕΧΝΩΝ: ΔΙΗΝΕΚΕΣ ΣτοιχειοΒεtιία - Σελιδοποίηtιη: Ακαδημίας 43, Αθιiνα τηλ.: 210 3606760,

fax.: 210 3606826 e-mail: [email protected]

Εκτύπωσ-η: ΙΝΤΕΡΠΡΕΣ Α.Ε. τηλ.: 210 8160330 Υπειίθuνο� τuπογpαφείοu: Β. Σωτηριάδης

• Τα διαφημιζόμενα βιβλία δε σημαίνει ότι προτείνονται από την Ε.Μ.Ε. • Οι συνεργάτες, τα άρθρα, οι προτεινόμενες ασκήσεις, οι λύσεις ασκήσεων κτλ. πρέπει

να σtέλνονται έγκαιρα, στα γραφεία της Ε.Μ.Ε. με την ένδειξη "Για τον Ευκλείδη β'". Τα χειρόγραφα δεν επισι;ρέφονται. Τιμή Τεύχους ευρώ 3,50

Ετήσια συνδQομή (10,00 + 4,00 Ταχυδρομικά = ευρώ 14,00)

Ετήσια συνδρομή για Σχολεία ευρώ 10,00

Το αντίτιμο για τα τεύχη που παραγγέλνονται σtέλνεται με απλή επιταγή σε διαταγή Ε.Μ.Ε. Ταχ. Γραφείο Αθήνα 54 Τ. Θ. 30044 ή πληρώνεται στα γραφεία της Ε.Μ.Ε.

Page 4: Ευκλειδης Β 59

Επιμέλεια: Χρήστος Κηπουρός

Η ΜΑΓΕΙΑ ΤΩΝ ΑΡΙΘΜΩΝ

Ο Πυθαγόρας (585 • 500 π.Χ.)

Α πό την επινόηση των αριθμών όπως τους χρησιμοποιούμε σήμερα (Ινδία, τέ­λος του 6ου αιώνα), οι άνθρωποι προ­

σπάθησαν να βρουν όχι μόνον σχέσεις μεταξύ των αριθμών, αλλά και σχέσεις μεταξύ των αριθμών και της φύσης.

Ο Πυθαγόρας διαπίστωσε ότι τα φυσικά φαι­νόμενα κυβερνώνται από νόμους, οι οποίοι μπο­ρούν να περιγραφούν με μαθηματικές εξισώσεις. Επίσης συνειδητοποίησε ότι οι αριθμοί βρίσκονταν κρυμμένοι σ' όλα τα πράγματα, από την αρμονία στη μουσική μέχρι τις τροχιές των πλανητών!

Σύντομα εδώ θα παρουσιάσουμε κάποιες χαρα­κτηριστικές περιπτώσεις: Τέλειοι αριθμοί

Σύμφωνα με τον Πυθαγόρα, η αριθμητική τε­

λειότητα εξαρτάται από τους διαιρέτες ενός αριθ­μού (τους αριθμούς που διαιρούν τέλεια τον αρχι­κό) . Σημαντικότεροι και σπανιότεροι είναι οι α­ριθμοί εκείνοι που το άθροισμα των διαιρετών τους είναι ακριβώς ίσο με αυτούς. Αυτοί οι αριθ­μοί ονομάζονται τέλειοι. Οι διαιρέτες του 6 είναι οι 1 ,2,3 και, συνεπώς, ο 6 είναι τέλειος διότι 1 +2+3=6. Ο επόμενος τέλειος είναι ο 28, γιατί 1 +2+4+7+ 14=28 .

του Γιάννη Σιούλα

Ο Πυθαγόρας (585- 500 π.Χ)

Η Πυθαγόρειος Αδελφότητα απέδιδε μεγάλη μαθηματική σπουδαιότητα στην τελειότητα του 6 και του 28. Αργότερα, αυτό αναγνωρίστηκε και από άλλους πολιτισμούς, οι οποίοι παρατήρησαν ότι μία πλήρης περιστροφή της σελήνης γύρω από τη γη χρειάζεται 28 μέρες και δήλωσαν ότι ο Θεός δημιούργησε τον κόσμο σε 6 μέρες. Ο Άγιος Αυ­γουστίνος , μάλιστα, στην Πόλη του Θεού γράφει ότι ενώ ο Θεός μπορούσε να δημιουργήσει τον κό­σμο σε μια στιγμή, αποφάσισε να χρησιμοποιήσει 6 μέρες για να απεικονίσει έτσι την τελειότητα του σύμπαντος. Τρίτος τέλειος αριθμός είναι ο 496, τέ­ταρτος ο 8 . 1 28, πέμπτος ο 33 .550.336 . . κτλ.

ΕΥΚΛΕΙΔΗΣ Β ' λθ ' τ.3/2

Page 5: Ευκλειδης Β 59

Η ιστορία των αριθμών και των συμβόλων

Ενώ για τον Ευκλείδη, τον Θέωνα τον Σμυρ­ναίο και τους νεο-Πυθαγόρειους τέλειος αριθμός ήταν κάθε αριθμός της μορφής που περιγράψαμε πιο πάνω, οι Πυθαγόρειοι είχαν συμπεριλάβει και το 10 στους τέλειους αριθμούς. Αυτό, κατά τον Αριστοτέλη, οφειλόταν στο γεγονός ότι στον αριθ­μό 10 αναγνώριζαν έννοιες όπως είναι: το κενό, η αναλογία, η περιττότης κ.ο.κ. Οι λόγοι αυτής της αποδοχής επεξηγούνται με λεπτομέρεια από τον Θέωνα τον Σμυρναίο και από το απόσπασμα του Σπεύσιππου. Ο 10 είναι το άθροισμα των τεσσά­ρων πρώτων φυσικών αριθμών ( 10=1+2+3+4), οι οποίοι σχηματίζουν αυτό που ονομάζεται «τετρα­

κτύς» και στην οποίαν ορκίζονταν ότι δεν θα

προδώσουν τα μυστικά της Σχολής.

Φίλιοι αριθμοί ή αγαπητοί Με τους τέλειους αριθμούς έπρεπε να συγκρι­

θούν οι λεγόμενοι φίλιοι αριθμοί. Ο Ιάμβλιχος α­ποδίδει την ανακάλυψη αυτών των αριθμών στον Πυθαγόρα, ο οποίος όταν ρωτήθηκε «τί είναι φί­

λος» απάντησε «άλλος εγώ» και, στη βάση αυτής της φιλοσοφίας εφάρμοσε τον όρο «φίλος» για δύο αριθμούς, καθένας από τους οποίους ισούται με το άθροισμα των γνήσιων διαιρετών του άλλου.

Οι Πυθαγόρειοι είχαν κάνει την καταπληκτική ανακάλυψη ότι οι αριθμοί 220 και 284 είναι φί­

λιοι.

Οι διαιρέτες του 220 είναι οι αριθμοί 1 ,2,4,5, 10, 1 1 , 20,22,44,55, 1 10 και το άθροισμά τους είναι ο αριθμός 284. Αντίστοιχα οι διαιρέτες του 284 είναι οι αριθμοί 1 ,2,4,7 1 , 142 και το άθροισμά τους είναι ο αριθμός 220!. Το ζευγάρι των αριθμών αυτών έλεγαν ότι αποτελεί σύμβολο φιλίας.

Ο Μάρτιν Γκάρντνερ στο βιβλίο του Mathe­matical Magic Show αναφέρει ότι στο Μεσαίωνα πουλιόνταν φυλαχτά που είχαν χαραγμένους αυ­τούς τους αριθμούς και ευνοούσαν ερωτικά, ό­ποιον τα φορούσε. Κάποιοι επίσης θεολόγοι των πρώιμων χρόνων σημείωσαν το γεγονός ότι στη Γένεση ο Ιακώβ έδωσε 220 κατσίκια στον Ησαύ, πιστεύοντας ότι ο αριθμός αυτός , το μισό του φι­λικού ζευγαριού (220,284), αποτελούσε έκφραση

αγάπης του Ιακώβ προς τον Ησαύ. Αυτό, όμως, ακούγεται παράξενα γιατί θα πρέπει να δεχθούμε ότι και οι δύο γνώριζαν τα «περί φίλων αριθ­

μών», τη σημασία τους και θα ήσαν, τουλάχιστον, σύγχρονοι του Πυθαγόρα.

Ο Γάλλος μαθηματικός Καρτέσιος (Descartes 1 596- 1650) και ο Ολανδός μαθηματικός Van Schooten ( 1 6 1 5- 1 660) ανακάλυψαν άλλα τρία ζευ­γάρια: (2620, 2964), (5020,5564), (6232,6368).

Descartes 1 596- 1650 Van Schooten 1 6 1 5- 1 660

Euler ( 1707- 1783

Ο Γερμανοελβετός Euler ( 1 707- 1783) ανακά­λυψε άλλα 6 1 αγαπητά ζευγάρια. Το 1 866 όμως ο δεκαεξάχρονος Ιταλός Νικολό Παγκανίνι ανακά­λυψε το ζευγάρι 1 . 1 84 και 1 .2 1 Ο που το είχαν όλοι παραβλέψει!

Ο δι.:��σ�'ιιuως π Ο πιο διάσημος άρρητος αριθμός είναι ο π

(δηλ. ο λόγος της περιμέτρου ενός κύκλου προς τη διάμετρό του). Για τις σχολικές ανάγκες παίρ-

, ' 22 νου με ως προσεγγιστικη τιμ η του π το ? ή το

3 , 14. Ο Ήρων εγκλωβίζει τον π στο ανοικτό διά­στημα:3 , 141 5904<π<3 , 14 160 16.(βλ:Ηρωνος Α­λεξ/ρέως τ.4ος εκδ.ΕΜΕ). Η πραγματική του όμως τιμή βρίσκεται κοντά στον ασύμμετρο 3 , 14 1 59265358979323846.

Ο καθηγητής Χανς-Χένρικ Στέλουμ (γεωλόγος στο Κέμπριτζ), σκεπτόμενος ίσως τον ισχυρισμό του Πυθαγόρα για τους αριθμούς που διέπουν τα

ΕΥΚΛΕΙΔΗΣ Β ' λθ ' τ.3/3

Page 6: Ευκλειδης Β 59

Η ιστορία των αριθμών και των συμβόλων -----------

φυσικά φαινόμενα, υπολόγισε το λόγο μεταξύ του πραγματικού μήκους ενός φιδωτού ποταμού από την πηγή μέχρι την εκβολή και της αντίστοιχης ευ­θείας γραμμής. Ο λόγος αυτός είναι περίπου 3 , Ι 4 ! Στο συμπέρασμα αυτό πρώτος είχε καταλήξει ο Αϊνστάιν.

Χρυσή τομή ή φ:::::1,618

Ένα πρόβλημα που σχετίζεται με την αισθητική των καλλιτεχνημάτων στην αρχαιότητα, είναι το πρόβλημα της χρυσής τομής που γνώριζαν οι Πυ­θαγόρειοι το οποίο, σε απλή διατύπωση, έχει ως εξής: Να χωριστεί ένα τμήμα ΑΒ=λ (Σχ. Ι ) , σε μέ­σο και άκρο λόγο δηλ. σε δύο μέρη Α Τ=χ και ΤΒ=λ-χ, ώστε : ΑΒ ΑΤ

,

λ χ ΑΤ = ΤΒ η χ= λ-χ (1)

:--4 ι ι λ τ

ι Α: ι ι

ι :Β ι ι

ι· --------- · χ · --------- ............ ι---- λ-χ ... ι

Θ ' λ ο ' ετουμε: - = φ > , οποτε χ χ

--

λ-χ

Ι = -- και

φ-Ι ( Ι ) γίνεται: φ2 - φ - Ι

= Ο . Η θετική ρίζα αυτής

ξ, ' I+J5 ' 6 8 της ε ισωσης ειναι: φ = η φ�Ι , Ι και 2

χ=0,6 Ι 8λ.

ολ, I+J5 ' ζ , , ογος φ = 2 ονομα εται <<χρυση τομη»

και συμβολίζεται, διεθνώς, με το γράμμα φ, προς τιμή του γλύπτη Φειδία, ο οποίος, στην κατασκευή του Παρθενώνα, χρησιμοποίησε τη χρυσή τομή

' λ ' 6 9 ' λ ' και τη μουσικη ανα ογια: 8 = Ι2 .η οποια εμπ ε-

κεται στην κατασκευή της μουσικής Πυθαγόρειας

κλίμακας.

Ο όρος «χρυσή» δόθηκε επειδή παρατηρήθηκε, ιδίως κατά την Αναγέννηση, ότι ο λόγος φ χρησι­μοποιήθηκε ευρύτατα από τους Αρχιτέκτονες κα­τασκευαστές των κλασικών αριστουργημάτων, όπως π.χ. είναι τα θέατρα τα στάδια, οι λατρευτι­κοί χώροι, οι ακροπόλεις, τα Δημόσια κτίρια.κ.λ.π.

Ο Θεός, στο τελειότερο δημιούργημά του, απο­τύπωσε, εντέχνως, τον λόγο φ, δίνοντάς του έτσι τον χαρακτήρα της τελειότητας. Ο Λεονάρντο ντα Βίντσι π.χ. μελέτησε τον ανατομικό κανόνα του ανθρώπινου σώματος, και παρατήρησε ότι ο ομ­φαλός διαιρεί το ύψος του ανθρώπου κατά τη χρυ­σή του τομή ενώ ο Λούκα Πατσιόλι, αφού ονόμα­σε τη σχέση αυτή «θεία αναλογία», την έλαβε ως βάση της αισθητικής αναλογίας των κτηρίων.

Έχει παρατηρηθεί ότι η τήρηση του λόγου φ στις αναλογίες των καλλιτεχνικών κατασκευών δημιουργεί την αίσθηση της αρμονίας. Ο αριθμός αυτός αποτελούσε για τους aρχαίους Έλληνες γε­ωμέτρες, γλύπτες και αρχιτέκτονες το «δόγμα της

ωραιότητας». Όπως φαίνεται στο Σχήμα 2, η κα­τασκευή των διαζωμάτων στο θέατρο της Επιδαύ­ρου (τέλος 4ου π.Χ. αιώνα) έχει γίνει σύμφωνα με το λόγο φ.

Ο λόγος αυτός εμφανίζεται στις αναλογίες του πεντάλφα , μυστικού σήματος της Πυθαγό­ρειας Σχολής. (Σχ.3) .

Α Δ

Σχ.3 Ο λόγος της χρυσής τομής εμφανίζεται και

στον τρόπο διάταξης των φύλλων των φυτών. Ει­δικά στα φυτά σπουδαίο ρόλο παίζει η ακολουθία του Fibonacci:

1 , Ι ,2,3,5,8, 1 3 ,21 ,34,55,89, . . . (Κάθε όρος από τον τρίτο και μετά προκύπτει από την πρόσθεση

ΕΥΚΛΕΙΔΗΣ Β ' λθ ' τ.3/4

Page 7: Ευκλειδης Β 59

Η ιστορία των αριθμών και των συμβόλων

των δύο προηγούμενων). Παίρνοντας το λόγο ενός οποιουδήποτε όρου .προς τον προηγούμενό του (α­πό τον 5° όρο και πέρα) παρατηρούμε ότι: 8/5= Ι ,6 2 1/1 3= 1 ,6 1 5 55/34= 1 ,6 17 ...

Δηλαδή οι όροι αυτοί της ακολουθίας προσεγ- ·

γίζουν όλο και περισσότερο τη χρυσή τομή φ. «Χρυσή τομή» ονομάζουμε και τη σχέση 4:9

που κυριαρχεί στον Παρθενώνα. Για παράδειγμα το πλάτος του στυλοβάτη προς το μήκος του, η διάμε­τρος των κιόνων προς το μεταξόνιο( 1 ,905μ:4,296μ), το ύψος του ναού προς το πλάτος του ( 13 ,72μ:30,88=4:9), το πλάτος του κυρίως ναού προς το μήκος του, ενώ το πλάτος του ναού προς το ύψος έχουν μια σχέση 16: 8 1 , δηλ. 42 :9 2

Όπως αναφέραμε πιο πάνω, η μουσική αναλο-, 6 9 'ζ λ ' θ ' για -= - σχετι εται με το π η ος των κιονων

8 12 του Παρθενώνα, γιατί ο Πυθαγόρας, στην κατα­σκευή της μουσικής του κλίμακας, χρησιμοποίησε

λ ' 9 ( ' ' ' ' το ογο - , που εχει ως ορους, τους μεσους ορους 8

της μουσικής αναλογίας) και ο οποίος εκφράζει τον συντελεστή συχνότητας του βασικού φθόγγου

της κλίμακας. Το άθροισμα των όρων του κλάσμα-

τος 9 δηλ. το 1 7, μας δίνει το πλήθος των κιόνων 8

της μεγάλης πλευράς του Παρθενώνα ενώ το 8, που είναι ο μέσος αρμονικός όρος . των άκρων ό­ρων 6 και 1 2 της μουσικής αναλογίας, μας δίνει το πλήθος των κιόνων της μικρής πλευράς του. (Βλ. εισαγωγή Στερεομετρικών σελ.χίν).

Πηγές: 1. Simon Sinh. Το τελευταίο θεώρημα του

Feπnat, Έκδ. ΤΡ Α Υ ΛΟΣ 2. ΙΣΤΟΡΙΑ ΤΟΥ ΕΛΛΗΝΙΚΟΥ ΕΘΝΟΥΣ,

έκδ:Εκδοτική. Αθηνών 3. ΜΑΘΗΜΑτΙΚΆ Γ ΓΥΜΝΆΣΙΟΥ, ΟΕΔΒ 4. Ήρωνος Αλεξανδρέως, τ.III,IV,V, Έκδ.ΕΜΕ. 5. sir Thomas L.Heath. Ιστορία των ελληνικών

Μαθηματικών. έκδ.Κ.Ε.ΕΠ.ΕΚ. Αθήνα 200 1 . 6. Θέων ο Σμυρναίος έκδ. E.Hiller;B .G.Teubner,

Λειψία 1 878. 7. Ιάμβλιχος, Σχόλ. στην Αριθ . Εισαγωγή του Νι­

κομάχου. έκδ. Pistellί, Teubner, Λειψία.1894.

ΕΥΚΛΕΙΔΗΣ Β ' λθ ' τ.3/5

Page 8: Ευκλειδης Β 59

"Τα Μαθηματικά θεμελιώνονται, κατανοούνται και αναπτύσσονται με τη βοήθεια της Μαθηματικής Λογικής"

του Αντώνη Κυριακόπουλου

(Συνέχεια από το προηγούμενο τεύχος)

2.1. ΠΡΟΤΑΣΙΑΚΟΙ ΤΥΠΟΙ ΜΙΑΣ

ΜΕΤΑΒΛΗΤΗΣ.

• ροτασιακός τύπος μιας μεταβλητής χ ο-

νομάζεται κάθε έκφραση, η οποία περιέχει

τη μεταβλητή χ και η οποία (έκφραση) γί­

νεται μία πρόταση (αληθής ή ψευδής), όταν

η μεταβλητή χ αντικατασταθεί με ένα ο­

ποιοδήποτε στοιχείο ενός δοσμένου (μη κε­

νού) συνόλου Ω. Π.χ. η έκφραση :

«Ο χ είναι άρτιος» όπου το χ διατρέχει το σύνολο: Ω = {1, 2, 3, 4, 5,} ,

είναι ένας προτασιακός τύπος μιας μεταβλητής χ (δεν είναι μια πρόταση). - Τους προτασιακούς τύπους μιας μεταβλητής χ τους συμβολίζουμε με:

p(x), q(x), r(x), ... - Έστω p(x) ένας προτασιακός τύπος μιας μετα­βλητής χ Ε Ω . Το σύνολο Ω που διατρέχει η με­ταβλητή χ, ονομάζεται σ6νολα ομωμο-!J του ρ( χ) . Λέμε ακόμα ότι ο p(x) ε�νω ορισμένος επί του συνόλου Ω. Γράφουμε:

p(x)IΩ . - Έστω ένας προτασιακός τύπος ρ( χ) I Ω . Ένα στοιχείο ξ του Ω λέμε ότι επαληθεύει τον ρ( χ) , αν, και μόνο αν, η πρόταση p(ξ) είναι αληθής. Λέμε ακόμα, τότε, ότι γω το στοιχείο ξ Ε Ω ι­··Γ{;οc ο ρ( χ) .

Το σύνολο των στοιχείων του Ω, που επαλη-

θεύουν τον p( χ) , δηλαδή το σύνολο {χ Ε Ω I ρ( χ) αληθής} ονομάζεται σύνολο αλη ­

(/είας του ρ( χ) . Πολλές φορές, το σύνολο αυτό το γράφουμε απλούστερα ως εξής: {χ Ε Ω I ρ( χ)} ή, αν δεν υπάρχει αμφιβολία για το σύνολο ορισμού Ω, ως εξής: {χ I ρ( χ)} . Είναι φανερό ότι το σύνολο αυτό είναι υποσύνολο του Ω. 2.2. ΠΡΟΤ ΑΣΙΑΚΟΙ ΤΥΠΟΙ ΔΥΟ Η

ΠΕΡΙΣΣΟΤΕΡΩΝ ΜΕΤΑΒΛΗΤΩΝ.

• Προτασιακός τύπος δύο (τριών, ... ) μεταβλη­

τών χ, y, (ω, ... ) ονομάζεται κάθε έκφραση, η

οποία περιέχει τις μεταβλητές χ, y, (ω, ... ) και

η οποία (έκφραση) γίνεται μία πρόταση (α­

ληθής ή ψευδής) όταν οι μεταβλητές χ, y, (ω, ... ) αντικατασταθούν με οποιαδήποτε

στοιχεία δύο (τριών, ... ) δοσμένων (μη κενών)

συνόλων Α, Β (Γ, . . . ), αντιστοίχως [εννοούμε

χ Ε Α, y Ε Β,( ω Ε Γ, ... )]. Π.χ. η έκφραση :

«0 χ είναι μεγαλύτερος του y» , όπου το χ διατρέχει το σύνολο Ν= {0, 1, 2, . . . } και το y διατρέχει το σύνολο IR των πραγματικών α­ριθμών, είναι ένας προτασιακός τύπος δύο μετα­βλητών χ και y (δεν είναι μία πρόταση). - Τους προτασιακούς τύπους δύο μεταβλητών χ και y τους συμβολίζουμε με:

ρ( χ, y), q(x, y), r(x, y), ... τριών μεταβλητών χ, y, ω με:

ρ( χ , y , ω), q(χ, y, ω), r(x, y, ω), ... κ.ο.κ.

ΕΥΚΛΕΙΔΗΣ Β ' λθ ' τ.3/6

Page 9: Ευκλειδης Β 59

------------- Στοιχεία Μαθηματικής Λογικής -------------- Έστω ένας προτασιακός τύπος δύο μεταβλητών p(x, y) , όπου χ Ε Α και y Ε Β . Προφανώς το (διατεταγμένο) ζεύγος (x, y) διατρέχει το σύνολο

(καρτεσιανό γινόμενο) : AχB = {(x, y) lxEA και yEB} .

Έτσι, ο ρ( χ, y) μπορεί να θεωρηθεί ως προτα­

σιακός τύπος με μεταβλητή το ζεύγος (x,y) και

σύνολο ορισμού το παραπάνω σύνολο Α χ Β . Γράφουμε:

p(x , y) IAχB . - Έστω ένας προτασιακός τύπος p( χ, y) I Α χ Β . Ένα στοιχείο (ξ, η) του Α χ Β λέμε ότι επαληθεύει

με σύνολο ορισμού το Ω, τότε οι παρακάτω εκ­φράσεις είναι επίσης προτασιακοί τύποι με σύνολο ορισμού το Ω:

ρ( χ), p(x) ν q(x) ,

p(x) � q(x), ρ( χ)<::> q(x) .

2.4. ΚΑΘΟΛΙΚΟΣ ΠΟΣΟΔΕΙΚΤΗΣ

Έστω ένας προτασιακός τύπος ρ( χ) I Ω . Το

σύνολο αληθείας αυτού είναι: {xEΩip(x)} (ς Ω) .

τον p(x, y) , αν, και μόνο αν, η πρόταση p(ξ, n) • Η πρόταση, η οποία είναι:

είναι αληθής. Λέμε ακόμα, τότε, ότι για το ζεύγος (ξ,n) ισχύει ο p(x,y).

Το σύνολο των στοιχείων του Α χ Β , που επαληθεύουν τον ρ( χ, y) , δηλαδή το σύνολο: {(χ , y) Ε Α χ Β I ρ( χ, y) αληθής} , ονομάζεται σύ­

νολο αληθείας του ρ( χ, y) . Πολλές φορές, το σύ­

νολο αυτό το γράφουμε απλούστερα ως εξής: {(x , y) Ε Αχ Β I p(x, y)} ή, αν δεν υπάρχει αμφιβο-

λία για το σύνολο ορισμού Α χ Β , ως εξής: {(x , y) I p(x , y)} . Είναι φανερό ότι το σύνολο αυτό

είναι υποσύνολο του Α χ Β . - Ανάλογα ορίζονται το σύνολο ορισμού και το σύνολο αληθείας ενός προτασιακού τύπου τριών, τεσσάρων κτλ. μεταβλητών. - Στους συμβολισμούς: p(x) , ρ( χ, y) κτλ. το p

παριστάνει μια ομάδα λέξεων (ή και άλλων γνω­στών συμβόλων), οι οποίες, όπως λέμε, εκφράζουν μία ιδιότητα, η οποία αποδίδεται (αναφέρεται) σε ένα, σε δύο κτλ. αντικείμενα, αντιστοίχως. Μία τέτοια ιδιότητα λέγεται και κατηγόρημα μιας θέ­σεως, δύο θέσεων κτλ., αντιστοίχως.

2.3. ΠΡΟΤΑΣΙΑΚΟΙ ΤΥΠΟΙ ΚΑΙ ΛΟΓΙΚΟΙ

ΣΥΝΔΕΣΜΟΙ.

Συνδέοντας με τους λογικούς συνδέσμους δο­σμένους προτασιακούς τύπους, όπως και τις προ­τάσεις, οι εκφράσεις που προκύπτουν είναι επίσης προτασιακοί τύποι.

Π. χ., αν ρ( χ) και q(x) είναι προτασιακοί τύποι

• αληθής, αν όλα τα στοιχεία του Ω επα­ληθεύουν τον p(x) (δηλαδή αν Α = Ω) και

• ψευδής στην αντίθετη περίπτωση (δη­

λαδή αν Α * Ω), συμβολίζεται με:

'ν'χ Ε Ω,p (χ ) (1) και διαβάζεται: «για κάθε χ Ε Ω, p( χ)».

Το σύμβολο: 'ν', το οποίο μετατρέπει τον προ­τασιακό τύπο p(x) σε πρόταση, ονομάζεται καθο-

λικός ποσοδείκτης και διαβάζεται: «για κάθε».

Π. χ. έχουμε τις εξής προτάσεις:

'ν'χ Ε JR , χ2 + 1 > Ο (αληθής),

'ν'ν Ε Ν, ν + 2 > 20 (ψευδής).

- Αν δεν υπάρχει αμφιβολία για το σύνολο ορι­σμού Ω του p(x) , τότε, αντί της ( 1 ) , γράφουμε:

V'x, p(x) .

Σημείωση: Πολλές φορές στα Μαθηματικά, αντί:

'ν'χ Ε Ω,p(χ) , γράφουμε: p(x), 'ν'χ Ε Ω .

2.5. ΥΠΑΡΞΙΑΚΟΣ ΠΟΣΟΔΕΙΚΤΗΣ Έστω ένα προτασιακός τύπος ρ( χ) I Ω . Το

σύνολο αληθείας αυτού είναι: A = {xEΩip(x)} (ς;Ω) .

ΕΥΚΛΕΙΔΗΣ Β ' λθ ' τ.3/7

Page 10: Ευκλειδης Β 59

------------- Στοιχεία Μαθηματικής Λογικής -------------

η οποία �:ίνυι: Για παράδειγμα, έστω ο προτασιακός τύπο;: •η· i:να τουλιίzιστο στοιzείο του « 2χ = y », όπου καθεμία από τις μεταβλητές χ και

ηη· ισ( χ) (δηλαδι� �i11·' y διατρέχει το σύνολο Ν των φυσικών αριθμών. c"' (Zi ! γυ1 i) Η έκφραση : « 3χ , 2χ = y » είναι ένας προτασια-

Τί"'Τ•' ιη·τiΟπη iη:ρiτηι•J:ση (δη- κός τύπος μιας μεταβλητής y (π.χ. για y = 8 γίνε-

( IJ : <<ΙΙπ{,φz�:ι χ ε Ω,μ( χ ι""

Το σύμβολο: 3 , το οποίο μετατρέπει τον προ­τασιακό τύπο p(x) σε πρόταση, ονομάζεται οrω;;-

- και διαβάζεται: «υπάρχει» (με την έννοια: «υπάρχει τουλάχιστο ένα») .

Π. χ. έχουμε τις εξής προτάσεις: 3χΕ�, χ3 - 4χ + 3 = 0 (αληθής), 3χΕ�, χ2 - χ + 2 = 0 (ψευδής) .

- Αν δεν υπάρχει αμφιβολία για το σύνολο ορισμού Ω του ρ( χ) , τότε αvri της ( 1 ), γράφουμε: 3χ, p(x) .

Στα Μαθηματικά, αν ένας προτα­σιακός τύπος ρ( χ) I Ω επαληθεύεται για ένα μόνο στοιχείο του Ω, γράφουμε: 3'χ Ε Ω, p(χ) και δια­βάζουμε: «υπάρχει ένα μόνο χ ΕΩ,p(χ) » .

2.6. ΠΟΣΟΔΕΙΚΤΕΣ ΚΑΙ ΠΡΟΤΑΣΙΑΚΟΙ

ΤΥΠΟΙ ΠΕΡΙΣΣΟΤΕΡΩΝ

ΜΕΤΑΒΛΗΤΩΝ

Έστω ένας προτασιακός τύπος δύο μεταβλητών p( χ , y) . Κάθε μία από τις εκφράσεις:

Vx, p(x , y) και 3x, p(x , y) ( 1 )

είναι ένα Επίσης, κάθε μία από τις εκφράσεις:

Vy, p(x , y) και 3y, p(x , y)

Έτσι, καθεμία από τις εκφράσεις: Vx, Vy, p(x , y) Vy, Vx , p(x, y) 3x, Vy, p(x , y) Vx, 3y, p(x , y) 3x, 3y, p(x , y)

3y, Vx, p(x, y) Vy, 3x, p(x , y) 3y, 3x , p(x , y)

(2)

ται μία πρόταση αληθής, για y = 5 γίνεται μία πρόταση ψευδής) . i�) Η έκφραση : « Vx, 3y, 2x = y » είναι μία πρότα-ση (αληθής, γιατί;) . Επίσης, η έκφραση : « 3y, Vx, 2x = y » είναι μία πρόταση (ψευδής, για-τί;) .

- Θεωρώντας έναν προτασιακό τύπο τριών, τεσσάρων κτλ. μεταβλητών φθάνουμε σε ανάλογα συμπεράσματα.

\ I ,;(i iϊjΗΊταιιη, που τr�:ριi:zι:ι και ποσοδt:ί-κτι:ς, κω τωσοδ::ικτικι] πρ{ιηωΙι.

- Στους προτασιακούς τύπους ( 1 ) λέμε ότι το χ είναι μία ιiαηυ:υμ{;νη μεταβλητή και ότι το y είναι μία ελεύθερη μεταβλητή. Ανάλογα στους προτασιακούς τύπους (2) (y δεσμευμένη , χ

ελεύθερη). Σε μία ποσοδεικτική πρόταση δεν

υπάρχουν ελεύθερες μεταβλητές (όλες είναι δεσμευμένες) .

2.7. ΠΑΡΑ ΤΗΡΗΣΕΙΣ

ω Αν σ' ένα προτασιακό τύπο, π.χ. δύο μετα­βλητών p(x , y) , οι μεταβλητές χ και y διατρέχουν το ίδιο σύνολο, έστω Α, τότε πολλές φορές στα Μαθηματικά, αντί:

Vx Ε Α, Vy Ε A,p(x , y) , γράφουμε: Vx, y Ε A, p(x , y)

ή ακόμα: ρ( χ, y), 'ν' χ, y Ε Α . �}ι Τα σύμβολα 'ν' και 3 , όπως και τα: ::::> και <::::> , δεν πρέπει να γράφονται στα κείμενα, λαμβα­νόμενα ως σύμβολα στενογραφίας. Ο συμ!�ολι­'Ψ''ς ι;τΗ \ω.Ιιημυηκ{; δ�;y· ιiναι μία μορφι1 ση-

Π.χ. δεν είναι σωστό να γράφουμε εκφράσεις όπως οι εξής:

«Να δείξετε ότι 'ν' αριθμό . . . » . 2. «Να δείξετε ότι 3 ευθεία . . . » .

ΕΥΚΛΕΙΔΗΣ Β ' λθ ' τ.3/8

Page 11: Ευκλειδης Β 59

------------- Στοιχεία Μαθηματικής Λογικής -------------

.:ι «Επειδή α ::F Ο � ότι . . . » . �'. «Για u = 20 � t = 20χ » . :::'" «Αν α � ο � Ια l = α ».

c' Το σωστό είναι: 'v'' · ε:

6. « 3χ Ε IR: χ3 - 2χ + 1 = Ο » (το σύμβολο : δεν είναι σύμβολο της Μαθηματικής Λογικής» .

c Το σωστό είναι: , ΞΙ ε - 2χ + 1 = 1h ι « Vx Ε IR � χ2 � 0 » (διαβάζουμε: «αν, για κάθε

χ Ε IR , τότε χ 2 � Ο ». Έχει νόημα;) . " Το σωστό είναι: < < 'v'x ε::

2.8. ΘΕΣΗ ΤΩΝ ΠΟΣΟΔΕΙΚΤΩΝ ΣΕ ΜΙΑ

ΠΟΣΟΔΕΙΚΤΙΚΗ ΠΡΟΤΑΣΗ.

"' ! Λ\' ('!.Τ ι� iH :;τοηο,i)εηπικ:i� ρu.{:χε�. �-:)(Jι,. �·�� Γ:�:μισιr;(Jτ��.Η) tJς .ι ι

Για παράδειγμα, η πρόταση : « Vx Ε JR, 3y Ε IR, χ < y »

είνυι αληΟι'�ς, ενώ η πρόταση : « 3y Ε JR, Vx Ε IR, χ < y »

�:ίνω ψι;:οδ�7� (γιατί;) . - Αποδεικνύεται (στη Μαθηματική Λογική) ότι, για κάθε προτασιακό τύπο ρ( χ, y) , οι παρακάτω προτάσεις f:'.<'ιc�.

Vx, Vy, p(x , y) <:::> Vy, Vx, p(x , y) 3x, 3y, p(x , y) <:::> 3y, 3x, p(x , y) 3χ , Vy, p(x , y) � Vy, 3x , p(x , y) 3y, Vx, p(x, y) � Vx, 3y, p(x, y) .

Αντιθέτως, οι παρακάτω προτάσεις {;J> ·'''''J τ,:,;,.,

Vx, 3y, p(x, y) � 3y, Vx, p(x , y) Vy, 3x , p(x , y) � 3χ, Vy, p(x , y) .

Για παράδειγμα, η πρόταση : ( Vx Ε JR, 3y Ε JR, χ ::F y) � (3y Ε IR, Vx Ε IR, χ ::F y)

είναι ψευδής (γιατί;) . 2.9. ΠΡΟΤΑΣΙΑΚΟΙ ΤΥΠΟΙ ΚΑΙ

ΠΡΟΤΑΣΙΑΚΟΣ ΛΟΓΙΣΜΟΣ.

Θεωρούμε τους προτασιακούς τύπους: p(x), q(x), r(x), . . . ορισμένους επί ενός συνόλου

Ω( ::F 0). Για οποιαδήποτε τιμή του χ από το Ω, καθένας από τους προτασιακούς αυτούς τύπους γίνεται μία πρόταση (αληθής ή ψευδής). Έτσι, όλοι οι νόμοι της Λογικής των προτάσεων (§1 . 1 3) εξα­κολουθούν να ισχύουν και για τους προτασιακούς αυτούς τύπους και τούτο για οποιαδήποτε τιμή του χ από το Ω. Με άλλα λόγια:

' >. �) ι ••' '' I \

Για παράδειγμα, ας θεωρήσουμε το νόμο: (p � q ) <=> (pvq ) (§1 . 1 3 , 1 6) .

Λόγω αυτού, αν p(x) και q(x) είναι δύο προ­τασιακοί τύποι με σύνολο ορισμού ένα σύνολο Ω, τότε ισχύει:

Vx Ε Ω, [p(χ) � q(x)] <:::> [ p(x) ν q(x) J. - Όμοια και για τους προτασιακούς τύπους με δύο ή περισσότερες μεταβλητές.

2.10. ΑΡΝΗΣΕΙΣ ΤΩΝ ΠΡΟΤΑΣΕΩΝ: «Vx,p (x) » ΚΑΙ «3x,p (x) ».

ο ri-•4x κάΟ:r: 1ψοτυcηω·,{, τ1JΠσ ;J'." _ .

ουν οι εξ!'jς uc·οδυνωJίe:;,:

Με άλλα λόγια: ,, Η άρνηση της πρότασης: Vx, p(x) είναι η :

3x, p(x) . θ Η άρνηση της πρότασης: 3x , p(x) είναι η :

Vx, p(x) . Πράγματι, έχουμε:

t< [ Vx, p(x) αληθής] <=> [(Vχ, p(χ)) ψευδής] <:::> [για ένα τουλάχιστο ξ Ε Ω , p( ξ) ψευδής] <:::> [για ένα τουλάχιστο ξ Ε Ω , p(ξ) αληθής] <=> [( 3x, p(x)) , αληθής] .

ΕΥΚΛΕΙΔΗΣ Β ' λθ ' τ.3/9

Page 12: Ευκλειδης Β 59

------------ Στοιχεία Μαθηματικής Λογικής ------------Συμπεραίνουμε τώρα εύκολα ότι η ισοδυναμία

(ι) ισχύει, β) [ 3 χ, ρ(χ) αληθής J � [(3 χ , ρ(χ) ) ψευδής]

� [για οποιοδήποτε ξ Ε Ω , ρ( ξ) ψευδής] � [για οποιοδήποτε ξ Ε Ω , ρ( ξ) αληθής] � [ V' χ, ρ(χ), αληθής J. Συμπεραίνουμε τώρα εύκολα ότι η ισοδυναμία

(2) ισχύει. Παραδείγματα.

1) όχι (V' x Ε IR, x2 2: ο) � (3 χ Ε IR, x2 < ο) 2) όχι (3 χ Ε Ζ, 2χ + 5 = 0) � (V' x Ε Ζ, 2χ + 5 :;t: Ο) . 3) όχι [ V'x Ε IR, ( χ2 2: ι ή l x l < ι) J �

[3 χ ε!R, όχι (χ2 2: ι ή l x l < ι)J � [3 χ ε!R, (χ2 < ι και ι χι;:::ι)] .

4) όχι [ 3 χ Ε IR, (χ < ο και χ 2 = ι) J � [ V' x ε ΙR,όχι ( χ < Ο και χ2 = 1) J � [ V' χ Ε IR, (χ 2: ο ή χ 2 :;t: 1) J ο

5) όχι (V' x EIR, x >ι� χ2 > ι) � [3 χ Ε IR, όχι (χ > l� χ2 > l)J � [ 3 χ Ε IR, {χ > 1 και χ 2 � 1) J .

2.11. ΑΡΝΗΣΕΙΣ ΠΡΟΤΑΣΕΩΝ ΜΕ

ΠΕΡΙΣΣΟΤΕΡΟΥΣ ΠΟΣΟΔΕΙΚΤΕΣ.

Με βάση τις ισοδυναμίες που είδαμε στην προηγούμενη παράγραφο, έχουμε π.χ. :

V' x,3 y, ρ(x , y) � V' x, [3 y, ρ(x , y)] � 3 x, [3 y, ρ(x , y)] � 3 χ, V' y, ρ(x , y) .

Δηλαδή, η άρνηση της πρότασης: V' x,3 y, ρ(x, y)

είναι η πρόταση : 3 χ, V' y, ρ(x , y) . Γενικότερα:

• Για να σχηματίσουμε την άρνηση μιας

ποσοδεικτικής πρότασης, aντικαθιστούμε

το V' με το 3 , το 3 με το V' και τον προ-

τασιακό τύπο, που ακολουθεί, με την άρ­

νησή του [δηλαδή, αν p(x,y, .. . ) είναι ο

προτασιακός τύπος, που ακολουθεί, τον

aντικαθιστούμε με τον: p(x,y, • • • ) ].

Παραδείγματα. 1) Η άρνηση της πρότασης:

V' x Ε N,3 y Ε Ζ, χ + y = 5 (αληθής) είναι: 3 χ Ε Ν, V' y Ε Ζ, χ + y :;t: 5 (ψευδής) .

2) Η άρνηση της πρότασης: V' α Ε IR,3 κ Ε Ζ, κ � α< κ+ 1 (αληθής) είναι: 3 α Ε IR, V' κ ε Ζ, ( κ > α ή α 2: κ + ι) (ψευδής) .

3) Ο γενικός ορισμός της περιοδικής συνάρτησης είναι ο εξής:

Ορισμός Μια συνάρτηση f:A�JR(A:;t:Θ,AςJR) ονομάζε-

ται περιοδική αν, και μόνο αν, υ­

πάρχει Τ ε IR * τέτοιος, ώστε για

κάθε χ Ε Α να ισχύουν: (χ+ Τ) ε Α και f {x+ Τ) =f (x) . Ο αριθμός Τ ονομάζεται, τότε, μία

περίοδος της f. Τώρα θέλουμε να μάθουμε πότε μία συνάρτηση

δεν είναι περιοδική. Η απάντηση δεν πρέπει να πε­ριέχει αρνήσεις, γιατί τότε δε σχηματίζουμε την άρ­νηση και στην ουσία δεν απαντάμε στο ερώτημα.

Λοιπόν, για να απαντήσουμε στο ερώτημα πρώτα θα πρέπει να γράψουμε τον παραπάνω ορι­σμό στη συμβολική του μορφή, δηλαδή : (f : A� IR)

περιοδική �

[ 3Τ ε!R* , V'x ε Α,( χ + Τ) ε Α και f{ χ + Τ) =f( χ) J. Έχουμε λοιπόν:

( f : Α � IR δεν είναι περιοδική) � � [ V'τEIR*,3xEA, (x +T)�A ήf(x +T):;t:f(x)] .

4) Έστω μία συνάρτηση f : Α� IR ορισμένη κο­ντά στο χ0 Ε IR . Θέλουμε να μάθουμε πότε lim f (χ ) :;t: .e , όπου .e Ε IR . Για να απαντήσουμε

Χ-Η0

στο ερώτημα πρώτα θα πρέπει να γράψουμε συμ­βολικά τι σημαίνει lim f (χ ) = .e .

χ�χ. Έχουμε:

lim f (x ) =.e � Χ�Χο

ΕΥΚΛΕΙΔΗΣ Β ' λθ ' τ.3/10

Page 13: Ευκλειδης Β 59

------------ Στοιχεία Μαθηματικής Λογικής-----------

[ 'v'ε >O, ΞJδ >O, 'v'xEA,O< Ix -x0l < δ� jf (x) - lj < ε J Έτσι, έχουμε: lim f (χ )* l <=>

χ�χ. [ ΞJε >Ο, 'v'δ >Ο,ΞJχ EA,O< Ix -x0l < δ και jf (χ) - ψ:�:ε J

2.12. ΜΕΡΙΚΟΙ ΑΛΛΟΙ ΧΡΉΣΙΜΟΙ ΠΟΣΟΔΕΙΚΤΙΚΟΙ ΝΟΜΟΙ.

1) Έστω ότι p(x) και q(x) είναι δύο προτασιακοί τύποι ορισμένοι επί ενός συνόλου Ω. Οι παρακάτω προτάσεις είναι αληθείς :

• [ Vx, (p(x) και q(x)) J <=> [(vx, p(x)) και (Vx, q(x))]

• [ ΞJx, (p(x) ή q(x) ) ] <:::::> [ (ΞJχ, p(x)) ή (ΞJx, q(x)) J

• [(vx, p(x)) ή (Vx, q(x))]� [ Vx, (p(x) ή q(x)) J

• [ ΞJx, (p(x) και q(x)) J � [(ΞJx, p(x)) και (ΞJx, q(x) ) ]

Αντιθέτως, ο ι παρακάτω προτάσεις δεν είναι πά­

ντοτε αληθείς :

• [vx, (p(x) ή q(x) ) J� [(vx, p(x)) ή (Vx, q(x)) ]

• [(ΞJx, p(x)) και (ΞJx, q(x) ) ] � [ ΞJx, (p(x) και q(x)) J

Π.χ. με Ω = {1, 2} , οι παρακάτω δύο προτάσεις είναι ψευδείς (γιατί;): i) Vx Ε Ω, ( χ - 1 = 0 ή χ - 2 = 0)�

(Vx Ε Ω, χ - 1 = 0) ή (Vx Ε Ω, χ - 2 = ο) ii) (ΞJχΕΩ, χ - 1 = 0) και (ΞJχΕΩ, χ-2 = 0)�

ΞJχ Ε Ω, (χ - 1 = Ο και χ - 2 = Ο) Έτσι, δεν ισχύουν πάντοτε οι ισοδυναμίες:

• [ Vx, (p(x) ή q(x)) J <=> [(vx, p(x)) ή (Vx, q(x))] .

• [ ΞJx, (p(x) και q(x)) J <=> [ (ΞJχ, p(x)) και (ΞJx , q(x)) J .

2) Έστω ότι p και q (χ ) I Ω είναι δύο προτασιακοί τύποι, από τους οποίους ο p δεν περιέχει ελεύ­θερες εμφανίσεις του χ. Τότε, οι παρακάτω

προτάσεις είναι αληθείς:

• [vx,(p και q(x))] <=> [p και (Vx, q(x))] • [ Vx, (p ή q(x)) J <=> [Ρ ή (Vx, q(x)) J • [ΞJx , (p και q(x))] <=> [p και (ΞJx, q(x))] • [ΞJx,(p ή q(x) )] <=> [Ρ ή (ΞJx, q(x) )] .

Π.χ. έχουμε: Vx Ε JR., ΞJκ Ε Ν, ( 2κ < 2 ή χ < κ) <:::::> Vx Ε JR., [ ΞJκ Ε Ν, ( 2κ < 2 ή χ < κ) J <:::::> 'v'χΕ1R, [(3κΕΝ, 2κ < 2) ή (ΞJκΕΝ, χ < κ)]<=> (3κΕΝ, 2κ < 2) ή (VxEJR., ΞJκEN, x < κ) .

ΑΣΚΗΣΕΙΣ 6) Να εξετάσετε αν είναι αληθής η πρόταση:

ΞJΤ Ε JR.:, Vx Ε JR., ημ( χ + Τ)2 = ημχ2• 7) Να εξετάσετε αν είναι αληθής η πρόταση:

ΞJΤ Ε JR.:, Vx Ε JR., ΞJκ Ε Ζ, 6χ + 3Τ = (2κ + l )π (π = 3, 14 ... ) .

8) Με τ Ε JR. * ' να δείξετε ότι: Vx Ε JR., ημ3( χ + Τ) = ημ3χ <=>τ Ε { 2;π I κ Ε z*}.

9) Έστω ότι p και q (χ ) I Ω είναι δύο προτασιακοί τύποι, από τους οποίους ο p δεν περιέχει ελεύθερες εμφανίσεις του χ. Να δείξετε ότι οι παρακάτω προτάσεις είναι αληθείς:

α) [vx , (p � q(x) ) ] <=> [Ρ� (Vx, q(x) ) ] β) [ Vx, ( q(x) � p) J <=> [ (ΞJx, q(x)) � p J γ) [ ΞJx, (p � q(x)) J <=> [Ρ� (ΞJx, q(x)) J δ) [ΞJχ , ( q(x) � p )] <=> [(vx, q(x)) � p J.

1 Ο) Με τις υποθέσεις της προηγούμενης ά­σκησης, να δείξετε ότι οι παρακάτω προτάσεις εί­ναι αληθείς:

α) [ Vx, ( q(x) <=> p) J � [ (Vx, q(x)) <=> p J β) [ (ΞJx, q(x)) <=> p J � [ ΞJχ, ( q(x) <=> Ρ)] .

(Συνεχίζεται)

ΕΥΚΛΕΙΔΗΣ Β ' λθ ' τ.3/1 1

Page 14: Ευκλειδης Β 59

- u . . ... · ΓΙ Επιμέλεια: Σωτήρης Ε. Λουρίδας

66ος ΠΑΝΕΛΛΗΝΙΟΣ ΜΑΘΗΤΙΚΟΣ ΔΙΑΓΩΝΙΣΜΟΣ ΣΤΑ ΜΑΘΗΜΑΤΙΚΆ

(I) "0 ΕΥΚΛΕΙΔΗΣ" ΣΑΒΒΑΤΟ, 21 IANOYAPIOY 2006

ΘΕΜΑΤΑ

Α' ΛΥΚΕΙΟΥ 1. Έστω ότι οι ακέραιοι αριθμοί α και α + 2

είναι πρώτοι με α> 3. Να αποδειχθεί ότι ο

αριθμός α + 4 είναι σύνθετος.

Λύση Ένας από τους αριθμούς α, α + 1 , α + 2 διαιρείται με τον 3 και αυτός πρέπει να είναι ο α + 1 αφού οι α, α + 2 είναι πρώτοι. Άρα ο αριθμός α + 4 = α + 1 + 3 διαιρείται με τον 3. (Γνωρίζουμε ότι το γινόμε­νο τριών διαδοχικών αριθμών διαιρείται με το 3). 2. Οι αριθμοί α και β είναι θετικοί και ισχύει

α+ β =λ. Να δεχθεί ότι

Έχουμε

4 1 1 3 - � --+ -·- <-. 3λ α + λ β + λ 2λ

Λύση

1 1 3λ -- +--= . α+λ β+λ αβ+2λ2

Η πρώτη ανισότητα είναι ισοδύναμη με 4αβ � (α + β)2 , που είναι αληθής πρόταση

και η δεύτερη με 3αβ > Ο που είναι αληθής πρόταση αφού

οι α, β είναι θετικοί αριθμοί. 3. Έστω ΑΒΓ ένα σκαληνό τρίγωνο. Πόσα

σημεία Δ υπάρχουν στο επίπεδο του τριγώ­

νου τέτοια ώστε το τετράπλευρο με κορυ­

φές τα σημεία Α, Β, Γ, Δ να έχει άξονα

συμμετρίας διαφορετικό από πλευρά του

τριγώνου;

Λύση Το Δ πρέπει να είναι συμμετρικό κορυφής του τρι­γώνου ως προς τη μεσοκάθετο της απέναντι πλευ­ράς. Άρα υπάρχουν 3 τέτοια σημεία.

4. Έστω Α και Β δύο μη κενά και ξένα μεταξύ

τους σύνολα των οποίων η ένωση είναι το

σύνολο {1, 2, 3, 4, 5}. Να αποδειχθεί ότι ένα

τουλάχιστον από τα Α και Β περιέχει του­

λάχιστον τη διαφορά δύο στοιχείων του.

Λύση Έστω ότι κανένα από τα δύο σύνολα δεν περιέχει τη διαφορά δύο στοιχείων του. Τότε προφανώς το 2 δεν μπορεί να ανήκει στο ίδιο σύνολο με το 1 ούτε με το 4 γιατί 2 - 1 = 1 και 4 -2 = 2. Έστω λοιπόν 2 Ε Α, οπότε 1 Ε Β και 4 Ε Β. Επειδή 4 - 1 = 3, έπεται ότι 3 �Β και επομένως 3ΕΑ. Επειδή 5 - 2 = 3, έπεται ότι 5 �Α και επειδή 5 - 1 = 4, έπεται 5�Β. οπότε 5!ϊ!ΟΑυΒ. Άτοπο επειδή Α υ Β= { 1, 2, 3, 4, 5}.

ΕΥΚΛΕΙΔΗΣ Β ' λθ ' τ.3/12

Page 15: Ευκλειδης Β 59

Μαθηματικοί Διαγωνισμοί - Μαθηματικές Ολυμπιάδες

Β' ΛΥΚΕΙΟΥ

1. Υπάρχει θετικός ακέραιος ν τέτοιος ώστε: Α) Ο 3ν είναι τέλειος κύβος, ο 4ν τέλεια τέ­

ταρτη δύναμη και ο 5ν τέλεια πέμπτη δύναμη;

Β) Ο 3ν είναι τέλειος κύβος, ο 4ν τέλεια τέ­ταρτη δύναμη, ο 5ν τέλεια πέμπτη δύνα­μη και ο 6ν τέλεια έκτη δύναμη;

Λύση

Α) Ναι, π.χ. ν = 230330524. Β) Όχι. , διότι, αν υπήρχε, τότε 2lν. Έστω α ο μεγα-

λύτερος εκθέτης τέτοιος ώστε 2αlν. Τότε 4lα +2 (λόγω του ότι ο 4ν είναι τέλεια τε­τάρτη δύναμη) και 2lα + 1 (λόγω του ότι ο 6ν είναι τέλεια έκτη δύναμη) και άρα 211, που εί­ναι άτοπο.

Λύση

Προφανώς ΑΒ//ΓΔ και ΒΓ//ΔΕ επομένως: ΑΓ = ΒΔ = ΕΓ.

Επίσης έχουμε ΒΓ = ΑΔ, ΔΓ = ΒΕ και ΑΒΕ = ΑΔΕ = 90° .

Άρα ΑΒ2 + ΓΔ2 = ΑΒ2 + ΓΔ2 = ΑΕ2 =

= ΑΔ2 + ΔΕ2 = ΒΓ2 + ΔΕ2. 4. Μια πραγματική συνάρτηση f είναι ορισμέ­

νη στο ffi. με την ιδιότητα: f(x + 1)f(x)+f(x + 1)+ 1 = f(x)

Να δειχθεί ότι για κάθε χ ε ffi. ισχύουν: ι. r( χ) :;t -1, 2. f(x) :;t Ο 3. f(x+4)=f(x)

Λύση

2. Να βρεθούν πραγματικοί αριθμοί χ, y, z, w 1) Αν υπάρχει τουλάχιστον ένας α ε ffi. ώστε για τους οποίους ισχύει f(a) = - 1 , τότε 2 =Ο άτοπο.

�χ_ y + �Υ_ z + -Jz _ w + -Jx + w = χ+ 2. 2) Αν υπάρχει τουλάχιστον ένας β ε ffi. ώστε

Λύση

Η σχέση είναι ισοδύναμη με (�x - y - 1)2 + (�y - z - 1)2 +

(-Jz - w - 1)2 +(-Jx + w - 1)2 = 0 και άρα χ = 2, y = 1 , z = Ο, w = - 1 .

f(β) = Ο, τότε f(β + 1 ) = -1 . Άτοπο από την προηγούμενη σχέση .

3) Έχουμε f(x + 1) = (f(x) - 1 )/(f(x) + 1 ) , f(x + 2) = - 1 /f(x) και επομένως

f(x + 4) = -1/f(x + 2) = f(x) .

Γ ΛΥΚΕ ΙΟΥ

3. Οι κορυφές Α, Β, Γ, Δ, Ε μιας τεθλασμένης 1. Για μια συνάρτηση f: R � R ισχύει γραμμής βρίσκονται πάνω σε ένα κύκλο όπως στο σχήμα έτσι ώστε οι γωνίες ΑΒΓ ,

Λ Λ ΒΓ Δ , Γ ΔΕ να είναι ίσες με 45°. Ν α απο-δειχτεί ότι

ΑΒ2 + ΓΔ2 = ΒΓ2 + ΔΕ2

,' -'

Δ I / I /

�-/1 / I

....... "....... / ' I I I I I I I I I

Β

f(f(x)) = χ3 -2χ2 + 3χ -1,

για κάθε χ ε R . Α) Να βρεθεί το f(1) .

Β) Να εξετασθεί αν η συνάρτηση g(x) = χ3 + x2f(x)- 2xf2 (χ)+ 3

είναι 1-1.

Λύση

Ισχύει f(f( l )) = 1 επομένως f(l) = f(f(f(l))) =(f( l ))3 - 2(f( l ))2 + 3f(l) - 1.

Λύνοντας ως προς f( l ) έχουμε f( l ) = 1. Επίσης, g(O) = g( l ) = 3, οπότε η g δεν είναι 1-1 . 2. Έστω α, β θετικοί ακέραιοι τέτοιοι ώστε

ΕΥΚΛΕΙΔΗΣ Β ' λθ ' τ.3/13

Page 16: Ευκλειδης Β 59

Μαθηματικοί Διαγωνισμοί - Μαθηματικές Ολυμπιάδες

Ν α αποδειχθεί ότι

JS - α > _l_ β 4αβ

Λύση

Έχουμε G α 1 G 1 v::J -- > -<=>ν5β > α+- <=> β 4αβ 4α

2 2 1 1 2 2 1 1 <::> 5β > α + -+--<::> 5β - α > - +--2 16α2 2 16α2 ι 1 1 1 9 με - +-- � -+ -= - < 1 2 16α2 2 16 16

Άρα αρκεί 5β2 - α2 � 1 που αληθεύει αφού ο 5β 2 - α 2 είναι θετικός ακέραιος και αυτό επειδή

.J5β > α => 5β2 - α2 > Ο με α, β ακέραιους 3. Έστω ΑΒΓ Δ κυρτό τετράπλευρο τέτοιο

ώστε ΑΔ = ΒΓ, ΑΔ μη παράλληλη προς τη ΒΓ και Ο το σημείο τομής των διαγωνίων ΑΓ και ΒΔ. Να αποδειχθεί ότι υπάρχει ση­μείο Ρ διάφορο του Ο τέτοιο ώστε ο λόγος των εμβαδών των τριγώνων ΡΒΔ και ΡΑΓ

να ισούται με το τετράγωνο του λόγου των πλευρών ΡΒ και ΡΑ αντίστοιχα.

Λύση

Θεωρούμε τους περιγεγραμμένους Κι και Κ2 στα τρίγωνα ΟΑΔ και ΟΒΓ με διαφορετικά σημεία

τομής Ο και Ρ λογω του ότι η πλευρά ΑΔ δεν είναι παράλληλη προς την πλευρά ΒΓ. Επειδή οι γωνίες ΑΟΔ και ΒΟΓ είναι ίσες ως κατά κορυφήν κα­θώς και οι πλευρές ΑΔ , ΒΓ από την υπόθεση, οι

Δ κύκλοι Κι και Κ2 είναι ίσοι και τα τρίγωνα ΡΒΔ ,

Δ Ρ Α Γ είναι όμοια. Επομένως ο λόγος των εμβα-δών τους ισούται με το τετράγωνο του λόγου ο­μοιότητας. 4. Έστω 2ν > κ και έστω ότι οι ακέραιοι α­

ριθμοί α1, α2, . . . , «ν αφήνουν διαφορετικά υπόλοιπα όταν διαιρεθούν δια του κ. Να αποδειχθεί ότι για κάθε ακέραιο αριθμό λ

υπάρχουν δείκτες i, j από το σύνολο {1, 2, . . . , ν} τέτοιοι ώστε

κlα;+αj λ Λύση

Οι αριθμοί λ - αι, λ - α2, • • • , λ - «ν αφήνουν δια­φορετικά υπόλοιπα όταν διαιρεθούν δια του κ. Ο μέγιστος αριθμός αυτών των υπολοίπων είναι κ. Επειδή 2ν > κ, δύο από τους 2ν αριθμούς α ι. α2, • • • , αv, λ - α ι, λ - α2, • • • , λ - «ν θα αφήνουν το ίδιο υ­πόλοιπο όταν διαιρεθούν δια του κ και δεν μπο­ρούν να ανήκουν στο ίδιο σύνολο {λ - αι, λ - α2, . . . , λ - «ν} ή {αι. α2 . . . . , «ν}, δηλ. ο ένας θα είναι κάποιος αί και ο άλλος κάποιος λ - αj . Η διαφορά τους διαιρείται με τον κ.

66ος ΠΑΝΕΛΛΗΝΙΟΣ ΜΑΘΗΤΙΚΟΣ ΔΙΑΓΩΝΙΣΜΟΣ ΣΤΑ ΜΑΘΗΜΑΤΙΚΆ

(11) ''Ο ΑΡΧΙΜΗΔΗΣ" l:ΑΒΒΚΓΟ, 25 ΦΕΒΡΟΥ ΑΡΙΟΥ 2006

ΘΕΜΑΊΆ ΜΕΓ ΑΛΟΙ

Ι. Πόσοι διαφορετικοί πενταψήφιοι θετικοί ακέραιοι αριθμοί υπάρχουν που το καθένα από τα ψηφία τους, εκτός του τελευταίου, είναι μεγαλύτερο ή ίσο του επομένου ψηφί­ου τους;

Λύση

Από τους συνδυασμούς με επαναλήψεις των 1 Ο ψηφίων ανά 5 πρέπει να αφαιρέσουμε το συνδυα­σμό 00000. Άρα η απάντηση είναι (14

J 10 . 1 1 . 12 . 13 ·14 -1 = -1 = 2001. 5 1·2·3·4·5

ΕΥΚΛΕΙΔΗΣ Β ' λθ ' τ.3/14

Page 17: Ευκλειδης Β 59

Μαθηματικοί Διαγωνισμοί - Μαθηματικές Ολυμπιάδες

2. Έστω n ένας θετικός ακέραιος αριθμός. Να αποδειχθεί ότι η εξίσωση

x+y +!+!=3n χ Υ

δεν έχει λύσεις στο σύνολο των θετικών ρη­τών αριθμών.

Λύση Έστω χ = κ/λ και y = μ/ρ με (κ,λ) = 1 και (μ,ρ) = 1 μία πιθανή λύση. Τότε θα έχουμε:

ρμ(κ2 + λ2) + λκ(ρ2 + μ2) = 3ηρμλκ . ( 1 ) Άρα ρμlλκ(ρ2 +μ2) και επειδή (ρμ, μ2 + ρ2) = 1 (αφού (μ,ρ) = 1 ) , έχουμε ρμlλκ. Παρόμοια έχουμε λκlρμ, οπότε λκ = ρμ. Από την ( 1 ) συμπεραίνουμε ότι

3 lκ2 + λ2 + ρ2 + μ2, και επειδή (κ,λ) = 1 , (μ,ρ) = 1 και λκ = ρμ, μεταξύ των κ, λ, μ, ρ δύο ή κανένας θα διαιρείται δια του 3, οπότε κ2 + λ2 + ρ2 + μ2 Ξ 1 ή 2 (mod 3). Άτοπο.

Ά ΡΚ _

ΡΒ ( 1 ) ρα - - -ΑΛ ΑΒ Επίσης από τα όμοια τρίγωνα ΝΡΓ και ΛΑΓ έχου­με

ΡΓ . ΜΒ .ΝΑ = 1 ΡΒ ΜΑ ΝΓ Από το Θεώρημα των Διχοτόμων έχουμε

ΒΛ ΑΒ ΛΓ ΑΓ

Από ( 1 ) , (2), (3) έχουμε ΡΝ ΡΓ•ΒΛ ΡΓ•ΒΛ

= = ---

ΡΚ ΡΒ•ΑΓ ΡΒ•ΛΓ

(2) .

(3)

(4)

Αρκεί λοιπόν τα σημεία Ρ, Β, Λ, Γ να αποτελούν αρμονική τετράδα. Αυτό ισχύει γιατί Α) Στο τρίγωνο ΑΒΓ από το Θεώρημα του Ceνa

, ΒΛ ΝΓ ΑΜ ισχυει -•-•- = 1 δηλ. ΛΓ ΝΑ ΜΒ ΒΛ ΝΑ ΜΒ

- = -·-- (5) . ΛΓ ΝΓ ΑΜ 3. Δίνεται τρίγωνο ΑΒΓ και τα σημεία Λ, Μ, Β) Στο τρίγωνο ΑΒΓ με διατρέχουσα την ΡΜΝ

Ν των πλευρών ΒΓ, ΑΒ και ΑΓ αντίστοιχα, έτσι ώστε η ΑΛ να είναι διχοτόμος της γω­νίας Α και οι ΒΝ και ΓΜ να διέρχονται από ένα κοινό σημείο Σ της ΑΛ. Αν η γωνία ΑΛΒ ισούται με τη γωνία ΑΝΜ, να αποδει­χθεί ότι η γωνία ΜΝΑ είναι ορθή.

Λύση 1 ''ς τρόπος Θα δείξουμε ότι η Ρ Λ είναι διάμετρος του κύκλου που περιγράφεται στο τετράπλευρο ΛΝΑΡ ( <rANP = <rΑΛΡ ). Έστω Κ το σημείο τομής της ΑΒ με τον κύκλο. Επειδή <rΚΡ Λ = <rΚΑΛ και <rΛRN = <rΛΑΝ , έχουμε <rΚΡ Λ = <rΛΡΝ και επομένως αρκείνα δείξουμε ότι ΡΝ = ΡΚ. Τα τρίγωνα ΡΒΚ και ΑΒΛ είναι όμοια.

Α �Ν

(Θεώρημα του Μενελάου) έχουμε ΡΓ ΝΑ ΜΒ , ΡΓ ΜΑ ΝΓ -•-•- = 1 και αρα - = -•- (6) ΡΒ ΝΓ ΜΑ ΡΒ ΝΑ ΜΒ Από τις (4), (5) και (6) έχουμε ΡΝ = ΡΚ.

2"ς τρόπος

Το τετράπλευρο ΑΜΣΝ είναι πλήρες, αφού οι πλευρές του ΑΜ και ΝΣ τέμνονται στο Β και οι πλευρές του ΑΝ και ΜΣ στο Γ. Από γνωστό θε­ώρημα η τομή Ρ της ΝΜ με την ΒΓ είναι σημείο αρμονικό συζυγές του Λ ως προς τα Β, Γ με Λ το ίχνος της εσωτερικής διχοτόμου ΑΛ. Άρα η ΑΡ είναι η εξωτερική διχοτόμος του τριγώνου ΑΒΓ. Επομένως Ρ λλ = 90° οπότε από το εγ-γράψιμο τετράπλευρο ΑΡ ΛΜ ( ΑΝΜ = ΑΑΒ) έχουμε: ΡΝΛ = ΡΑΑ = 90°

4. Να εξετάσετε αν υπάρχει συνάρτηση f : R� τέτοια ώστε να ικανοποιεί τις συνθήκες 1) f(x+y+z)�3(xy+yz+zx) για όλους

τους πραγματικούς αριθμούς χ, y, z και 2) υπάρχει φυσικός αριθμός ν και συνάρ­

τηση g τέτοια ώστε

ΕΥΚΛΕΙΔΗΣ Β ' λθ ' τ.3/15

Page 18: Ευκλειδης Β 59

Μαθηματικοί Διαγωνισμοί - Μαθηματικές Ολυμπιάδες

g(g(x))=x2v+l και f(g(x))=(g(x))2 για κάθε πραγματικό αριθμό χ.

Λι'Jση Είναι προφανές ότι η συνάρτηση g είναι 1 - 1 και επί και άρα υπάρχει η aντίστροφός της. Επομένως:

f(x) = f(g(g-Ι (χ)) = (g(g-Ι (χ))2 = χ2 για κάθε πραγματικό αριθμό χ και συνεπώς για τους πραγματικούς χ, y, z έχουμε

δηλαδή

3(xy + yz + zx) s (χ + y + z) 2 = =f(x + y + z) s 3(xy + yz + zx)

f(x + y + z) = 3(xy + yz + zx) αλλά τότε, θέτοντας (χ, y, z) = (0, Ο, Ο) και (χ, y, z) = (0, 1 , - 1 ) , λαβαίνουμε

f(O) = Ο και f (Ο) = -3 αντίστοιχα. Άρα δεν υπάρχει τέτοια συνάρτηση .

.� ?ΕΑ\Γ[l!

Μιχαήλ Θ. Ρασσιάς Φοιτητής Ι ou έτους της Σχολής Ηλεκτρολόγων Μηχανικών και

Μηχανικών Υπολογιστών του Ε.Μ .Π.

«Για κάθε πρώτο αριθμό p > 2 να εξετάσετε αν υπάρχουν πρώτοι αριθμοί ρ , , Pz με Ρι < Pz τέτοιοι ώστε οι αριθμοί (p - 1)p1 , p2 να είναι διαδοχικοί» .

Ο Leonhard Euler ( 1707 - 1783) απέδειξε πως έ­νας θετικός ακέραιος αριθμός μπορεί να εκφραστεί ως άθροισμα δυο τετραγώνων ακεραίων αριθμών μόνο αν ικανοποιούνται συγκεκριμένες συνθήκες.

Έχει επίσης αποδειχθεί πως δεν είναι δυνατόν κάθε θετικός ακέραιος να εκφραστεί σαν άθροισμα τριών τετραγώνων ακεραίων αριθμών.

Ο Joseph - Louis Lagraηge ( 1 736 - 1 8 1 3) απέ­δειξε πως κάθε θετικός ακέραιος αριθμός μπορεί να εκφραστεί σαν άθροισμα τεσσάρων τετραγώ­νων ακεραίων αριθμών.

Στις αρχές του 20ου αιώνα ο Daνid Hilbert ( 1 862- 1943) απέδειξε πως για κάθε θετικό ακέραιο η υπάρχει ένας αριθμός Kn τέτοιος ώστε κάθε θετι­κός ακέραιος να μπορεί να γραφεί σαν άθροισμα όχι περισσότερων από Kn θετικών η-οστών δυνά­μεων.

Από την ταυτότητα του Lagraηge έχουμε: ( xt + χ; + xj + χ� ) ( Υ� + Υ; + yj + Υ� ) -(χ ι Υι + XzYz + Χ3Υ3 + Χ4Υ4 )2

χ3 12 + lx,

Υ3 Υ ι + ι;: ;: Ι' + ι;: ;: Ι ' + ι;: ;: Ι' = (ΧιΥz - ΧzΥι )2 + (Χ ι Υ3 - Χ3Υι )2 + (Χι Υ4 - Χ4Υι )2 +(XzY3 - X3Yz )2 + (XzY4 - X4Yz )2 + (x3y4 - x4y3 )2 = (Χ ι Υz - ΧzΥ ι + Χ3Υ4 - x4y3 )2 -2(Χ ι Υz - ΧzΥ ι )(Χ3Υ4 - x4y3 ) +(Χ ι Υ3 - X3Yt + Χ4Υ2 - x2y4 )2 +2(Χ ι Υ3 - χ3yι )(Χ2Υ4 - x4y2 ) 2 +(Χ ι Υ4 - X4Yt + Χ2Υ3 - x3y2 ) -2(Χ ι Υ4 - χ4yι )(Χ 2Υ3 - x3y2 ) = (Χ ι Υ2 - χ2Υι + x3y4 - Χ4Υ3 )2 +(Χ ι Υ3 - Χ3Υι + Χ4Υ2 - Χ2Υ 4 )2 +(Χ ι Υ 4 - Χ4Υ ι + XzY3 - Χ3Υ2 )2 -2 [(Χ ι Υ2 - χ2y ι )(Χ3Υ4 - x4y3 ) -(Χ ι Υ3 - Χ3Υι )(x2y 4 - Χ4Υ2 ) +(Χ ι Υ4 - χ4yι )(Χ2Υ3 - Χ3Υ2 )] � (χ� + χ; + xj + χ� ) ( Υ� + Υ; + yj + Υ� ) = (Χ ι Υ ι + Χ2Υ2 + Χ3Υ3 + Χ4Υ 4 )2

ΕΥΚΛΕΙΔΗΣ Β ' λθ ' τ.3/16

Page 19: Ευκλειδης Β 59

Μαθηματικοί Διαγωνισμοί - Μαθηματικές Ολυμπιάδες

+ (Χ ι Υ2 - χ2Υι + Χ3Υ4 - Χ4Υ3 )2 2 +(Χ ι Υ3 - Χ3Υι + Χ4Υ2 - x2y4 )

+(Χ ι Υ4 - Χ4Υι + Χ2Υ3 - Χ3Υ2 )2 ·

Συνεπώς, προκύπτει ότι το γινόμενο δύο ακέραιων αριθμών, που ο καθένας από αυτούς γράφεται ως άθροισμα τεσσάρων τετραγώνων ακέραιων αριθμών, είναι ακέραιος αριθμός που γράφεται επίσης σαν άθροισμα τεσσάρων τετραγώνων ακεραίων αριθμών. Είναι γνωστό πως κάθε ακέραιος αριθμός μπορεί να είναι πρώτος ή να γράφεται ως γινόμενο δυνά­μεων πρώτων αριθμών. Αρκεί, λοιπόν, να αποδείξουμε ότι κάθε πρώτος αριθμός p γράφεται ως άθροισμα τεσσάρων τετρα­γώνων ακέραιων αριθμών και αυτό διότι αν ένας πρώτος αριθμός είναι άθροισμα τεσσάρων τετρα­γώνων ακέραιων αριθμών τότε και κάθε θετική ακέραιη δύναμή του γράφεται επίσης σαν άθροι­σμα τετραγώνων τεσσάρων ακέραιων αριθμών (αποδεικνύεται εύκολα). 1 : Για την ειδική περίπτωση όπου p = 2 έχουμε:

2 = 12 + 12 + 02 + 02 . 2 : Θα δείξουμε ότι για κάθε πρώτο αριθμό p >2

υπάρχουν ακέραιοι αριθμοί χ, y τέτοιοι ώστε: χ2 + y2 + 1 = mp όπου Ο < m < p .

Θεωρούμε τα σύνολα: Α = {χ2 : χ = 0, 1, . . . , Ρ� 1} (πλήθος στοιχείων Ρ; 1 )

Β = { -1 - / : y =0, 1, . . , Ρ�1} (πλήθος στοιχείων p;1 ) Δεν υπάρχει ζεύγος στοιχείων του Α που τα στοι­χεία του ζεύγους αυτού να είναι ισότιμα modp, διότι αν υπήρχε τουλάχιστον ένα τέτοιο ζεύγος ( χ� , χ� ) θα ίσχυε: χ� = x� (mod p) <::::> χ � - :ιι: � = κp, κ Ε Ζ <::::> (χ ι - χ2 )(χ ι + χ2 ) = κp, κ Ε Ζ , οπότε p I (χ ι - χ2 ) ή p I Χ ι + χ2 . Άτοπο, διότι: Αν p l (x ι - χ2 ) => χ ι - χ2 = ΚιΡ• Κ Ε Ζ => l χ ι - χ2 1 � Ρ

χ < Ε. } ι 2 Όμως => Χι + χ2 < p => lx ι - x2 l < p .

χ < Ε. 2 2 Αν p l (χ ι + χ2 ) => χι + χ2 = κ2p, κ2 Ε Ζ

:::::> Χ ι + χ2 � Ρ . χ < Ε. } ι 2 Όμως =:> χι + χ2 < p . χ < Ε. 2 2

Ομοίως στο σύνολο Β δεν υπάρχει ζεύγος στοιχεί­ων που να είναι ισότιμα modp. Το πλήθος των στοιχείων του συνόλου Α υ Β εί­ναι p+ 1 . Αυτά τα p+ 1 στοιχεία είναι διακεκριμένα. Υποθέτουμε ότι διαιρώντας κάθε έναν από αυτούς τους p+ 1 ακέραιους με το p λαμβάνουμε p+ 1 δια­φορετικά μεταξύ τους υπόλοιπα (αν ο ακέραιος είναι μικρότερος του p τότε το υπόλοιπο είναι ο ίδιος ο ακέραιος). Όμως, γνωρίζουμε ότι αν διαιρέσουμε έναν ακέ­ραιο αριθμό με το p τότε τα δυνατά υπόλοιπα είναι 0, 1 ,2 . . . ,(p - 1 ) . Συνεπώς, έχουμε το πολύ p διαφο­ρετικά υπόλοιπα. Άρα, σύμφωνα με την αρχή «της φωλιάς των περι­στεριών», υπάρχει ένα τουλάχιστον ζεύγος ακέ­ραιων αριθμών στο σύνολο Α υ Β οι οποίοι διαι­ρούμενοι με το p δίνουν το ίδιο υπόλοιπο. Έστω πως ένα τέτοιο ζεύγος ακέραιων αριθμών είναι το ( u , ν ). Τότε, σύμφωνα με τα παραπάνω, ισχύει ότι: u Ε Α και ν Ε Β καθώς δεν μπορούν και οι δύο αυτοί ακέραιοι να ανήκουν στο ίδιο σύνολο. Ισχύει, λοιπόν, ότι:

u = νmodp ή x2 = (-1 - y2 ) modp ' 2 2 1 � η χ + y + = mp, m Ε !LJ •

( 1 )2 ( - 1 )2 Όμως χ2 � Ρ� και y2 � � 2 ( p - 1 )2 Άρα χ 2 + Υ + 1 � 2 . -2- + 1

2 = (p - 1) + 1 < p2 , άρα Ο < m < p .Επομένως υ-2 πάρχουν ακέραιοι χ, y με

Ο < χ < Ε. και Ο � y < Ε. , - 2 2 τέτοιοι ώστε χ 2 + y2 + 1 = O(mod p) . Από τα παραπάνω προκύπτει ότι υπάρχουν ακέραιοι αριθμοί m, χ ι , χ2 , χ3 , χ4 ( χ ι = χ, χ 2 = y, χ3 = 1, χ4 = 0 )

' ' 2 2 2 + 2 με Ο < m < p , τετοιοι ωστε mp = Χ ι + χ2 + χ3 χ4 με τους ακέραιους αριθμούς χ ι , χ2, Χ3, Χ4 να μην

ΕΥΚΛΕΙΔΗΣ Β ' λθ ' τ.3/17

Page 20: Ευκλειδης Β 59

Μαθηματικοί Διαγωνισμοί - Μαθηματικές Ολυμπιάδες

είναι όλοι διαιρετοί με το ρ, διότι αν ήταν όλοι διαι­ρετοί με ρ τότε θα ίσχυε

mp = κ�ρ2 + κ�ρ2 + κ�ρ2 + κ�ρ2 = - (κ2 + κ2 + κ2 + κ2 )Ρ2 - I 2 3 4 •

οπότε m = (κ� + κ� + κ� + κ� ) ρ > ρ , που είναι άτοπο. Αρκεί να αποδείξουμε πως το ελάχιστο m είναι το ένα (m = 1 ) . Έστω m0 · ρ είναι το ελάχιστο πολλαπλάσιο του ρ που ικανοποιεί την ιδιότητα:

m0 · p = x� + χ� + χ� + χ� με m0 > 1 (0<m0<p). Α ν ο m0 είναι άρτιος τότε το άθροισμα

Χι + Χ2 + Χ3 + χ4 είναι άρτιος διότι αν ήταν περιττός θα ίσχυε

(χι + Χ2 + Χ3 + χ4 )2 = χ� + χ� + χ� + χ� + +2(Χ ι Χ2 + Χ ιΧ 3 + Χ ιΧ4 + Χ2Χ3 + Χ2Χ4 + Χ3Χ4 )

= ffio · p + 2(x 1x2 + χ1χ3 + χ1χ4 + +χ2χ3 + χ2χ4 + χ3χ4 )

που είναι άρτιος, άτοπο. Έτσι, για τους ακέραιους χ 1 , χ2 , χ3 , χ4 έχουμε τις περιπτώσεις: i) να είναι όλοι άρτιοι ii) να είναι όλοι περιττοί iii) να είναι οι δύο άρτιοι και οι άλλοι δύο περιττοί. Οι (i), (ii) οδηγούν αμέσως σε άτοπο. Έστω, πως ισχ6ει η (iii) με Χι ,χ2 άρτιους και χ3,χ4 περιττούς. Τότε οι αριθμοί

είναι άρτιοι. Ισχ6ει ότι:

1 ( Χ ι + χ 2 )2 ( Χ ι - χ2 )2 2mo · P = 2 + 2

+ ( χ3 ; χ4 )2 + ( χ3 ; χ4 )2 από τη γνωστή ταυτότητα

α2 ; β2 = ( α; βy + ( α;βΥ

Τα τετράγωνα ( Χι ;χ2 y .( Χι ;Χ2 Ι , ( χ3 ; χ4 Υ , ( χ3 ; χ4 Υ δεν είναι όλα διαιρετά με το ρ, διότι αν αυτό ίσχυε τότε θα είχαμε:

I Χ ι + Χ2 I Χ ι - Χ2 I Χ3 + Χ4 I Χ3 - Χ4 Ρ 2 , ρ 2 , ρ 2 , ρ 2 . (Σημείωση : Αν ρ I α · β με ρ πρώτο => ρ I α ή ρ I β , άρα αν p l α2 => p l α ). Επομένως ρ I ( Χ ι ; χ2 ) + ( Χ ι ; χ2 ) => ρ I χ 1 , ομοίως ρ I χ2 , ρ I χ3 , ρ I χ4 , άτοπο. Αφού mo άρτιος τότε m0 = 2ν , ν ε Ζ

( Χ ι + χ2 )2 ( χ 1 - χ2 )2 ::::> νp = 2 + 2 + ( Χ3 ; Χ4 Ι + ( Χ3 ; Χ4 )2 με ν < mo. άτοπο, αφού έχουμε θεωρήσει το γινόμενο m0 · ρ το ελάχιστο πολλαπλάσιο του ρ που να ανα-παρίσταται ως άθροισμα τεσσάρων τετραγώνων α­κέραιων. Αν ο mo είναι περιττός, τότε οι χ1 , χ2 , χ3 , χ4 δεν θα διαιρούνται όλοι με το m0 διότι θα ίσχυε:

λ2 2 λ2 2 λ2 2 λ2 2 mo · P = ι · mο + 2 · mo + 3 · mo + 4 · mo = = (λ� + λ� + λ� + λ� ) · m�

οπότε ρ = (λ� + λ� + λ� + λ� ) · m0 => m0 I ρ, άτοπο, αφού ο ρ είναι αριθμός πρώτος. Επειδή ο m0 είναι περιττός ας θεωρήσουμε m0 � 3 , οπότε μπορούμε να επιλέξουμε ακέραι-ους αριθμούς b ι ,b2,b3,b4 έτσι ώστε χί = bimo + Yi για i = 1 ,2,3,4

με I Yi I < � · m0 και y� + y� + y� + y� > Ο <*> ' 2 2 2 2 ( 1 2 ) 2 Επομενως Υι + Υ2 + Υ3 + Υ4 < 4 · 4mo = mo

(δ ' 2 1 2 ) ιοτι y1 < 4m0 •

ΕΥΚΛΕΙΔΗΣ Β ' λθ ' τ.3/18

Page 21: Ευκλειδης Β 59

Μαθηματικοί Διαγωνισμοί - Μαθηματικές Ολυμπιάδες

Τελικά y� + y� + y� + y� < m� ( 1 ) Έχουμε Υ� + Υ� + Υ� + Υ� = (χ ι - bιm0 )2 + (χ2 - b2m0 )2

Άρα:

+(χ3 - b3mo )2 + (χ4 - b4mo )2

= (χ� + χ� + χ� + χ� ) - 2χιbιmο

-2x2b2m0 - 2x3b3m0 - 2x4b4m0

+(b� + b� + b� + b� )% = πολλmσ . Συνεπώς y� + y� + y� + y� = Omod m0 •

{x f + χ� + χ� + χ� = m0p (m0 < p) (2)

yf + Υ� + Υ� + Υ� = mιmo (Ο < mι < m0 ) (3)

[ Ο < mι < m0 διότι αν ήταν mι � m0 τότε mιmo � m� , άτοπο λόγω της ( 1 )] . Από τις (2), (3) προκύπτει:

m�mιp = (χ� + χ � + χ� + χ� ) ( yf + Υ� + Υ� + Υ� ) 2 2 2 2 = Ζι + z2 + z3 + z4 ,

όπως παρατηρήσαμε στην αρχή του άρθρου. Όμως

zι = Χ ιΥι + Χ2Υ2 + x3y3 + x4y4 = Χ ι (χ ι - bιmo ) + Χ2 (χ2 - b2mo ) + χ3 (χ3 - b3mo )

+χ4 (χ4 - b4mo )

= (χ� + χ� + χ� +χ� ) - (χιbι + x2b2 + x3b3 + x4b4)ffio

= mσp - (χιbι + x2b2 + x3b3 + x4b4 )ffio = Omodmσ .

Ομοίως αποδεικνύεται ότι: z2 , z3 , z4 = Omodm0 .

Ά 2 2 2 2 2 2 2 2 2 ρα m0mιp = tι m0 + t2m0 + t3m0 + t4m0 ' 2 2 2 2 η mιp = tι + t2 + t3 + t4 ,

όπου ο p δεν διαιρεί όλα τα t ι ,tz,t3 ,t4 διότι αν τα διαιρούσε τότε θα ισχύει

mιp = ( ξf + ξ� + ξ� + ξ� )p2

ή mι = ( ξf + ξ� + ξ� + ξ� )Ρ > Ρ , άτοπο, διότι Ο < mι < m0 < p . Άρα έχουμε καταλήξει σε αναπαράσταση του mι · p σε άθροισμα τεσσάρων τετραγώνων με mι<mo, άτοπο. Συνεπώς, σε κάθε περίπτωση η υπόθεση mo> 1 ο­δηγεί σε άτοπο.

Επομένως ο m = 1 είναι ο ελάχιστος ακέραιος για τον οποίο το m · p είναι ελάχιστο και ισχύει

2 2 2 2 mp = Χι + Χ2 + Χ3 + Χ4 ·

για κάθε πρώτο p. Ά 2 2 2 2 ρα Ρ = Χ ι + Χ2 + Χ3 + Χ4 ·

Συνεπώς κάθε θετικός ακέραιος αριθμός μπορεί να γραφεί ως άθροισμα τεσσάρων τετραγώνων ακέ­ραιων αριθμών.

<*)Κλάση υπολοίπου [α] με το mo ορίζουμε το σύ­νολο των ακέραιων αριθμών που διαιρούμενοι με το mo δίνουν υπόλοιπο α. Προφανώς οι ακέραιοι αριθμοί Χι ,Χz,χ3,� ανήκουν σε κάποια κλάση υπολοίπων (όχι αναγκαστικά στην ίδια). Τότε αν

xi ε [α] υπάρχει Yi ε [α] , i = 1 ,2,3 ,4

Συνεπώς οι ακέραιοι Χϊ, Yi είναι ισοϋπόλοιποι modmo. Έτσι, επιλέγουμε ακέραιους αριθμούς bi τέτοιους ώστε:

1 1 1 2 2 2 2 0 με Yi < 2mo και Υι + y2 + y3 + y4 > ·

Εδώ χρησιμοποιούμε το γεγονός ότι ο m0 είναι περιττός, καθόσον αν ο m0 ήταν άρτιος θα ίσχυε m0 � 2 . Έτσι, για m0 = 2 => IYi l < 1 :::::> IYi l = 0 , άτοπο, διότι τότε θα ίσχυε xi = m0bi , δηλαδή mo I xi για i = 1, 2, 3, 4 που αποδείξαμε πως δεν αληθεύει. Α ν mo περιττός τότε m0 � 3 .

'Ετσι, για m0 = 3 => IYi I < � . Άρα μπορεί 2

IYi I = 1 * Ο το οποίο είναι δεκτό, (αφού Ο < 1 < � ) . 2

Βιβλιογραφία

[ 1 ] G. Η. Hardy and Ε.Μ. Wήght, An Introduction to the Theory of Numbers, Oxford Science Pub­lications, Clarendon Press, Oxford, 2003.

[2] J. Hunter, Αριθμοθερωρία (Μετάφραση Ν. Κριτικού), Αθήνα, 1 98 1 .

[3] Η.Ν. Wήght, First Course in Theory of Num­bers, John Wiley & Sons, Inc. , London, 195 1 .

ΕΥΚΛΕΙΔΗΣ Β ' λθ ' τ.3/19

Page 22: Ευκλειδης Β 59

�·•ιιιι•"�••• "� �· �� ��111 �-�� A•d•

Ζαχαρόπουλος Κωνσταντίνος Το παρακάτω άρθρο έχει σαν σκοπό να παρουσιάσει μια σειρά ασκήσεων που αφορούν τα συστήματα, τη

μελέτη συνάρτησης τριωνύμου 2ου βαθμού και τις εξισώσεις και ανισώσεις 2ου και ανωτέρου βαθμού.

Ο τρόπος με τον οποίο έχουν λυθεί μερικές από τις παρακάτω ασκήσεις δεν είναι ο μοναδικός, απλά διευ­

κολύνει κατά πολύ τις λύσεις τους και βοηθάει το μαθητή να αποφύγει μια ατέρμονη σειρά πράξεων.

Άσκηση 1 Να βρεθεί το είδος των ριζών της εξίσωσης

αz βz -- + -- = 1 αν α · β * Ο και χ * λ , χ * μ · χ - λ χ - μ

α2 β2 -- + -- = 1 <:::> χ - λ χ - μ

ΛΥΣΗ

α2 (χ - μ) + β2 (χ - λ) = ( χ - λ) (χ - μ) <=::> α2χ - α2μ + β2χ - β2λ = χ2 - (λ + μ) χ + μλ <=::> α2χ - α2μ + β2χ - β2λ - χ2 + (λ + μ) χ - μλ = Ο <=::> χ 2 - (α2 + β2 + λ + μ) χ + λμ + α2μ + β2λ = Ο . Βρίσκουμε τη Διακρίνουσα της δευτεροβάθμιας εξίσωσης Δ = { α2 + β2 + λ + μ )2 - 4 (λμ + α2μ + β2λ) = (α2 )2 + (β2 )2 + λ2 + μ2 + 2α2μ + 2α2β2 + 2α2λ + +2β2λ + 2β2μ + 2λμ - 4λμ - 4α2μ - 4β2λ = (α2 )2 + (β2 )2 + λ2 + μ2 + 2α2β2 + 2λα2 --2μα2 - 2λβ2 + 2μβ2 - 2λμ Προσθέτουμε και αφαιρούμε την ποσότητα 2α2β2 οπότε: Δ = (α2 )2 + (β2 )2 + λ2 + μ2 - 2α2β2 + 2λα2 - 2μα2 --2λβ2 + 2μβ2 - 2λμ + 4α2β2 = (α2 - β2 + λ - μ)2 + 4α2β2 � 0 .

Επειδή όμως αβ * Ο ισχύει Δ > Ο άρα η εξίσωση έχει δύο ρίζες πραγματικές και άνισες. Άσκηση 2

Ν α λυθεί η εξίσωση ( χ2 + χ + 1 ]2 4(2 1 ) ο + + χ + - = •

χ χ

ΛΥΣΗ (χ2 + χ + 1)2 ( 1 ) ....:..__-,...-___..:..._ + 4 2 + χ + - = ο <=::> χ2 χ (χ2 + χ + 1)2 + 4χ2 (2 + χ +�) = Ο <=:>

(χ2 + χ + 1)2 + 8χ2 + 4χ3 + 4χ = Ο <=:> (x2 + x + l)2 + 4χ {χ2 + 2χ + 1) = 0 <=:> {x2 + x + x + l - x )2 + 4χ {χ2 + 2χ + 1) = 0 <=:> {x2 + 2x + l - x )2 + 4x {x2 + 2x + l) = O <=:>

[(χ + 1)2 - χ τ + 4χ (χ + 1)2 = 0 <:::> (χ + 1)4 - 2χ (χ + 1)2 + χ2 + 4χ (χ + 1)2 = 0 <:::> (χ + 1)4 + 2χ (χ + 1)2 + χ2 = 0 <:::> [ (χ + 1 )2 + χ ]2 = ο <=::> {χ2 + 2x + l + x )2 = 0 <:::>

ΕΥΚΛΕΙΔΗΣ Β ' λθ ' τ.3/20

Page 23: Ευκλειδης Β 59

Μαθηματικά για την Α · Λυκείου

(χ2 + 3χ + 1)2 = 0 � χ2 + 3χ + 1 = 0 ο

Βρίσκουμε τη διακρίνουσα της εξίσωσης χ2 + 3χ + 1 = 0 ο

Δ = β2 - 4αγ = 32 - 4 = 9 - 4 = 5 > Ο .

Οι ρίζες της εξίσωσης λοιπόν είναι: -β+/Δ -3+J5 . -β - /Δ -3 - JS Χι = η Χ2 =

� 2 � 2 Άσκηση 3

Δίνεται η εξίσωση χ2 - { .Jλ + 3 ) χ + λ = Ο (I)

όπου λ Ε IR είναι μια πραγματική παράμετρος και ρ1 , ρ2 οι ρίζες της εξίσωσης α) Να βρεθεί για ποιες τιμές του λ Ε IR ορίζε­

ται η εξίσωση β) Να βρεθεί για ποιες τιμές του λ Ε IR η εξί­

σωση έχει ρίζες πραγματικές που ικανο-, , 4 2 2 2 2 ποιουν τη σχεση ρ1 ρ2 < ρι + ρ2 ·

ΛΥΣΗ

α) Η εξίσωση ορίζεται όταν ορίζεται η .Jλ + 3 , δηλαδή λ + 3 � Ο � λ � -3 . ( 1 )

β) Για να έχει η εξίσωση (I) ρίζες πραγματικές πρέπει Δ � Ο άρα,

2 { Ι ) Δ = ( .Jλ + 3 ) - 4λ = I λ + 31 - 4λ =

οπότε = λ + 3 - 4λ = 3 - 3λ '

3 - 3λ � ο � λ � 1 (2). Χρησιμοποιώντας τις σχέσεις του Vίeta παίρνουμε

ρ1 + ρ2 = _f = .Jλ + 3 και ρ1 • ρ2 = 1 = λ . α α

Από τη σχέση 4ρ;ρ� < ρ; + ρ� � � 4(ριρ2 )2 < (ρ ι + ρ2 )2 - 2ριρ2 � παίρνουμε 4λ2 < { .Jλ + 3 )2 - 2λ �

� 4λ2 < λ + 3 - 2λ � 4λ2 < 3 - λ � 4λ2 + λ - 3 < 0 (II).

με ρίζες λ1 = -1 και λ2 = � και λόγω της 4 σχέσης (11) θέλουμε να είναι αρνητικό, δηλαδή ετερόσημο του συντελεστή δευτεροβάθμιου όρου, που γίνεται όταν

3 -1 < λ < - (3) 4

του

Συναληθεύοντας τις σχέσεις ( 1 ) , (2) , (3) προκύπτει

-3 Ι Ι ι

-1 ο 1. 1 4

3 Συνεπώς - 1 < λ < - . 4

Δίνεται η εξίσωση

χ4 + ( μ2 + ν2 _ 13) χ3 _ (μ + ν )χ2 + (I)

+ (μν - 6)χ + μν - 2 = Ο

όπου μ, ν Ε IR . Να βρεθούν οι τιμές των μ, ν ώστε η εξίσωση να είναι διτετράγωνη και ύστε­ρα να λυθεί.

Για να είναι η εξίσωση διτετράγωνη αρκεί οι συ­ντελεστές τρίτου και πρώτου βαθμού αντίστοιχα να είναι μηδέν. Άρα πρέπει: μ2 + ν2 - 1 3 = 0 και μν - 6 = 0 . Λύνουμε το σύστημα Σ:

μ2 + ν2 - 1 3 = 0} � (μ + ν)2 - 2μν - 1 3 = 0} � μν - 6 = 0 μν = 6 (μ + ν)2 - 2 · 6 - 1 3 = 0} � (μ + ν)2 = 25} � μν - 6 = 0 μν = 6 Ιμ + νl = 5}� μ + ν = 5} (1) ή μ + ν = -5} (2) μν = 6 μν = 6 μν = 6 Επίλυση του ( 1 ) συστήματος: Έστω S = μ + ν και Ρ = μ ν άρα ω2 - Sω + Ρ = 0 (3) � ω2 - (μ + ν)ω + μν = 0 � � ω2 - 5ω + 6 = Ο οπότε ω1 = 2 ή ω2 = 3 αφού λοιπόν μ, ν ρίζες της (3) τότε: μ = 2 και ν = 3 (4) ή μ = 3 και ν = 2 (5) . Αλλά το 4λ2 + λ - 3 είναι τριώνυμο 2ου βαθμού

ΕΥΚΛΕΙΔΗΣ Β ' λθ ' τ.3/21

Page 24: Ευκλειδης Β 59

Μαθηματικά για την Α ' Λυκείου

Επίλυση του (2) συστήματος: Αφού μ, ν ρίζες της ω2 - 5ω + 6 = Ο με όμοιο τρό­πο ω1 = -2 ή ω2 = -3 άρα μ = -2 και ν = -3 (6) ή μ = -3 ή ν = -2 (7). Αντικαθιστώντας στην αρ­χική εξίσωση (I) τις τιμές των μ, ν Ε JR από τις σχέσεις (4), (5), (6), (7) έχουμε: α) χ4 - 5χ2 + 4 = Ο , σύμφωνα με τις σχέσεις (4), (5) β) χ4 + 5χ 2 + 4 = Ο , σύμφωνα με τις σχέσεις (6), (7).

Λύση της εξίσωση χ 4 - 5 χ 2 + 4 = Ο . Θ . 2 ' ετουμε χ = y οποτε:

y2 - 5y + 4 = 0 <=> y = 4 ή y = 1 ' 2 4 . 2 1 αρα χ = η χ =

χ = ±2 ή χ = ±1 . Λύση της εξίσωσης χ4 + 5χ2 + 4 = 0 . Θέτουμε χ 2 = y οπότε: y2 + 5y + 4 = Ο <=> y = -1 ή y = -4 άρα

χ2 = -1 ή χ2 = -4 αδύνατες. Άσκηση 5

Να ορίσετε τον χ Ε R ώστε οι αριθμοί α = χ2 - χ + 1 , β = χ + 2 και γ = χ2 - 4χ + 3 να είναι πλευρές τριγώνου.

ΛΥΣΗ

Για να είναι οι αριθμοί α, β, γ πλευρές τριγώνου θα πρέπει α > Ο , β > Ο , γ > Ο και α < β + γ , β < α + γ , γ < α + β . Επομένως λύνουμε το σύστη-μα των παραπάνω ανισώσεων. Οπότε: α = χ 2 - χ + 1 > Ο ( 1 ) για κάθε χ Ε JR Δ = -3 < Ο και α = 1 > Ο . Πρέπει β > Ο , συνεπώς

χ + 2 > ο <=> χ > -2 (2) Πρέπει γ > Ο , συνεπώς

χ2 - 4χ + 3 > Ο <=> (χ - 1) ( χ - 3) > Ο

διότι

και το τριώνυμο με α = 1 > Ο είναι θετικό εκτός των ριζών, άρα χ < 1 ή χ > 3 (3) . Επίσης α < β + γ άρα χ2 - χ + 1 < χ + 2 + χ2 -4χ + 3 <::> 2χ < 4 <::> χ < 2 (4) Ομοίως β < α + γ άρα χ + 2 < χ 2 - χ + 1 + χ 2 - 4 χ + 3 <=> 2χ 2 - 6χ + 2 > ο

( 3 + ./5 )( 3 - ./5 ) <::> 2 χ - -2

- χ + -2- > 0 ,

το τριώνυμο είναι θετικό εκτός των ριζών συνεπώς: 3 - ./5 . 3 + ./5

(5) Χ <-- η Χ > --2 2

Ομοίως γ < α + β άρα χ2 -4χ + 3 < χ2 -χ + 1 +χ + 2 <::>-4χ <Ο <::> χ >0 (6)

Συναληθεύουμε τις σχέσεις (2), (3), (4), (5), (6)

Άρα

· i I I ι -2 Ο 3-/5 Ι

4

3 - ./5 Ο < χ <-- .

2

Άσκηση 6

I I I

i ) Να κατασκευάσετε εξίσωση 200 βαθμού που οι ρίζες της χ1 , χ2 επαληθεύουν τις σχέσεις:

ΧιΧ2 - 3 (χι + χ2 ) = -5 (1) και χ1χ2 - μ (χ1 + χ2 ) = -1 (2)

ii) Να προσδιοριστεί ο μ Ε JR ώστε η παραπάνω εξίσωση που βρήκατε να έχει δύο ρίζες ίσες.

ΛΥΣΗ

Θέτουμε χ1 • χ2 = ρ και χ1 + χ2 = 5 οπότε οι σχέ-σεις ( 1 ), (2) γίνονται

P - 3S = -5 (2) Ρ - μ · S = -1

Επιλύουμε το σύστημα (Σ) με ορίζουσες ή όπως λέγεται με τη μέθοδο Cramer

D = ι1 -3 ι = 3 - μ 1 -μ

Dρ = ι-5 -3 ι = 5μ - 3 -1 -μ

Ds = ι: =�Ι = 4 .

Βρίσκουμε τις τιμές της παραμέτρου για τις οποίες είναι D = 0 . D = 0 <=> 3 - μ = 0 <=> μ = 3 . Εξετάζουμε περιπτώσεις ανάλογα με τις τιμές της παραμέτρου. 1) Α ν 3 - μ :;t Ο δηλαδή μ :;t 3 τότε είναι D :;t Ο

οπότε το σύστημα έχει μοναδική λύση την D 5μ - 3 . 5μ - 3 Ρ = -Ρ =-- αρα χ 1 · χ 2 =-- (3) D 3 - μ 3 - μ

ΕΥΚΛΕΙΔΗΣ Β ' λθ ' τ.3/22

Page 25: Ευκλειδης Β 59

Μαθηματικά για την Α ' Λυκείου

και S = 0s = -4- άρα Χι + χ2 = -4- (4) .

D 3 - μ 3 - μ

2) Αν 3 - μ = Ο τότε, μ = 3 οπότε D = O και επειδή 05 = 4 :;t: O και DP = I2 ;t: O το σύστημα

είναι αδύνατο. Η εξίσωση που θέλουμε να κατασκευάσουμε έχει τη μορφή y2 - (χ ι + χ2 ) y + χι χ2 = 0 (5)

οπότε από τις σχέσεις (3), (4) προκύπτει ότι η (5) γίνεται

y2 - (-4-) y + 5μ - 3 = 0 <:::>

3 - μ 3 - μ

(3 - μ) y2 - 4y + 5μ - 3 = 0 (6)

Η διακρίνουσα της εξίσωσης (6) είναι:

ΔΥ =42 -4(3-μ)(5μ-3) = Ι6+( 4μ- Ι2) (5μ- 3) =

Ι6 + 20μ 2 - 72μ + 36 = 20μ 2 - 72μ + 52 .

Επειδή η εξίσωση (6) έχει δύο ίσες ρίζες πρέπει ΔΥ = 0 άρα

20μ 2 - 72μ + 52 = ο <:::> 5μ 2 - Ι 8μ + 1 3 = ο (7)

Βρίσκουμε τη διακρίνουσα της (7) Δμ = Ι 82 - 4 · 5 · 1 3 = 324 - 260 = 64 > 0

Άρα η η εξίσωση (7) έχει δύο λύσεις τις

-β + ν'Δ 1 8 + 8 26 1 3 μι = 2α

= ιο = ιο = s ' -β - ν'Δ 1 8 - 8 η μ2 = = -- = 1 .

2α 1 0

Γ ' λ ' 1 3 · Ι ια τις τιμες οιπον μι = 5 η μ2 = η

εξίσωση (6) έχει δύο ρίζες ίσες.

Άσκηση 7 Να λυθεί το σύστημα των ανισώσεων Σ:

( χ2 - χ + ι) ( 2χ2 + χ) 3 > 0

(χ - ι)

( χ3 - sx2 + 6χ) ( χ2 + ι) --'-------'-''----'- < ο

χ2 - 4χ + 3 ΛΥΣΗ

Λύνουμε την κάθε aνίσωση χωριστά (χ2 - χ + 1) (2χ2 + χ )

3 > 0 <:::> (χ - 1)

<:::> (χ - 1 )3 (χ 2 - χ + 1 ) . χ . ( 2χ + 1) > ο ( 1 )

Πρέπει χ :;t: 1 . Το χ2 - χ + 1 είναι θετικό διότι έχει διακρίνουσα Δ = -3 < Ο και συντελεστή δευτεροβάθμιου όρου το α = 1 > Ο άρα χ 2 - χ + 1 > Ο για κάθε χ ε IR . Η ( 1 ) λοιπόν είναι ισοδύναμη με

χ (2χ + Ι) (χ - 1)3

> 0 .

Το (χ -1 )3

είναι θετικό για χ > 1 και αρνητικό για χ < 1 .

Το χ είναι θετικό για χ > Ο και αρνητικό για χ < Ο .

Το 2χ + 1 είναι θετικό όταν 2χ + 1 > Ο <:::> χ > _ _!._ και αρνητικό όταν 2χ + 1 < Ο <:::> χ < _ _!._ .

2 Συνοπτικά από τον πίνακα έχουμε:

χ _ _! ο Ι (χ-1 )3 - - - +

χ - - + +

2x+ l - + + +

Γινόμενο - + - +

Επομένως η aνίσωση ( 1 ) επαληθεύεται για 1 -- < χ < Ο και χ > 1 (2) 2

Η δεύτερη aνίσωση

(χ3 - 5χ2 + 6χ ) (χ2 + 1) --'------:----'--'-----'- < Ο με χ :;t: 1 και χ :;t: 3

χ2 - 4χ + 3

γίνεται:

χ (χ2 - 5χ + 6) (χ2 + 1) ---'-�-----''--'-----'- < ο <:::>

χ2 - 4χ + 3

<==> χ ( χ2 - 5χ + 6) ( χ2 + ι) ( χ2 - 4χ + 3) < Ο .

2

Το χ2 + 1 > Ο για κάθε χ ε IR ως άθροισμα τετρα­γώνων, χ 2 + Ι = χ 2 + 12 . Το τριώνυμο χ 2 - 5 χ + 6 έχει ρίζες τις 2, 3 οπότε χ 2 - 5 χ + 6 > Ο όταν χ < 2 ή χ > 3 και χ2 - 5χ + 6 < 0 όταν 2 < χ < 3 . Το τριώνυμο χ 2 - 4 χ + 3 έχει ρίζες τις 1 , 3 οπότε χ 2 - 4 χ + 3 > Ο όταν χ < 1 ή χ > 3 και χ 2 - 4 χ + 3 < ο όταν ι < χ < 3 . Συνοπτικά προκύπτει ο πίνακας

ΕΥΚΛΕΙΔΗΣ Β ' λθ ' τ.3/23

Page 26: Ευκλειδης Β 59

Μαθηματικά για την Α' Λυκείου

χ -Jo b l 2 3 +I οο Επομένως η ανίσωση επαληθεύεται για χ < Ο και χ - +

Χ2-5χ+6 + +

χ2-4χ+3 + +

χ2+ 1 + +

Γινόμενο - ( +

+ + +

+ - +

- - +

+ + +

- + +

_l ο ι 2 2

1 < χ < 2 (3) Από (2), (3) συναληθεύουμε κι έχουμε

Άρα οι λύσεις του συστήματος είναι

_.!_ < χ < ο ή 1 < χ < 2 . 2

JΊ) 'Ί r JίJ ·;' ! __ /, r / :.. ' J ( ' . . . . . - '; ' rJ-_· J . r_ :'J _, ι .. ιJ ι ; rJ · · · · ' , J " . . -' :.J Γ

.. / j' .

Να λυθεί το σύστημα 3 · (χ + y) - 5xy = 69 } -2(χ + y) + xy = -11

Θέτουμε χ + y = κ και χ · y = λ οπότε το σύστημα 3κ - 5λ = 69 }

γίνεται: -2κ + λ = -1 1

Με τη μέθοδο των οριζουσών έχουμε:

D = 1 3 -5 1 = -7 D = 1 69 -5 1 = 14 -2 1 ' κ - 1 1 1 '

Dλ = 1 3 69 1 = 105 . -2 -1 1

Επειδή D = -7 *- Ο το σύστημα έχει μία λύση :

κ = 0κ = -2 και λ = Dλ = -15 .

D D

Άρα έχω χ + Υ = -2} X · y = -1 5

Τότε τα χ , y είναι ρίζες της εξίσωσης:

ω2 - ( -2) · ω + ( -15) = Ο <=> ω2 + 2ω - 15 = 0

Λύνω την εξίσωση και βρίσκω ρίζες ρ1 = -5 και

ρ2 = 3 . Άρα χ = -5} ή

χ = 3 } y = 3 y = -5

Για ποιες τιμές του λ Ε IR το σύστημα λ3χ - λy = -1} -x + y = λ

Έχει μοναδική λύση;

Νίκος Σ. Ταπεινός Έχει μοναδική λύση το ζεύγος ( χ0 , y 0 ) που επαληθεύει την εξίσωση 4χσ +y0 =6; Είναι αόριστο;

Με τη μέθοδο των οριζουσών έχουμε:

D = � -λ = λ3 - λ = λ(λ - 1) (λ + 1) - 1 1

Dx =1�1 -

1λ l = -1 + λ2 = (λ - 1) (λ + 1)

λ3 1 Dy = _1

� = λ4 - 1 = (λ - 1) (λ + 1) (λ2 + 1) .

Για να έχει μοναδική λύση πρέπει D :F- 0 <=> λ(λ - 1)(λ + 1) :F- 0 <=>

λ :F- Ο και λ *- 1 και λ :F- -1 , δηλαδή πρέπει λ Ε IR - { - 1, Ο, 1} .

Όταν λ :F- Ο και λ *- 1 και λ :F- -1 η μοναδική D 1

λύση του συστήματος είναι: χ0 = _χ = - και D λ

Dy λ2 + 1 Υο = ο = -

λ- .

Τότε έχουμε:

4 λ2 + 1 2 4χ0 + Υ ο = 6 <=> - + -- = 6 <=> λ - 6λ + 5 = 0 . λ λ

Λύσεις της εξίσωσης είναι λ = 5 (δεκτή) ή λ = 1 (απορρίπτεται) . Για να είναι αόριστο πρέπει αρχικά D = Ο <=> λ = -3 ή λ = Ο ή λ = 1 . Για λ = -1 το σύστημα γίνεται:

ΕΥΚΛΕΙΔΗΣ Β ' λθ ' τ.3/24

Page 27: Ευκλειδης Β 59

Μαθηματικά για την Α' Λυκείου

-x + y = -1} -x + y = - 1 που είναι αόριστο.

Για λ = Ο το σύστημα γίνεται: Ο · χ + Ο · y = -1} που είναι αδύνατο. -χ + Υ = 0

Για λ = 1 το σύστημα γίνεται: χ - Υ = - 1} που είναι αόριστο. -χ + Υ = 1

Άρα το σύστημα γίνεται αόριστο όταν λ = 1 ή λ = - 1 .

3. Για ποιες τιμές του λ Ε JR - {1} η aνίσωση 1 (λ - 1)χ2 - 5χ + 3λ + - < 0 , (1) αληθεύει v

για κάθε χ Ε JR •

Λ Υ Σ Η Για να ισχύει η ( 1 ) για κάθε χ Ε JR , πρέπει

και λ - 1 < ο <=> λ < 1 (2)

Δ < ο <=> 25 - 4 · (λ - ι ) ( 3λ +�) < ο <=>

-12λ2 + 1 1λ + 26 < 0 (3) Η ανίσωση αυτή έχει Δ' = 1 369 = 372

και ρίζες λ1 = -� , λ2 = 2 . 1 2

Επειδή - 1 2 <Ο λύσεις της (3) είναι:

λ < -� ή λ > 2 (4) 1 2

Η συναλήθευση των (4) και (3) δίνει λ < -� . 1 2

4 . Για τις διάφορες τιμές τον λ Ε JR * , να εξε­τασθεί αν η εξίσωση ι.χ2 -2ν'3 · χ-λ+4=0 , (1) έχει ρίζες και πόσες.

ΛΥΣΗ

Η διακρίνουσα της ( 1 ) είναι:

Δ = (-2J3)2 - 4 · λ (-λ + 4) =

= 4λ2 - 1 6λ + 1 2 = 4 (λ2 - 4λ + 3) που είναι τριώνυμο 2ου βαθμού με ρίζες λ1 = 3 και λ2 = 1 .

Επειδή 4 > Ο , το πρόσημο της Δ δείχνει ο παρακά­τω πίνακας.

λ -00 1 3 +σο

Δ + - +

Άρα α) Α ν λ < 1 ή λ > 3 τότε Δ > Ο και η ( 1 ) έχει δύο

πραγματικές ρίζες διάφορες μεταξύ τους. β) Αν 1 < λ < 3 τότε Δ < Ο και η ( 1 ) δεν έχει

πραγματικές ρίζες. γ) Α ν λ = 1 ή λ = 3 τότε Δ = Ο και η ( 1 ) έχει μία

διπλή πραγματική ρίζα.

5. Δίνεται η παραβολή y = 2χ2 - ( 3λ + 2) χ + 2 με λ Ε !R . Να προσδιορισθούν οι τιμές της παραμέ­τρου λ ώστε η παραβολή : i) να εφάπτεται στον άξονα χ ' χ ii) να έχει άξονα συμμετρίας τον άξονα y 'y ί ί ί ) να έχει κορυφή με τεταγμένη -2 .

ΛΥΣΗ

ί ) Η παραβολή εφάπτεται στον άξονα χ ' χ αν και μόνον αν

Δ = 0 <::> (3λ + 2)2 - 4 · 2 · 2 = 0 <::> (3λ + 2)2 - 16 = 0 <=> (3λ + 2 - 4) (3λ + 2 + 4) = 0

<=> λ =� ή λ = -2 . 3

ίί ) Γνωρίζουμε ότι η παραβολή y = αχ 2 + β χ + γ με

α * Ο , έχει άξονα συμμετρίας την ευθεία

χ = -1._ . Ο άξονας y 'y έχει εξίσωση χ = Ο . 2α

Άρα για να έχει άξονα συμμετρίας τον y 'y πρέπει

_1._ = ο <=> β = ο <=> 3λ + 2 = ο <=> λ = -� . 2α 3

ί ίί) Γνωρίζουμε ότι κορυφή της παραβολής είναι το

σημείο κ(-:α . r (-!α )) .

Άρα πρέπει f (-:α) = -2 <::>

( 3λ + 2 )2 3λ + 2 2 · -4

- - (3λ + 2) · -4- + 2 = -2 <=>

4

ΕΥΚΛΕΙΔΗΣ Β ' λθ ' τ.3/25

Page 28: Ευκλειδης Β 59

Μαθηματικά για την Α · Λυκείου

(3λ + 2)2 = 32 <::::> l3λ + 2 Ι = m <=> 3λ +Ί = 4J2 ή 3λ + 2 = -4J2

<=> λ = 4J2 - 2 ή λ = - 4../2 + 2 . 3 3

6. Δίνεται η καμπύλη y = J;. + 2 με χ 2:: Ο . Να βρεθεί σημείο της Μ (χ, y) που να απέχει

την ελάχιστη απόσταση από το σημείο Α(3,-2) καθώς και η απόσταση αυτή.

ΛΥΣΗ

Επειδή Μ σημείο της καμπύλης, έχει τεταγμένη y = J;. + 2 δηλαδή Μ ( χ , J;. + 2) . Η απόσταση των σημείων Α και Μ είναι:

(ΑΜ) = �(χ - 3)2 + (J;. + 2 - 2)2 =

= �χ2 - 6χ + 9 + χ = �Χ2 - 5χ + 9 . Για να είναι ελάχιστη η απόσταση ( ΑΜ) , αρκεί

να γίνει ελάχιστη η ποσότητα χ 2 - 5χ + 9 . Θεωρούμε τη συνάρτηση f ( χ ) = χ 2 - 5 χ + 9 με

χ 2:: Ο που επειδή α = 1 > Ο έχει ελάχιστο για

χ = _1_ δηλαδή χ = � . Τότε 2α 2 f (%) = (%Υ - 5 ·% + 9 = 14

1 . Άρα η ελάχιστη α-

. , (ΑΜ) {ϊϊ J1ϊ ποσταση ειναι = '{'4 = l και το σημείο είναι: Μ [% , J% + 2) . 7. Να προσδιορισθεί η παραβολή που διέρχε­

ται από τα σημεία A(l,O) , Β {-1,-10) και Γ(Ο,-3) .

ΛΥΣΗ

Γνωρίζουμε ότι η παραβολή έχει εξίσωση :

y = αχ 2 + β χ + γ με α ::;:. Ο . Επειδή διέρχεται από τα σημεία Α, Β και Γ οι συ­ντεταγμένες τους την επαληθεύουν, δηλαδή : α · 12 + β · 1 + γ = Ο ) α · ( - 1)2 + β · ( -1) + γ = -10 δηλαδή

α · 02 + β · Ο + γ = -3

α + β + γ = Ο } α - β + γ = -10 <=> γ = -3

: � : = � = � 1 0} <=> : � : : �7} <=> ; :;2} . γ = -3 γ = -3 γ = -3

Άρα y = -2χ2 + 5χ - 3 η ζητούμενη παραβολή.

8. Να βρεθεί το πλήθος των κοινών σημείων του κύκλου με εξίσωση χ2 + y2 = 1 και της ευθείας y = χ - λ με λ ε 1R. •

ΛΥΣΗ

Τα κοινά σημεία βρίσκονται από τη λύση του συ­χ2 + y2 = 1} . στήματος y = χ - λ

Θέτουμε την πρωτοβάθμια στη δευτεροβάθμια και έχουμε:

χ 2 + (χ - λ )2 = 1 <=> 2χ 2 - 2λχ + λ2 - 1 = ο Δ = 4λ2 - s ( λ2 - 1) = -4λ2 + 8 .

α) Αν Δ > Ο <=> -4λ2 + 8 > Ο <=> λ2 < 2 <=> lλ l < J2 <=> -J2 < λ < J2 τότε

η ευθεία και ο κύκλος έχουν δύο κοινά σημεία (Η ευθεία τέμνει τον κύκλο) .

β) Αν Δ = Ο <=> -4λ2 + 8 = Ο <=> λ = J2 ή λ = -J2 τότε η ευθεία και ο κύκλος έχουν ένα κοινό σημείο (Η ευθεία εφάπτεται του κύκλου) .

γ) Α ν Δ < Ο <=> -4λ2 + 8 < Ο <=> λ2 > 2 <=> lλ l > J2 <=> λ < -J2 ή λ > J2

τότε η ευθεία και ο κύκλος δεν έχουν κοινά σημεία (Η ευθεία εκτός του κύκλου)

9. Να λυθεί το σύστημα 2χ2 + 3y2 = 14 } ( 3χ - y )2 = 13 - 6xy

ΛΥΣΗ

( 1) <=> 2χ2 + 3y2 = 14} 9χ2 + y2 = 1 3

Θέτουμε χ 2 = κ και y2 = λ (2)

(1)

(Πρέπει κ 2:: Ο και λ 2:: Ο ) και έχουμε:

ΕΥΚΛΕΙΔΗΣ Β ' λθ ' τ.3/26

Page 29: Ευκλειδης Β 59

Μαθηματικά για την Α ' Λυκείου

2κ + 3λ = 14} 9κ + λ = 1 3 Με τη μέθοδο των οριζουσών έχουμε:

D = l� �I = 2 - 27 = -25

D = 114 3 1 = 14 - 39 = -25 κ 1 3 1

12 14 1 Dλ = = 26 - 126 = -100 . 9 1 3

D D Επειδή D = -25 -::;:. Ο έχω κ = _κ = 1 , λ = ___],_ = 4 . D D Από (2) � �: : :}� :�: : �} Άρα χ = 1 }

ή χ = - 1} ή

χ = 1 } ή

χ = - 1} . y = 2 y = 2 y = -2 y = -2

fEΩMETPIA Ι<ιίιιλος ιιαι αναλογίcς

λίγο πριν: • Φιλοδοξία του μικρού αυτού άρθρου είναι να δώσει μία

σειρά από ασκήσεις στο χώρο του κύκλου και των ανα­

λογιών με διάθεση να υπάρξει μία πιο ενεργητική συμμε­

τοχή του λύτη αναγνώστη.

• Στοχεύει επίσης στη δημιουργία της ανάπτυξης μιας

μεθόδευσης και μιας συλλογιστικής που θα βοηθούν

στην καλύτερη αντιμετώπιση αυτών των ασκήσεων. Θα

θέλει επίσης να γίνει αφορμή για την εμβάθυνση σε πιο

ουσιαστικά σημεία του χώρου της Ευκλείδειας

γεωμετρίας.

• Νομίζουμε ότι τόσο ο κύκλος, όσο και οι αναλογίες

προσφέρονται για μια τέτοιου είδους ουσιαστική επαφή.

Προϋποθέτει βέβαια να βλέπουμε όσο μπορούμε πιο κα­

θαρά τις σχέσεις που υπάρχουν στον κύκλο και να κάνου­

με γρήγορες μεταφορές γωνιών στις συνθήκες που ισχύ­

ουν, αλλά και με τις αναλογίες να καταλαβαίνουμε την

έννοια της παραλληλίας και της ομοιότητας.

• Σίγουρα κάτι τέτοιο δημιουργεί δυσκολίες . Α ν όμως μας

αρέσει η περιπέτεια της άλλης ματιάς και η δυνατότητα

της προσωπικής μας παρέμβασης σ' αυτό που δημιουρ­

γείται, θα ήταν πολύ καλό να προσπαθούσαμε . . . και να

καταγράφαμε την απόπειρα αυτής της διαδρομής . . .

Ζώτος Βαγγέλης

Να βρεις μιαν άλλη θάλασσα μιαν άλλη απαλοσύνη . . . "Αμοργός" Ν. Γκάτσος

Α 1 • Δίνεται τετράγωνο ΑΒΓ Δ και επί της δια­γωνίου ΒΔ θεωρούμε τυχαίο σημείο Ε και με πλευρά την ΕΓ κατασκευάζουμε τετρά­γωνο ΕΓΖΗ. i) Να αποδειχτεί ότι η διαγώνιος ΕΖ του

νέου τετραγώνου και οι πλευρές ΒΕ και ΕΖ σχηματίζουν ορθογώνιο τρίγωνο

ii) Τα τρίγωνα ΕΓ Δ και ΒΖΓ είναι ίσα Απόδειξη

Α ν φέρουμε την ΒΖ θα έχουμε το ΖΒΕΓ εγ­γράψιμο αφού ω = φ = 45° (από τα τετράγωνα) και

βλέπουν την ΕΓ. Αλλά Erz = 90° , οπότε και Ε:ΒΖ = 90° .

Α Β Η - · - - ' '

' · '

'· , \

ρ - - - -xr ';\ z

θ ν- - - -

Δ�-----'Υ:Γ Τα τρίγωνα ΕΓΔ και ΒΖΓ είναι ίσα αφού ΓΔ = ΒΓ, ΓΖ = ΓΕ και θ = p (χ+ θ = χ + ρ = 90° ) Α2• Θεωρούμε μία χορδή ΑΒ ενός κύκλου και το

ΕΥΚΛΕΙΔΗΣ Β' λθ ' τ.3/27

Page 30: Ευκλειδης Β 59

Μαθηματικά για την Α ' Λυκείου

μέσο Μ του κυρτού τόξου ΑΒ. Α ν Α και Β εί­

ναι δύο οποιαδήποτε σημεία του μη κυρτού �

τόξου ΑΒ και οι χορδές ΜΓ και ΜΔ τέμνουν

την ΑΒ στα σημεία Κ και Λ, να δείξετε ότι το

τετράπλευρο ΓΚΛΔ είναι εγγράψιμο.

Απόδειξη

Θέλουμε ΓΚΛΔ εγγράψιμο, αρκεί ω = φ . Φέρ-----Α Α ΜΒ ΑΓ νουμε την ΑΜ τότε: ω = Αι + Μ ι = -- + - ( 1 ) 2 2 --

Δ = Γ ΑΜ

= ΜΑ

+ ΜΓ = ΜΒ

+ ΑΓ (2) 2 2 2 2 2

Τότε ω = Δ άρα ΓΚΛΔ εγγράψιμο (από (1) , (2)). Α3• Ευθεία (ε) είναι παράλληλη στην πλευρά

ΒΓ τριγώνου ΑΒΓ και τέμνει τις πλευρές

ΑΒ και ΑΓ στα σημεία Δ και Ε. Από το Γ

φέρνουμε παράλληλη στη ΒΕ που τέμνει την

Β Z Ν δ , , ΑΔ ΑΒ

Α στο . α απο ειχτει οτι - = ­ΑΒ ΑΖ

Απόδειξη

Είναι χρήσιμο λίγο πριν . . . δούμε αναλογίες . . . να σκεφτό-

μαστε από που προέρχονται . . . από Θ. Θαλή ή ομοιότητα . . . Αν

προέρχονται από Θ. Θαλή σημαίνει ότι έχουμε παραλληλίες άρα

βοηθάει πολύ να τις ιχνογραφούμε έντονα ή με διαφορετικό χρώ­

μα και να παίρνουμε τις σχετικές αναλογίες.

Στη συγκεκριμένη περίπτωση έχουμε παραλλη­λία άρα:

ΔΕ // ΒΓ τότε ΑΔ = ΑΕ ( 1 ) ΑΒ ΑΓ ΒΕ//ΓΖ τότε ΑΒ

=ΑΕ (2) ΑΖ ΑΓ

Α

z

Από ( I ), (2) έχουμε ΑΔ =

ΑΒ ΑΒ ΑΖ

Α4• Δίνεται τρίγωνο ΑΒΓ και ΑΜ διάμεσος. Παίρ-" Α Α Α

νουμε ΒΑΧ = ΑΜΓ και ΑΒΥ = Γ . Αν Κ το

κοινό σημείο των ΑΧ, ΒΥ τότε ισχύουν: Δ Δ

i) τα τρίγωνα Α Β Κ , Α Μ Γ είναι όμοια

ii) ΒΚ = 2βγ

α 2 2 . . . ) ΜΚ

μα 111 = --α

ίν) Η ΑΜ διχοτομεί την ΒΜΚ Λ Λ Λ ν) ΓΒΚ = Β - Γ (σχήμα Laisant)

Απόδειξη Δ Δ

i) Α ΒΚ , ΑΜ Γ (όμοια, αφού ΒΑχ = ΛΜr. ΑΒΥ = Γ ) Α

χ

.. ) Α , , , ΒΚ ΑΒ ΑΚ ( l ) 11 πο την ομοιοτητα εχουμε: ΑΓ = ΜΓ = ΑΜ

, ΒΚ ΑΒ · ΑΓ , ΒΚ 2βγ οποτε: = οποτε = - . ΜΓ α Δ Δ

iii) Α ΒΜ όμοιο με ΑΜ Κ από την ( 1 ) έχουμε ΑΒ ΑΚ , ΒΑ ΑΚ - - - η - - -ΜΓ ΑΜ ΒΜ ΑΜ και αφού ΑΜΓ = ΒΑκ τότε ΜΑκ = ΜΒΑ οπότε από την ομοιότητα των τριγώνων έχουμε:

ΜΚ ΑΜ 2 2 -- = -- τότε ΜΚ = � ΑΜ ΒΜ α

Δ ίν)Από την ομοιότητα των τριγώνων Α ΒΜ και Α Μ Κ έχουμε ακόμη ότι: ΑΜΒ = ΑΜΚ , δη­λαδή η ΑΜ διχοτομεί την ΒΜΚ .

ν) Αφού ΑΒΚ = Γ θα είναι και ΓΒΚ = Β - Γ Α5.Από το μέσο Μ της πλευράς ΒΓ τριγώνου

ΑΒΓ φέρνουμε τυχαία ευθεία (ε) που τέμνει

την ΑΓ στο Ν, την ΑΒ στο Ρ και την από το

Α παράλληλη προς τη ΒΓ στο Κ. Να δειχτεί

, ΡΚ ΝΚ

οτι: -- = -- . ΡΜ ΝΜ

Απόδειξη

�ΥΚΛΕΙΔΗΣ Β ' λθ ' τ.3/28

Page 31: Ευκλειδης Β 59

Μαθηματικά για την Α ' Λυκείου

Τα τρίγωνα ΡΑΚ και ΡΒΜ είναι όμοια λ λ λ ΡΚ ΡΑ ΑΚ ( Ρ = κοινή, Κ = Μ ) άρα PM = PB = BM ( l ) .

Τα τρίγωνα ΝΑΚ και ΝΜΓ είναι επίσης όμοια ΝΚ ΑΚ ΑΝ ΑΝ - = - = - = - (2) ΝΜ ΜΓ ΜΓ ΒΜ

οπότε από ( 1 ) , (2) ΡΚ = ΝΚ . ΡΜ ΝΜ

Α6.0ι κύκλοι (01 , ρ1 ) , (02 ,ρ2 ) , (03 ,ρ3 ) είναι ανά δύο εκτός και Κ, Μ, Λ τα σημεία τομής των εξωτερικών εφαπτομένων ανά δύο. Να

δ , , κοι ΛΟ3 ΜΟ3 1 ειχτει οτι: -- · -- · -- = ΚΟ2 ΛΟ1 ΜΟ3

(θεώρημα D' Alembert) Απόδειξη

'Εχουμε: ΚΟι = Ε.!_ , ΛΟ3 = ..!2. , MOz = Εl. οπότε ΚΟ2 ρι ΛΟι ρι ΜΟ3 ρ3

, Μ

Κ01 • Λ03 . Μ02 = Ε.ι_ . .!2_ . El_ = 1 Κ02 Λ01 Μ03 ρ2 ρ 1 ρ3

κ

Α7.Δίνεται κύκλος (Ο, ρ ) ευθεία (δ) που διέρ­χεται από το κέντρο Ο του κύκλου και δύο σημεία Α και Β της (δ) συμμετρικά μεταξύ τους ως προς το Ο. θεωρούμε τυχαίο ση­μείο Ρ του κύκλου και φέρνουμε τις Ρ Α, ΡΒ που τέμνουν τον κύκλο στα Μ και Ν. Να

δ , , ΑΡ ΒΡ , θ , απο ειχτει οτι -- +-ει ναι στα ερο. ΑΜ ΒΝ

Απόδειξη Θέτουμε ΟΑ = ΟΒ = μ . 'Ε ΡΑ ΑΕ ( , χουμε - = -- απο ΑΖ ΑΜ ομοιότητα τριγώνων

ΑΡ · ΑΜ = ΑΕ · ΑΖ ΑΕΜ και ΑΡΖ) άρα: (ρ - μ) (ρ + μ) = ρ2 - μ2

(δ)

Μ

Αλλά ΑΡ · ΑΜ = ρ2 - μz ή ΑΡ = ΑΡ2 ΑΡ2 ΑΡ2 ΑΜ ρ2 - μ2

ομοίως ΒΡ · ΒΝ = ΒΖ · ΒΕ (από ομοιότητα τριγώ-• Δ ΒΡ ΒΡ2 νων ΒΖ Ν και Β Ρ Ε ) οπότε: - = 2 , • ΒΝ ρ - μ-

ΑΡ ΒΡ ΑΡ2 + ΒΡ2 Άρα - + - = ( 1 ) αλλά ΑΜ ΒΝ ρ2 - μ2 ΑΡ2 + ΒΡ2 = 2ρ2 + 2μ 2 από θεώρημα διαμέσων

ΑΡ ΒΡ 2 (ρ2 + μ2 ) οπότε -- +- = 2 2 = σταθερό ΑΜ ΒΝ ρ - μ

As. Δύο σημεία Α και Β βρίσκονται προς το ίδιο μέρος μιας ευθείας (ε) . Ονομάζουμε Α' το συμμετρικό του Α ως προς την (ε) και Μ το σημείο τομής των ευθειών Α 'Β και (ε). Η

μεσοκάθετη του τμήματος ΑΒ, τέμνει την (ε) στο Ν. Να αποδειχτεί ότι τα σημεία Α, Β, Ν και Μ είναι ομοκυκλικά.

Απόδειξη Το Ν είναι το περίκεντρο του τριγώνόυ ΑΒΑ ' , (η (ε) μεσοκάθετη του ΑΑ 'και η (ε 1 ) μεσοκάθετη του

-λ ΑΝΑ' ΑΒ). Άρα έχουμε: Β 1 = -- και & = &1 , άρα 2

Βλ - cOI + cO ­� - -- - ω 2

Α '

( ι; , )

ΕΥΚΛΕΙΔΗΣ Β ' λθ ' τ.3/29

Page 32: Ευκλειδης Β 59

Μαθηματικά για την Α· Λυκείου

Άρα ΑΒΝΜ εγγράψιμο σε κύκλο οπότε Α, Β, Ν, Μ ομοιαικλικά. Α9• Δίνεται τρίγωνο ΑΒΓ και τα μεταβλητά ση­

μεία Α', Β ', Γ των πλευρών ΒΓ, ΓΑ, ΑΒ. Αν Α'Β Β'Γ Γ'Α -- = -- = -- να βρεθούν οι γ.τ των μέ­Α'Γ ΒΆ Γ'Β σων των πλευρών του τριγώνου Α 'Β 'Γ.

Λί)ση : Θα προσπαθήσουμε να βρούμε το γ.τ του μέσου Μ

Β 'Γ' , Β'Γ ΓΆ , , , της , εχουμε: Β' Α

= Γ'Β

απο την αρχικη σχε-

ση . Οπότε προκύπτει Α

Β I\ Μ, Γ Β'Γ

_ ΓΆ , Β 'Γ _ � ΑΓ - ΑΒ

η Γ'Α

( 1 ) .

Α ν Μ2 και Ρ τα μέσα των ΑΓ, ΑΒ ' τότε ΑΓ' ΓΒ' , ΡΜ ΑΓ' γ

ΡΜ//-- και ΡΜ2 = - τοτε --

= -- = - και 2 2 ΡΜ2 ΓΒ' β

ΜΡΜ2 = ΑΒΓ οπότε τρίγωνα ΜΡΜ2 , ΑΒΓ ό­

μοια οπότε ΡΜ2Μ = AfB οπότε το Μ βρίσκεται στην (ε) όπου (ε) η ευθεία που περνάει από το Μι και είναι παράλληλη προς τη ΒΓ. Επειδή το Β ' μεταβάλλεται μεταξύ των Α, Γ σημαίνει ότι ο γ.τ του είναι το τμήμα Μ2 Μ3 , ομοίως Μ2 , Μ3 τα μέσα των ΑΓ, ΑΒ. Ομοίως οι άλλοι δύο γ. τόποι είναι τα τμήματα Μι Μ2 και Μι Μ3 •

Προτεινόμενες ασκήσεις Α ι · Δίνεται γωνία χδψ , η διχοτόμος της ΟΔ και

Μ ένα σημείο εσωτερικό της ΔΟΨ . Αν Α, Β, Γ είναι οι προβολές του Μ στις ημιευθείες ΟΔ, ΟΧ, ΟΨ να δειχτεί ότι: α) τα σημεία Ο, Β, Α, Μ, Γ είναι ομοιαικλικά β ) τα τμήματα ΑΒ και ΑΓ είναι ίσα

Α2. Δίνεται χορδή ΓΔ ενός κύκλου (Ο,ρ) και η διά­μετρος του ΑΒ. Θεωρούμε τις προβολές των άκρων της χορδής επί της διαμέτρου όπως επίσης και τις προβολές των άκρων της διαμέ­τρου επί της χορδής. Να αποδειχτεί ότι οι τέσ­σερεις προβολές έχουν την ίδια απόσταση από το μέσο Μ της χορδής ΑΒ.

Α3. Οι κορυφές εγγεγραμμένου σ� κύκλο ορθογω­νίου τριγώνου διαιρούν την περιφέρεια σε τρία τόξα. Στο κάθε ένα από αυτά φέρνουμε μία εφαπτομένη έτσι ώστε το σημείο επαφής της να είναι μέσο του ευθ. τμήματος που ορίζεται πάνω σ' αυτήν από τις προεκτάσεις των καθέ­των πλευρών του ορθογωνίου. Να αποδειχτεί ότι τα σημεία επαφής των εφαπτομένων αυτών είναι κορυφές ισοπλεύρου τριγώνου. (Θεώρημα Pollok)

Α4. Θεωρούμε δύο παράλληλες ευθείες ει και ε2 και μία κοινή κάθετη αυτών ΑΒ. Αν Ρ το μέσο του Α.Β. και Α, Β μεταβλητά σημεία

των ει , ε2 έτσι ώστε ΑΡΒ = 90· τότε να βρεθεί ο γ. τόπος της προβολής Μ του Ρ στην ΑΒ.

Αs.Σε τρίγωνο ΑΒΓ φέρνουμε τα ύψη ΑΑ', ΒΒ ' , ΓΓ ' καιΑ'Ζ .l ΓΓ', ΑΈ .l ΑΓ . Να αποδειχτεί ότι ΖΕ/ /Β 'Γ ' .

Α(,• Σε παραλληλόγραμμο ΑΒΓ Δ φέρνουμε τυχαία ημιευθεία Αχ που τέμνει τις ΒΓ και Γ Δ στα Ε, Ζ αντίστοιχα. Να αποδειχτεί ότι το γινόμενο ΒΕ · ΔΖ είναι σταθερό.

Α7. Θεωρούμε κύκλο (Ο,ρ) τη χορδή ΓΔ κάθετη στη διάμετρο ΑΒ και σημείο Μ της Γ Δ. Οι ευ­θείες ΑΜ και ΒΜ τέμνουν τον κύκλο στα Ε

Z Ν δ , , ΖΓ ΕΓ και . α απο ειχτει οτι - = - .

ΖΔ ΕΔ

Αι;. Δίνεται ισοσκελές τρίγωνο ΑΒΓ το ύψος Γ Δ και η ημιευθεία Γ χ .l ΑΓ που τέμνει την ΑΒ

Ε Ν δ , , ΕΒ ΓΕ στο . α ειχτει οτι: - = - .

ΒΔ ΓΔ

Βιβλιογραφία Ι . "Μεθοδολογία των γ. τόπων" Γ. Ντάνης

2. "Προβλήματα Γεωμετρίας" Ν. Κισκ6ρας

3. "Μέθοδοι γ. προβλημάτων" Αρ. Δημητρίου

4. "Μεγάλη Γεωμετρία" Α. Πάλλας

5. "Γεωμετρία. 3, 6". Ι . Ιωαννίδης

6. 'Άides pour le cycle" . ΑΡΜΕΡ. Νο. 49 7. "Gours de Geometήe" Ligel. 266Μ

8. "Θεωρία Γεωμετρικής Άσκησης" Δ. Ι. Λιβέρη

9. "Ευκλείδειος Γεωμετρία" Χ. Παπανικολάου

Ι Ο. "Notes sur la geometήe du tήangle" Ε Lemoine. 1 1 . "Θ. Γεωμετρία" Δ. Παπαμιχαήλ-Α. Σκιαδά

1 2. "Inequalities'' . G. Polya

ΕΥΚΛΕΙΔΗΣ Β ' λθ ' τ.3/30

Page 33: Ευκλειδης Β 59

.......... ,.. �μ r ��'' �-" Α,_,_,

ΑΛfΕΒΡΑ . . · ' - · · . .. .

Από τον Σέσα στον Gauss Παναγιώτης Χριστόπουλος - Νίκος Κόντζιας

Όταν στην Ινδία βασιλιάς ήταν ο Ιαντάβα, εμφανίστηκε στο παλάτι ο Λαχούρ Σέσα και του χάρισε το παι­

χνίδι που είχε ανακαλύψει, το σημερινό σκάκι.

Ο βασιλιάς ενθουσιάστηκε και του πρότεινε να τον γεμίσει χρυσάφι. Ο Σέσα όμως ζήτησε το δώρο του να

είναι σπόροι σταριού. « Το σκάκι έχει 64 τετράγωνα, βάλτε στο πρώτο ένα σπόρο, στο δεύτερο δύο, στο τρί­

το 4, στο τέταρτο 8 και θα διπλασιάζετε τους σπόρους μέχρι το 64° ». Όλοι γέλασαν, αλλά αργότερα οι μα­

θηματικοί της Αυλής διαπίστωσαν ότι η ποσότητα του σταριού είναι μεγάλη και ξεπερνά την παραγωγή όλης

264 της Γης. ( S64 = ------=-! = 1 8 τρισεκατομμύρια τόνοι. Γεωμετρική πρόοδος). 2 - 1 Αργότερα το 1 785 ο οκτάχρονος Gauss πρόσθεσε τους αριθμούς από το 1 μέχρι το 40 πολύ γρήγορα και παραξέ­

νεψε το δάσκαλό του ο οποίος του ζήτησε στη συνέχεια να προσθέσει από 1 μέχρι το 100. Όμως και πάλι απάντησε

στον ίδιο χρόνο. Τότε ο δάσκαλος ζήτησε να του δείξει το μυστικό που δεν ήταν άλλο από το άθροισμα ν - όρων

ν (ν + 1) της αριθμητικής προόδου. ( Σ = , 40 + 1 = 39 + 2 = 38 + 3 = . . . Δηλ. 20 ζευγάρια με άθροισμα 41). 2

Άσκηση lη Δίνεται μια ακολουθία � =4ν-8 για κάθε ν Ε Ν* . Να εξετάσετε αν η ακολουθία είναι αριθμητική ή γεωμετρική πρόοδος. ΛΥΣΗ α1 = 4 · 1 - 8 = -4 για κάθε ν Ε Ν* ισχύει αν+Ι = 4(ν + 1) - 8 = 4ν + 4 - 8 = 4ν - 8 + 4 = αν + 4 Δηλαδή αν+ Ι - αν = 4 άρα είναι αριθμητική πρόο­δος με α1 = -4 και ω = 4 . Προς λύση Να βρείτε τον όρο της α10 •

Ποιος όρος είναι ίσος με 40.

Άσκηση 2η Δίνεται μια ακολουθία αν = 2

ν για κάθε ν Ε Ν* .

Να εξετάσετε αν η ακολουθία είναι αριθμητική ή γεωμετρική πρόοδος. ΛΥΣΗ α1 = 2 , Για κάθε ν Ε Ν* ισχύει

αν+Ι = 2ν+Ι = 2ν · 2 = αν · 2 .

Δηλαδή αν+Ι = 2 άρα είναι γεωμετρική πρόοδος

με α1 = 2 , λ = 2 .

Άσκηση 3η Σε μια αριθμητική πρόοδο ισχύει α,. +� =α,.+ν •

( μ, ν Ε Ν ) Ν α βρεθεί η πρόοδος.

ΛΥΣΗ Έστω α1 ο πρώτος όρος και ω η διαφορά, τότε

ΕΥΚΛΕΙΔΗΣ Β ' λθ ' τ.3/31

Page 34: Ευκλειδης Β 59

Μαθηματικά Β ' Λυκείου

α1 + (μ - Ι)ω + α1 + (ν - l)ω = α1 + (μ + ν - l)ω τελικά α1 = ω . Άρα η πρόοδος είναι α1 , 2α1 , 3α1 , • • • •

Άσκη ση 41J

Σε μια αριθμητική πρόοδο έχουμε «ι = 3 ,

α5 = 11 , α6 = 13 και Σ ν = 120 .

Να βρείτε το πλήθος των όρων της προόδου. Λ Υ Σ Η ω = α6 - α5 = 1 3 - 1 1 = 2

, 2α1 + (ν - 1) ω Ισχυει Σν = · ν 2 2 · 3 + (ν - 1) · 2 Άρα 1 20 = · ν 2

240 = 6ν + 2ν2 - 2ν � 2ν2 + 4ν - 240 = 0 � ν2 + 2ν - 1 20 = Ο , ν = 10 . Άσκη ση 511 Α ν «ι , α2 , α3 διαδοχικοί όροι γεωμετρικής

προόδου με λόγο λ και ισχύει α3 > 4α2 - 3αι •

Ποιες είναι οι δυνατές τιμές του λ. \ΥΣΗ

α2 = λα1 , α3 = λ2 α1 Άρα λ2α1 > 4λα1 - 3α1 ή

α1 { λ2 - 4λ - 3) > Ο ο) Α ν α1 > Ο τότε λ2 - 4λ - 3 > Ο � λ < 1 ή λ > 3 . β ) Α ν α1 < Ο τότε λ2 - 4λ - 3 < Ο � 1 < λ < 3 . Άσκη ση 6'1

Να υπολογισθεί το άθροισμα των ν όρων Σ = 5 + 55 + 555 + 5555 + •.. 55 .•• 5

..__...... ν πεντάρια , \ Υ Σ Η Το άθροισμα Σ = 5 · ( 1 + 1 1 + 1 11 + . . . ) ( 1 ) πο/Jζω την ( 1 ) επί 9 και 9Σ = 5 · (9 + 99 + 999 + . . . ) 9Σ = 5 · ( 10 - 1 + 102 - 1 + 103 - 1 + . . . ) = = 5 {10 + 102 + . . . + 10ν-Ι _ ν ] =

[ 10ν - 1 ] [ 10ν - 1 ] = 5 · 10 _ 1 · 1 0 - ν = 5 · -9- · 10 - ν =

� sΓο - ( ιο'9- ι) - 9ν ] � s { ιο>+1 -;ο - 9ν ] �

5 · 1 0ν+Ι - 50 - 45ν = 9 50{10ν - ι) - 45ν Σ = _ _,_____.....:...._ __ 8 1

Άσκη ση 7'�

50( 10ν - Ι ) - 45ν 9

Τρεις μη μηδενικοί αριθμοί αποτελούν διαδοχικούς

όρους Αριθμητικής προόδου και τα τετράγωνά

τους διαδοχικούς όρους Γεωμετρικής προόδου.

Να βρείτε τις δυνατές τιμές του λόγου λ της

Γεωμετρικής προόδου.

ΛΥΣΗ

Έστω α, β , γ αριθμ. πρόοδος α2 , β2 , γ2 γεωμ. Πρόοδος τότε β = α + γ και β4 = α2γ2 •

2 ( )2 Προκύπτει β2 = Ιαγl � α + γ = lαγΙ . 4

1) Αν αγ > Ο τότε (α - γ)2 = 0 � α = γ και επει-δή γ2 = λ2α2 � λ = �� � = � = 1 .

2) Αν αγ < Ο τότε α2 + γ2 + 6αγ = Ο και 1 + (�Υ + 6(�) = 0 ( 1 ) . Επειδή � < 0 θέτουμε

��� = -� = -λ και ( Ι ) γίνεται λ2 + 6λ + Ι = Ο � λ1 = -3 + 2.J2 ή λ2 = -3 - 2.fi .

Άσκηση 8'�

Το γινόμενο τριών διαδοχικών όρων Γεωμετρι­

κής προόδου είναι 216 και το άθροισμά τους 26. Ποια είναι η πρόοδος;

ΛΥΣΗ

'Εστω � , α1 , λα1 οι τρεις όροι της προόδου είναι λ � · α1 · λα1 = 2 16 ή αi = 2 Ι 6 , αi = 63 , α1 = 6 . λ

'Ετσι έχουμε i + 6 + λ · 6 = 26 λ i + λ · 6 = 20 6 + λ2 · 6 = 20 λ 1 , λ 6 + 6λ2 = 20λ , 6λ2 - 20λ + 6 = ο

ΕΥΚΛΕΙΔΗΣ Β ' λθ ' τ.3/32

Page 35: Ευκλειδης Β 59

Μαθηματικά Β ' Λυκείου

'Αρα: Η πρόοδος είναι 6 - 6 6 · 3 η' 3 ' '

Άρα 2, 6, 1 8 ή

� 6 .!. . 6 1 ' ' 3 3 1 8 , 6, 2.

Για το ίδιο πρόβλημα σε αριθμητική πρόοδο όταν γνωρίζουμε το άθροισμα τους συμβολίζουμε με: α - ω , α, α + ω . Άσκη ση 911 Θεωρούμε τους 2ν πρώτους όρους μιας γεωμε­τρικής προόδου α1 ,α2 , ... ,α2ν . Το άθροισμά

τους είναι ίσο με το πενταπλάσιο του αθροίσμα­

τος των όρων άρτιας τάξης.

Δ η λαδή ισχύει s2ν = ss2n όπου s2ν = α ι + α2 + ... + α2ν

Να βρεθεί ο λόγος ω της προόδου.

Λ Υl: Η Έχουμε

α Ι ( 1 + ω + . . . + ω2ν-Ι ) = 5αΙ (ω + ω3 + . . . + ω2ν-Ι ) 2ν-1 3 2v-l � � ) <=> 1 + ω + . . . + ω = 5 (ω + ω + . . . + ω

1 + ω + . . . + ω = 5ω 1 + ω + ω . . . + ω 2v-l ( 2 ( 2 )2 ( 2 )ν-Ι ) 1 - ω2ν 1 - ω2ν 1-ω;tQ --- = 5ω · <=> 1 - ω 1 - ω2

� = 5ω · ���--

1 1 + ω = 5ω <=> 4ω = 1 <=> ω = - . 4 Ά σκηση 1 011 Δείξετε πότε μια ακολουθία αριθμών β1 , β2 , ... βν

είναι συγχρόνως αριθμητική και γεωμετρική πρό­

οδος.

Λ Υ Σ Η Εάν β1 , β2 , . . . βν είναι όροι αριθμητικής προόδου θα έχουμε β _ βν + βν+2 ν+Ι - 2 ν 2 1 ( Ι )

Εάν β1 , β2 , • • . βν είναι όροι γ.π. θα έχουμε βν = βιων-Ι και βν+Ι = βιων βν+2 = βιων+Ι αντί­στοιχα. Εάν αντικαταστήσουμε τις τιμές των βν , βν+Ι , βv+2 στην ( 1 ) μετά τις πράξεις έχουμε

( Ι ) 2 ω2 - 2ω + 1 = 0<=>(ω - 1) = 0 άρα ω = 1 . Η ακολουθία είναι συγχρόνως αριθμητική και γ.π. όταν είναι της μορφής β, β, β, . . . . Άσ κη ση 1 1 11

Η διαφορά μεταξύ δύο θετικών αριθμών είναι

72 και ο αριθμητικός μέσος των δύο αριθμών

υπερέχει από το γεωμετρικό μέσο κατά 8. Να

βρεθούν οι αριθμοί.

ΛΥΣΗ

Εάν χ , y είναι οι δύο αριθμοί θα έχουμε (Q x - y = 72 } x - y = 72 }

χ + (Α) <=> (Β) (2) -f = FY + 8 x + y = 2FY + ι6

Από την επίλυση του συστήματος έχουμε y = 49 , χ = 1 2 1 .

Ά σκη ση 1 211

Έστω Σν το άθροισμα ν όρων της προόδου

α, α + β, α + 2β, ... και Σ� το άθροισμα ν όρων

της προόδου α,α + γ,α + 2γ, ... .

Για τις δύο αριθμητικές προόδους ισχύει ότι ο

όρος κ τάξεως της πρώτης αριθμητικής προό­

δου είναι ίσος με το όρο κ2 τάξεως της δεύτε­

ρης αριθμητικής προόδου. Να αποδείξετε ότι η

διαφορά μεταξύ του αθροίσματος των ν πρώτων

όρων της δεύτερης προόδου από το άθροισμα

των ν όρων της πρώτης δίνεται από τον τύπο

Λ ΥΣ Η

Σ - Σ' = ν (ν - l)γκ ν ν

2

Ο όρος κ τάξεως της πρώτης αριθμητικής προόδου είναι ακ = α + (κ - l)β και ακ' =α+ (κ2 - 1)γ . Από την εκφώνηση έχουμε α + (κ - 1 )β = α + (κ2 - ι)γ <=> β = (κ + l)γ, κ :;t: l

ΕΥΚΛΕΙΔΗΣ Β ' λθ ' τ.3/33

Page 36: Ευκλειδης Β 59

Μαθηματικά Β ' Λυκείου

(ν - l) (β -γ) (ν - l)[(κ+ 1)γ-γ] ν(ν- 1)κy ����ν = ν 2 2 2

Άσκηση 13η Το άθροισμα 10 πρώτων όρων αριθμητικής

προόδου είναι 245 και το πηλίκο του τρίτου ό-

, ' ' 29

Ν β θ ' ρου προς τον εκτο ο ρο ειναι : "2 . α ρε ει η

πρόοδος.

ΛΥΣΗ

Σ10 = 245

Εάν α ο πρώτος όρος και ω η διαφορά σύμφωνα με την εκφώνηση έχουμε

· α + 2ω = .i <:::::> ω = 5α ( 1 ) α + 5ω 9 2 Σ1 0 = [ 2α � 9ω} 10 <:::::> 245 = (2α + 9ω) · 5 <:::::>

<:::::> 2α + 9ω = 49 (2) Από τη λύση του συστήματος ( 1 ) και (2) έχουμε

α = 2 ω = 5 . Άσκηση 14η Να βρείτε πόσα πολλαπλάσια του 7 περιέχονται

μεταξύ του 400 και 500.

ΛΥΣΗ

Ο πρώτος αριθμός μεγαλύτερος του 400 που είναι πολλαπλάσιο του 7 είναι ο 406. Ο τελευταίος είναι ο

αν = 406 + (ν - 1) 7 < 500 ( 1 )

λ ' ( 1 ) ' 1 0 1 ' Από τη υση της προκυπτει ν < - αρα . 7 ν = 14 . Άρα 14 ποf.Jσια τ�υ 7 παρεμβάλλονται με­ταξύ του 400 και 500. Άσκηση 15η Μεταξύ των αριθμών 3 και 48

α) Να παρεμβάλλετε 8 αριθμούς ώστε να έχετε

σύνολο 10 διαδοχικούς όρους αριθμητικής

προόδου.

β) Να παρεμβάλλετε 3 αριθμούς ώστε να έχετε

σύνολο 5 διαδοχικούς όρους γεωμετρικής

προόδου.

ΛΥΣΗ

α) Είναι α1 = 3 α10 = 48

Άρα α10 = α1 + 9ω <:::::> 48 = 3 + 9ω <:::::> ω = 5 Άρα όλοι οι αριθμοί είναι

3, 8, 1 3 , 1 8, 23, 28, 33 , 38, 43, 48 . β) α1 = 3 α5 = 48 άρα α5 = α1λ4 <:::::> 48 = 3λ4 <:::::> λ4 = 16 <:::::> λ ± 2 Άρα όλοι οι αριθμοί είναι

3, 6, 12, 24, 48 ή 3, - 6, 1 2, - 24, 48. Άσκηση 16η Οι πρώτοι δύο όροι μιας αριθμητικής και μιας γε­

ωμετρικής προόδου με θετικούς όρους είναι ίσοι.

Να αποδείξετε ότι όλοι οι άλλοι όροι της αριθ­

μητικής προόδου είναι μικρότεροι από τους α­

ντίστοιχους όρους της γεωμετρικής προόδου.

ΛΥΣΗ

Εάν χ και y είναι ο πρώτος και ο δεύτερος όρος της αριθμητικής και γεωμετρικής προόδου. Ο νιοστός όρος αν = χ + (ν - 1) ( y - χ ) και ο όρος

α� της γεωμετρικής α� = χ (�)ν-Ι Πρέπει να δείξουμε ότι χ + (ν - 1) (y - χ) � χ (�)ν-Ι ή

χ + (ν - 1 ) (y - Χ ) - Χ (�)ν-Ι � Ο <:::::>

χ { [ (�) - ι ] ι ν - ι) + [ι - (�Γ]} � ο �

ω χ(�-ι�(ν-ι) -[ω�' {�Γ' + +(�)+]�ο�

Διακρίνουμε τις κάτωθι περιπτώσεις: 1. > 1 , 1. < 1 , 1. = 1 χ χ χ

Εύκολα αποδεικνύεται η σχέση ( 1 ) Άσκηση 17η Τα μήκη των πλευρών ορθογωνίου τριγώνου

αποτελούν διαδοχικούς όρους Γεωμετρικής

προόδου. Να βρείτε τις εφαπτόμενες των οξειών

γωνιών του.

ΛΥΣΗ

Έστω β, γ οι κάθετες πλευρές και α η υποτείνουσα του τριγώνου.

ΕΥΚΛΕΙΔΗΣ Β ' λθ ' τ.3/34

Page 37: Ευκλειδης Β 59

Μαθηματικά Β ' Λυκείου

Έστω β, γ, α οι διαδοχικοί όροι της προόδου με λ > 1 ( 1 ) γ2 = αβ , α2 = β2 + γ2 <:::::> α2 = β2 + αβ

γ4 = α2β2 = β2 (β2 + γ2 ) = β4 + β2γ2 έχουμε

γ4 = β4 + β2γ2 (2)

( εφΒ =� , εφΓ =�) 4 β2 2

α) Από (2) έχουμε γ4 = 1 + ----:t-

β β

(Π -(�)' - ι � ο y2 - y - ι � ο όπου

y � (� )' και έχουμε y � -J52 + 1

(1)2 = J5 + 1 εφΓ = r = JJ5 + 1 β 2 ' β 2

β) Από ( Ι ) έχουμε � �ω + (Η αν y �ω τότε 2 J5 - 1 y + y - 1 = 0 y = --

2 β JJ5 - 1 εφΒ = - = -- , ομοίως αν Ο < λ < 1 γ 2 α, γ, β.

Προτεινόμενες Ασκήσεις Άσκηση lη . Διπλώστε ένα φύλλο χαρτί εφτά φορές. Μπορείτε περισσότερες; Αν το διπλώνατε 37 φορές τι πάχος θα είχε: α) 1 μέτρο β) 5 μέτρα γ) 100 μέτρα δ) από τη Γη στη Σελήνη περίπου 380.000 Κm; Άσκηση 2η. Αν α, β, γ διαδοχικοί όροι γεωμετρικής προόδου τότε οι ρίζες της χ 2 - 2βχ = α ( 4α - γ) είναι ρητοί αριθμοί.

Άσκηση 3η. Εάν οι αριθμοί α, β, γ, δ με τη σειρά που δίνονται οι τρεις πρώτοι είναι διαδοχικοί όροι γεωμετρικής προόδου ενώ οι 3 τελευταίοι αποτελούν όρους α­ριθμητικής προόδου και ο πρώτος και ο τελευταίος όρος έχουν άθροισμα 14 και ο δεύτερος και ο τρί­τος έχουν άθροισμα 12. Να βρεθούν οι αριθμοί.

Άσκηση 4η . Εάν οι θετικοί αριθμοί α, β, γ είναι όροι αριθμητικής

' δου δ η: ' 1 1 προο να ε�ετε οτι οι Γα r:: , r:: Γ ,

νβ + νγ νγ + να

Jα � .jβ είναι όροι αριθμητικής προόδου .

• ΓΕΩΜΕΤΡΙΑ "Από τα κανονικά πολύγωνα στα καμπυλόγραμμα χωρία"

των Ηλία Αργυρόπουλου - Σπύρου Παναγιωτόπουλου - Βασίλη Καρκάνη

Τα κανονικά πολύγωνα είναι μια ειδική κατηγορία κυρτών πολυγώνων και αποτελούν το βασικό εργα­λείο για την μέτρηση του κύκλου.

Οι ασκήσεις που παρουσιάζουμε αναφέρονται στις ιδιότητες των κανονικών πολυγώνων, στη μέτρηση

του κύκλου και ορισμένων καμπυλόγραμμων χωρίων.

Κάθε άσκηση συνοδεύεται από σχόλιο σχετικά με την σκέψη κλειδί.

ΑΣΚΗΣΗ 1 Δίνεται κυρτό εξάγωνο ΑΒΓ ΔΕΖ το οποίο έχει

τις πλευρές του ίσες και τις γωνίες Α, Β, Γ και Ε

ίσες. Να αποδειχθεί ότι:

α. Το τρίγωνο ΒΔΖ είναι ισόπλευρο

β. Το ΑΒΓ ΔΕΖ είναι κανονικό

ΛVΣΗ

α. Τα τρίγωνα ΑΒΖ και ΒΓ Δ είναι ίσα γιατί έχουν ΑΒ = ΒΓ ΑΖ = Γ Δ και Α = Γ άρα ΒΖ = ΒΔ.

Α Α Όμοια ΑΒΖ = ΔΕΖ οπότε ΒΖ = ΖΔ. Άρα ΒΖ = =ΒΔ =ΖΔ.

ΕΥΚΛΕΙΔΗΣ Β' λθ ' τ.3/35

Page 38: Ευκλειδης Β 59

Μαθηματικά Β ' Λυκείου

β. Από τις προηγούμενες ισότητες των τριγώνων είναι ακόμα ΑΖΒ = ΔΖΕ = ΑΒΖ = ΔΒΓ = ΒΔΓ = zΔΕ απ' όπου ΑΖΕ = ΑΖΒ + 60° + ΔΖΕ = ΑΒΖ + 60° + ΓΒΔ = = Β .Ομοίως ΕΔΓ = Β . Άρα το πολύγωνο έχει και τις γωνίες του ίσες. Οδηγός Σκέψης: Ένα πολύγωνο είναι κανονικό όταν είναι ισόπλευρο και ισογώνιο.

Α Ζ

Γ

ΑΣΚΗΣΗ 2 Δίνεται κανονικό πεντάγωνο εγγεγραμμένο σε

κύκλο (0, R). Στην προέκταση της ΓΔ παίρνου­

με σημείο Ζ έτσι ώστε ΓΖ = Γ Α και στην προέ­

κταση της ΔΓ παίρνουμε το σημείο Η έτσι ώστε

ΔΗ = ΑΔ. Να αποδειχθεί ότι:

α. ΕΖ//ΑΔ και ΒW/ΑΓ

β. (ΑΒΓ ΔΕ) = (ΑΗΖ)

ΛΥΣΗ

α. Εφόσον ΒΓ = ΔΕ � ΒΓ = ΔΕ άρα το ΒΕΔΓ είναι ισοσκελές τραπέζιο. Όμοια είναι ισοσκελές τραπέζιο και το ΑΒΓΕ οπότε ΒΕ = ΑΓ. Έτσι είναι ΒΕ= =ΓΖ οπότε το ΒΓΖΕ είναι παραλληλόγραμμο απ' όπου έχουμε ΕΖ//ΒΓ. Τέλος, επειδή ΑΔ//ΒΓ συμπεραίνουμε ΕΖ//ΒΓ. Όμοια αποδεικνύεται ότι ΒΗ//ΑΓ.

Α

Οδηγός Σκέψης:

Η μεταφορά της κορυφής παράλληλα στη βάση τριγώνου δημιουργεί ισοδύναμα τρίγωνα.

ΑΣΚΗΣΗ 3 Δίνεται τρίγωνο ΑΒΓ εγγεγραμμένο σε κύκλο

(Ο, R) με ΑΓ = β, ΑΒ = γ και ΒΓ =�β2 + γ2 + βγ .

α. Να αποδειχθεί ότι το τρίγωνο είναι αμβλυγώνιο και ότι ΒΓ = λ3 •

β. Να βρεθεί το εμβαδόν των κυκλικών τμη­

μάτων στα οποία η ΒΓ χωρίζει τον κύκλο.

ΛΥΣΗ

α. Είναι ΒΓ>ΑΒ και ΒΓ>ΑΓ. Επίσης ΒΓ2 > ΑΒ2 + ΑΓ2 άρα Α > 90° . Από το Θ συνημιτόνων στο τρίγωνο ΑΒΓ έ­χουμε ΒΓ2 = ΑΒ2 + ΑΓ2 - 2ΑΒ · ΑΓ · συνΑ β2 + γ2 + βγ = β2 + γ2 - 2βγσυνΑ <:::::>

συν Α = _..!_

<:::::> Α = 1 20° οπότε BAr = 1 20° 2

άρα ΒΓ = λ3 . Α

Β ο

Δ

β. Εμβαδόν κυκ. τμημ. ΒΑΓ = (ο.ΒΑΓ) - (ΟΒΓ) =

πR 2 120° 1 πR2 R 2J3 360° 2 · λ3 · α3 = _3 _

_ _ 4_ Εμβαδόν κυκ. τμημ. Mr = πR 2 - Εμβαδόν κυκ. τμημ.

BAr = πR2 _ πR2 + R

2J3 = . . . = (8π + 3J3)R2 3 4 1 2

Οδηγός Σκέψης: Η βοήθεια του Θ. συνημιτόνων στον υπολογισμό γωνιών.

ΑΣΚΗΣΗ 4 β. Τα τρίγωνα ΑΕΔ και ΔΖΑ έχουν ίδια βάση ΑΔ Σε κύκλο (0, R) θεωρούμε τις χορδές ΑΒ = λ6

και ίσα ύψη αφού ΕΖ// ΑΒ άρα (ΑΔΕ) = (ΑΔΖ) και ΒΓ = λ3 • Στο σημείο Α φέρνουμε εφαπτο­και όμοια (ΑΒΓ) = (ΗΓ Α).

ΕΥΚΛΕΙΔΗΣ Β ' λθ ' τ3/36

Page 39: Ευκλειδης Β 59

Μαθηματικά Β ' Λυκείου

μένη του κύκλου που τέμνει την προέκταση της

ΓΒ στο Σ.

α. Να αποδειχθεί ότι η ΑΓ είναι διάμετρος του

' κλ 2 κυ ου και ΒΣ = - α6

β. Να βρεθεί το ΑΣ.

3

Ν λο

θ ,

λ ' (ΑΣΒ) γ. α υπο γισ ει ο ογος --'----'--

(ΑΒΓ)

ΛΥΣΗ α. Επειδή ΑΒ = λ6 θα είναι ΑΒ = 60° άρα η

f = 30 ο • Επίσης ΒΓ = λ3 άρα Br = Ι 20° οπότε η BAr = 60o άρα ΑΒΓ= Ι80° - (60° + 30° ) = 90° οπότε η ΑΓ είναι διάμετρος του κύκλου. Στο ορθογώνιο τρίγωνο ΑΓΣ το ΑΒ είναι ύψος άρα ΑΒ 2 = ΣΒ · ΓΒ <::::> λ� = ΣΒ · λ3 <::::>

<::::> R 2 = ΣΒ · R J3 <::::> ΣΒ = � = �α6 ν3 3

β. Στο ορθογώνιο τρίγωνο· ΑΒΣ η ΣΑΒ = 30° (χορδής εφαπτομένης) άρα

ΑΣ = 2ΣΒ => ΑΣ = 2_!_

= 2RJ3 J3 3

γ. Τα τρίγωνα ΑΣΒ και ΑΒΓ ΑΣΒ = ΒΑΓ = 60° άρα

R 2RJ3 (ΑΣΒ) ΣΒ · ΣΑ J3

·-3- Ι --- = = (ΑΒΓ) ΑΒ · ΑΓ R · 2R 3

Δ Δ Αλλιώς: είναι ΑΣΒ - ΑΒΓ (γιατί;) άρα (ΑΣΒ) = λ2 = (ΑΣ )2

_ _ Ι

(ΑΒΓ) ΑΓ 3

Οδηγός Σκέψης:

έχουν

α) Μια χορδή κύκλου είναι διάμετρος όταν αντι­στοιχεί σε τόξο 1 80°.

β) Το ύψος ορθογωνίου τριγώνου προς την υπο-

τείνουσα παραπέμπει στις μετρικές σχέσεις στο ορθογώνιο τρίγωνο.

γ) Σε τρίγωνα με γωνίες ίσες ο λόγος των εμβα­δών τους ισούται με τον λόγο των γινομένων των πλευρών που περιέχουν τις ίσες γωνίες.

δ) Ο λόγος εμβαδών όμοιων τριγώνων.

ΑΣΚΗΣΗ S Έστω ΑΒ = λν είναι η πλευρά κανονικού ν­

γωνου εγγεγραμμένου σε κύκλο (0, R) και (Κ,

ρν) είναι ο κύκλος ο εγγεγραμμένος στον κυκλι--.. κό τομέα Ο.ΑΒ (δηλαδή ο κύκλος που εφάπτε-ται στις δύο ακτίνες και το τόξο του τομέα). Να αποδειχθεί ότι:

1 1 2 α. - = - + -

ρν R λν

β. για ν = 3 να βρεθεί το εμβαδό του κύκλου

(Κ, ρ3). ΛΥΣΗ α. Το κέντρο του κύκλου βρίσκεται στη διχοτόμο

ΟΜ της ΑόΒ και προφανώς το Μ είναι το μέ­σο του τόξου ΑΒ. Είναι (ΑΟΜ) = (ΟΑΚ) + (ΚΑΜ) ( 1) (ΟΑΜ) = _!_ΟΜ · ΑΗ =

_!_ΟΜ · ΑΒ = _!_ R . � =

2 2 2 2 2 =_!_· R · λ 4 ν

1 1 (ΟΑΚ) = -ΟΑ · ρ = - · R · ρ 2 ν 2 ν Ι 1 ΑΒ (ΚΜΑ) = -ΚΜ · ΑΗ = - · ΚΜ ·- = 2 2 2 .!.

. ρ ·�

=_!_

· ρ · λ 2 ν 2 4 ν ν

ο

Α Μ

Οπότε η ( Ι ) γράφεται:

Β

_!_· R · λ =

_!_· R · ρ +

_!_· ρ · λ <::::>

4 ν 2 ν 4 ν ν 1 2 1 R · λ = 2R · ρ + ρ · λ <::::> - =-+-ν ν ν ν λ R ρν ν

ΕΥΚΛΕΙΔΗΣ Β ' λθ ' τ3/37

Page 40: Ευκλειδης Β 59

Μαθηματικά Β ' Λυκείου

β. Είναι 1 1 2 1 1 2 J3 + 2 - = - + - <::::> - = - +-- = -- <=> ρ3 R λ3 ρ3 R R J3 R J3

<=> ρ3 = � J3 = . . . = (2./3 - 3)R οπότε το ζη-ν3 + 2 τούμενο εμβαδόν είναι Ε3 = πρ� = . . . = 3πR 2 (7 - 4./3) Οδηγός Σκέψης: Η χρήση εμβαδών για εύρεση σχέσης μεταξύ τμημάτων.

ΑΣΚΗΣΗ 6 Δίνεται ημικύκλιο διαμέτρου ΒΓ, κέντρου Ο και

ακτίνας R. Έστω Α σημείο του τόξου ΒΓ ώστε

ΑΒ=λ.; και ΑΔ κάθετη στη ΒΓ. Αν Μ μέσο του

τόξου ΑΓ, και η ΒΜ τέμνει τις ΑΔ, ΑΓ στα Ε, Ζ

αντίστοιχα, τότε:

α. Να βρεθεί η περίμετρος του τριγώνου ΑΒΜ.

β. Να αποδειχθεί ότι η ΑΜ είναι κάθετη στην ΑΔ. γ. Να αποδειχθεί ότι ΑΕ = !λ3 3 δ . Να αποδειχθεί ότι η ΒΜ τριχοτομείται από

τα Ε και Ζ. ΛΥΣΗ

α. Είναι ΑΒ = 60° οπότε ΑΜ = Mr = 60° και ΕΑΜ = 120° (γιατί;) . Άρα ΒΜ = λ3 . Η ζητού-μενη περίμετρος είναι: ΑΒ + ΑΜ + ΒΜ = R+R+RJ3 = 2R + RJ3

β . Επειδή MAr = Μ'Β = 30° είναι ΑΜ//ΒΓ οπότε ΑΜ .l ΑΔ .

Β Δ ο Γ γ. Τα τρίγωνα ΑΒΕ και ΑΒΜ είναι ισοσκελή με

γωνίες βάσης 30° άρα είναι όμοια. Έτσι, έχουμε: ΑΕ ΑΒ ΑΕ R RJ3 -=- <::::> -=-- <::::> ΑΕ=-- <::::> ΑΒ ΒΜ R RJ3 3

1 <::::> ΑΕ-λ3 . 3 δ. Επειδή ΑΕz=ΒΕΔ=60ο και FAz=� -ΒΜ=ω'

το τρίγωνο ΑΕΖ είναι ισόπλευρο, οπότε ΑΖ=ΖΕ=ΑΕ=!� . Άρα ΒΕ=ΕΖ=!ΒΜ=ΖΜ . 3 3

Οδηγός Σκέψης:

α) Οι χορδές με μήκος � αντιστοιχούν σε τόξα 60°.

β) Η ομοιότητα τριγώνων οδηγεί σε λόγους τμημάτων .

ΑΣΚΗΣΗ 7 Δυο κύκλοι (Κ, R) και (Λ, ρ) εφάπτονται εξωτε­

ρικά. Ο κύκλος (Μ, σ) εφάπτεται εξωτερικά

στους (Κ, R), (Λ, ρ) και εσωτερικά στον κύκλο

διαμέτρου ΚΛ. Ν α αποδειχθεί ότι:

1 2 2 α. - = - + -

β.

γ.

σ R ρ 1 2 2

ό S L D ' ' S = L + D που , , τα μηκη των κυ-

κλων (Μ, σ), (Κ, R) και (Λ, ρ) αντίστοιχα

�ER · Ε ρ � 16Εσ , όπου ER ,Ε ρ ,Ε σ τα εμ-

βαδά των τριών κύκλων αντίστοιχα. ΛΥΣΗ

α. Έστω Α το μέσο του ΚΑ. Τότε ΚΜ = R+σ ,

ΜΛ = ρ+σ ΚΛ = R + ρ ΑΜ = R + ρ - σ ' 2

κ

Με εφαρμογή του πρώτου θεωρήματος των διαμέσων στο τρίγωνο ΚΜΛ έχουμε

ΚΛ2 ΜΚ2 + ΜΛ2 = 2ΜΑ2 +-- <::::> 2 (R + σ)2 + (ρ + σ)2 = 2(R;ρ - σ Υ + (Ri)2

<::::> . . . <::::> 2Rσ + 2ρσ = Rρ , και διαιρώντας με R ' 1 2 2 ρς παιρνουμε - = - + - . σ R ρ

β. Πολλαπλασιάζουμε και τα δυο μέλη της σχέσης 1 2 2 1 ' 1 2 2 - = - +- με το - και βρισκουμε - =-+-σ R ρ 2π S L D

γ. Από την ισότητα ΕΥΚΛΕΙΔΗΣ Β' λθ ' τ3/38

Page 41: Ευκλειδης Β 59

Μαθηματικά Β ' Λυκείου

1 2 2 1 2(R + ρ) 2 · 2� - =-+- <=> - = ;?: <=> σ R ρ σ R · ρ R · ρ

<=> _!_ �-4- <=> � ;?: 4ρ => R · ρ ;?: 1 6ρ2 σ �

<=> πR · ρ ;?: 16πρ2 <=> �ER · Ερ ;?: 1 6Εσ

Οδηγός Σκέψης: Η διάμεσος μας οδηγεί στην α­ξιοποίηση του 1 ou Θ. Διαμέσων για την εύρεση σχέσης μεταξύ R, ρ, σ.

ΑΣΚΗΣΗ 8 Δίνεται κανονικό εξάγωνο ΑΒΓ ΔΖ εγγεγραμμέ­

νο σε κύκλο (0, R). Αν Μ είναι το μέσο της

πλευράς του ΑΒ και η ΓΜ τέμνει τον κύκλο στο

Ν, να δειχθούν:

α. ΓΜ = RJ7

β.

γ.

2 Τα τρίγωνα ΜΒΝ και ΑΜΓ είναι όμοια κα­

θώς και τα τρίγωνα ΜΒΓ και ΑΜΝ και να

υπολογισθούν τα τμήματα ΒΝ και ΑΝ.

(ΑΝΒ) = J2Ϊ (ΑΒΓ ΔΕΖ) 42

ΛΥΣΗ

α. Στο τρίγωνο ΑΒΓ η ΓΜ είναι διάμεσος. Με εφαρμογή του 1 ou Θ. διαμέσων έχουμε:

2ΒΓ2 + 2ΓΑ2 - ΑΒ2 ΓΜ2 = --------------4

2R2 + 2 (R·J3Y - R2 7R2 , = =--- οποτε

ΓΜ = R.J7 2

4 4

Ε

β. Τα τρίγωνα ΜΒΝ, ΑΜΓ είναι όμοια (γιατί;) R

ΒΝ ΒΜ ΒΝ 2 1 άρα - =- <=> -- = -- = - <=> ΑΓ ΜΓ RJ3 RJ3 .J7

<=> ΒΝ = R .J2ι 7

2

Ανάλογα από την ομοιότητα των τριγώνων ΒΜΓ και ΑΜΝ παίρνουμε:

R ΑΝ = ΜΑ <=> ΑΝ = 2 = -�- <=> ΑΝ = � ΒΓ ΜΓ R R.J7 .J7 .J7

2 1 R2 R2J3 γ. (ΑΒΓ) = -R · R · ημ1 20° =-ημ60° =--2 2 4

1 1 RJ3 R2J3 (ΑΒΓΔΕΖ) = 2Ρ6 · α6 = 26R ·-2-= 6-4-

= 6(ΑΒΓ) λ

ι - λ ι -­Επειδή ΑΝΒ =-ΑΖΒ και ΒΓ Α = -ΑΝΒ είναι

2 2 ANB+ Bf'A =�(ΑΖΒ+ΑΝΒ) =� 360° = 1 80°

Οπότε:

(ΑΝΒ) ΝΑ · ΝΒ -'------'- = ------(ΒΑΓ) ΒΑ · ΒΓ

(ΑΝΒ) = .J2ι (ΑΒΓ) <=> 7

(ΑΝΒ) = .J2ι ·.!_(ΑΒΓΔΕΖ) = .J2ι (ΑΒΓΔΕΖ) 7 6 42

Οδηγός Σκέψης: α) Η διάμεσος τριγώνου συναρτήσει των πλευ­

ρών του εκφράζεται με εφαρμογή του ι οu Θ. Διαμέσων.

β) Η ομοιότητα συνήθως αντιμετωπίζεται με τον έλεγχο των γωνιών.

γ) Αν μια γωνία ενός τριγώνου είναι παραπλη­ρωματική με μια γωνία ενός άλλου τότε λό­γος των εμβαδών τους ισούται με τον λόγο των γινομένων των πλευρών που περιέχουν τις παραπληρωματικές γωνίες.

ΑΣΚΗΣΗ 9 Θεωρούμε κανονικό εξάγωνο ΑΒΓ ΔΕΖ εγγε­

γραμμένο σε κύκλο (0, R). Οι διαγώνιές του

ΑΕ και ΖΓ τέμνονται στο Κ και η ΒΚ τέμνει

την ΖΕ στο Λ. Ν α αποδείξετε ότι: α. η ΑΕ είναι κάθετη στη ΓΖ.

β. ΕΚ2 = ΚΖ · ΚΓ

ΕΥΚΛΕΙΔΗΣ Β ' λθ ' τ3/39

Page 42: Ευκλειδης Β 59

Μαθηματικά Β ' Λυκείου

1 1 1 γ. -- =-- + --

ΑΚ2 ΑΖ2 ΕΓ2

δ. ΛΕ = �R 3 R2J3

ε. (ΛΕΚ) =--12

ΛΥΣΗ ____.._

Ε. ΕΚΓΑ _ ΕΔΓ+ΖΑ _ 1sσ _900 . ΑΕ ..l ΓΖ

α. ιναι - 2 - 2 - αρα

Ε.�----�

β. Το τρίγωνο ΖΕΓ είναι ορθογώνιο στο Ε (γιατί;) και το ΕΚ είναι το ύψος τσu οπότε ΕΚ 2 = ΖΚ · ΓΚ

γ. Το τρίγωνο ΖΑΓ είναι ορθογώνιο στο Α και

Ακ ' ' ' 1 1 1 ειναι υψος του αρα ΑΚ 2

= ΖΑ 2

+ ΑΓ2

και επειδή το τρίγωνο ΕΑΓ είναι ισόπλευρο θα ' ΑΓ ΕΓ ' 1 1 1 ειναι = οποτε

ΑΚ2 =

ΑΖ2 +

ΕΓ2

δ. Τα τρίγωνα ΖΛΚ και ΚΒΓ είναι όμοια γιατί ΛΖΚ = 60° και KfB = 60° και zΚΛ = ΒΚΓ (κατά κορυφήν) οπότε:

R ΖΛ ΖΚ ΖΛ R - α3 ΖΛ R - 2 - = - <=> - =--<::::> - =-- = ΒΓ ΚΓ R R+α3 R R + R

1 R = - <::::> ΖΛ = -3 3

Τέλος ΛΕ = ΖΕ - ΖΛ = R -R = 3_R 3 3

2

Ε. (ΛΕΚ) = ..!.. ΕΛ · ΕΚημ30° = ..!.. . 3.R . λ3 . ..!.. = 2 2 3 2 2

R RJ3 R2J3 --- = --3 4 12

Οδιιγtίc: Σlί{ψικ: α ) Γωνία τεμνόμενων χορδών. β ) Το ύψος ορθογωνίου τριγώνου που αντι­

στοιχεί στην υποτείνουσα ικανοποιεί γνω­στές ισότητες.

γ) Ο λόγος τμημάτων υπολογίζεται και με α­ξιοποίηση της ομοιότητα τριγώνων.

ΑΣΚΗΣΗ Ι Ο Δίνεται ο κύκλος (0, R) και η χορδή ΑΒ = λ4 • Στα δυο κυκλικά τμήματα που χωρίζει η χορδή τον κύκλο εγγράφουμε κύκλους (Κ, ρ1) και (Λ, ρ2). Να βρεθεί α. το εμβαδόν του χωρίου που είναι εσωτερικά

του κύκλου (0, R) και εξωτερικά των δυο κύκλων (Κ, ρι) και (Λ, ρ2).

β. το εμβαδόν του κύκλου (Λ, ρ2). ΛΥΣΗ

α. Είναι φανερό ότι τα τρία κέντρα των κύκλων βρίσκονται πάνω στην μεσοκάθετο του ΑΒ. Έτσι 2ρ1 + 2ρ2 = 2R ή ρ1 + ρ2 = R . Το τρίγω-

νο ΔΓΒ είναι ορθογώνιο με ύψος το ΒΖ. Τότε

ΓΖ . ΔΖ = ΖΒ2 <=> 2ρ1 • 2ρ2 = ( � Υ <=>

R2 <=> ρ, · ρ2 = -g

Δ

Το ζητούμενο εμβαδό θα βρεθεί αν από το εμ-βαδό του κύκλου (0, R) αφαιρέσουμε τα εμβα­δά των κύκλων (Κ, ρ , )και (Λ, ρz) άρα: Ε = πR2 - π (ρ� + ρ0 =

= πR2 - π [ (ρ 1 + ρ2 )2 - 2ρ1 · ρ2 ] =

2 ( 2 R 2

J πR2 = πR - π R - 28 = 4 .

β. Είναι ΟΖ = R - 2ρ2 , αλλά ΟΖ = α4 οπότε

R - OZ R-� R (2 - h) ρ2 =

2 =

2 =

4 Έτσι το ζητούμενο εμβαδό είναι

, 2 R2 (2 - ν'2)2 πR2 (6 - 4ν'2) Ε = πρ2 = π = ---'---�

1 6 1 6

ΕΥΚΛΕΙΔΗΣ Β ' λθ ' τ3/40

Page 43: Ευκλειδης Β 59

Μαθηματικά Β ' Λυκείου

Οδηγός Σκέψης:

Επειδή ο υπολογισμός του ζητούμενου εμβαδού,

ανάγεται στον υπολογισμό του γινομένου ρ ιρ2 θεωρούμε το ορθογώνιο τρίγωνο ΒΓ Δ για να υ­

πολογίσουμε το γινόμενο αυτό.

ΑΣΚΗΣΗ 1 1 Δίνεται ο κύκλος (0, 5) και η χορδή ΑΒ = 5 . -Έστω Μ μέσο του κυρτογώνιου τόξου ΑΒ . Ε-πίσης η ΜΝ διάμετρος που τέμνει την ΑΒ στο Γ. Να υπολογιστούν: α. το μήκος του τμήματος ΑΜ

β. το εμβαδό του τριγώνου ΑΓΝ

γ. εμβαδό Ει του κυκλικού τμήματος που ορίζεται ...-από τη χορδή ΑΜ και το κυρτοyώνιο τόξο ΑΜ .

δ. το εμβαδό � του μικτόyραμμου τριγώνου ΑΜΒ. ΑΥΣΗ

Φέρνουμε τις ακτίνες ΟΑ, ΟΒ και την ΑΝ. � Έχουμε ΑΒ = R = � = 5 οπότε ΑΒ = 60° .

Επίσης ΟΜ .l ΑΒ άρα ΟΓ = α6 = R J3 = 5

J3 2 2

Μ Β

I

Ν α. Στο ορθογώνιο τρίγωνο ΑΜΝ είναι:

ΑΜ2 = ΜΝ · ΜΓ = ΜΝ (ΟΜ - ΟΓ) =

= 2R (R - α6 ) = . . . = 25 (2 -J3) άρα

ΑΜ = 5�2 -J3 β. Είναι:

1 1 1 (ΑΓΝ) = -ΑΓ · ΓΝ = - · -ΑΒ · (ΟΓ + ΟΝ) = 2 2 2

1 25 h = -λ6 (α6 + R) = . . . = -(ν3 + 2) 4 8

--- 1 �

γ. Επίσης: ΑΜ =-ΑΒ = 30° οπότε: 2

Ε ι = (0, ΑΜ) - (ΟΑΜ) =

= πR 2 • 30ο -_!_ R . R 30ο = . . . = 25 (π - 3) 360° 2 ημ

1 2

δ . Το ζητούμενο εμβαδό θα βρεθεί αν από το εμ­βαδό Ε3 του κυκλικού τμήματος που ορίζεται

� από τη χορδή ΑΒ και το κυρτογώνιο τόξο ΑΒ αφαιρέσουμε το εμβαδό Ε ι . Δηλαδή : Ε2 = Ε3 - Ει =

( (Ο,ΑΒ) - (ΟΑΒ)) - Ει = ( πR 2 60° 1 J = 360ο -2 R . R . ημ60ο - Ε ι = . . . =

= �� (π + 3 - 3J3)

Οδηγός Σκέψης: Γνωστές χορδές (λ3, �. �) ορίζουν γνωστά aπο­

στήματα ( α3, α4, α6) καθώς και γνωστά τόξα

( 1 20°, 90°, 60°) . ΑΣΚΗΣΗ 1 2 Δίνεται το τρίγωνο ΑΒΓ με πλευρές ΑΒ =2, ΒΓ = 4, ΑΓ = 2J3 • Ο εγγεγραμμένος κύκλος του

τριγώνου (Κ, ρ) εφάπτεται στις πλευρές ΒΓ και

ΑΒ στα σημεία Μ, Ν αντίστοιχα. α. Να βρεθεί το είδος του τριγώνου ΑΒΓ. β. Να βρεθεί το εμβαδόν Ε του κύκλου (Κ, ρ). γ. Να δειχθεί ότι ΒΜ = 3 - J3 δ. Να βρεθεί η περίμετρος Π και το εμβαδόν

Ει του μεικτόγραμμου τριγώνου ΒΜΝ. ΑΥΣΗ

Α

Γ

Β α. Είναι ΒΓ2 = 42 = 16 ,

ΑΒ2 + ΑΓ2 = 22 + (2J3)2 = 1 6 δηλαδή : ΒΓ2 = ΑΒ2 + ΑΓ2 οπότε το τρίγωνο ΑΒΓ είναι το ορθογώνιο με ορθή γωνία την Α.

β. Λόγω του (α) είναι: (ΑΒΓ) = _!_ΑΒ · ΑΓ =

_!_, 2 · 2J3 = 2J3

2 2 '

Επίσης (ΑΒΓ) = τ · ρ οπότε: (ΑΒΓ) 2J3 4J3

ρ = -2- = ΑΒ + ΒΓ + ΑΓ = 6 + 2J3 . . .

2 = J3 - 1 . Έτσι Ε = πρ2 = π(J3 - 1)2 = . . . = 2π(2 -J3)

γ. Εφόσον ΑΒ = ΒΓ στο ορθογώνιο τρίγωνο 2

ΑΒΓ θα είναι: Γ = 30° άρα Β = 60° . Φέρνουμε ΕΥΚΛΕΙΔΗΣ Β ' λθ ' τ3/41

Page 44: Ευκλειδης Β 59

Μαθηματικά Β ' Λυκείου

τα τμήματα ΒΚ, ΚΜ, ΚΝ. Η Β1 = 30° και ΚΜ ΚΜ l.. ΒΜ (γιατί;) οπότε εφ30° = -- ή ΒΜ

ΚΜ ρ 3 ( ..[3 - ι) ΒΜ = εφ30ο = ..{3 = ..{3 = . . . = 3 - .J3

3 δ . Είναι ΒΝ = ΒΜ = 3 - ..[3 και ΜΚΝ = 120°

. ' πρ · l 20° 2(..{3 - l)π (γιατι;) οποτε SM�N = = . . . = -'-----'--1 800 3 Επίσης ΒΝ = ΒΜ = 3 - ..{3 . Έτσι η ζητούμενη περίμετρος είναι: Π = ΒΜ + ΒΝ + S� = 2BM + S� = ΜΝ ΜΝ

6(3 - .J3) + 2(.J3 - l)π = 3 Ακόμη το ζητούμενο εμβαδόν είναι: Ε = (ΒΜΚΝ) - (Κ,ΜΝ) = 2(ΒΜΚ) - πρ;�:οο =

1 πρ2 = 2 · -ΒΜ · ΚΜ -- = 2 3 = (3 - .J3) · (.J3 - 1) - π(.J3 - 1)2 = . . . = 3 12.J3 - 1 8 - π(4 - 2.J3)

3 Οδιιγi)ς Σκ{ψης: α. Το είδος ενός τριγώνου το διαπιστώνουμε με

τις γνωστές μετρικές σχέσεις. β. Οι γνωστοί τύποι εμβαδού μας δίνουν τη δυ­

νατότητα υπολογισμού τμημάτων. γ. Η χρήση τριγωνομετρίας μας δίνει τη δυνατό­

τητα να υπολογίζουμε ευθόγραμμα τμήματα.

Α � Κ Η Σ Ε Ι Σ Γ ΙΑ Λ ΥΣΗ

ΑΣΚΗΣΗ 1 3 Δίνεται ο κύκλος (0, 5) και οι παράλληλες χορδές του ΑΒ, ΔΓ έτσι ώστε το Ο να βρίσκεται ανάμεσά τους. Αν είναι ΑΔ = ΒΓ = 5.J2 και ΑΔΓ = 75° να υπολογιστούν: α. τα μήκη των χορδών ΑΒ, Γ Δ β . τα εμβαδά Ε 1 , Ε2 των κυκλικών τμημάτων που

ορίζονται από τις χορδές ΑΒ, ΑΔ και τα αντί­στοιχα κυρτογώνια τόξα τους.

ΑΣΚΗΣΗ 1 4 Δίνεται το τετράγωνο ΑΒΓ Δ εγγεγραμμένο στον κύκλο (0, R) και τα μέσα Ε, Ζ, Η, Θ των τόξων ΑΒ, ΒΓ, Γ Δ, ΔΑ αντίστοιχα.

α . Να υπολογιστεί το εμβαδό του τετραπλεύρου ΟΑΕΒ

β. Να υπολογιστεί το εμβαδόν Ε του οκταγώνου ΑΕΒΖΓΗΔΘ.

γ. Αν το κυκλικό τμήμα που ορίζει η χορδή ΑΘ -με το κυρτογώνιο τόξο ΑΘ έχει εμβαδό π - 2.J2 τότε: i ) να υπολογιστεί η ακτίνα R

' λ " ) λ ' λ ' Ε2 του κυκ ου ιι να υπο ογιστει ο ογος -Ε3 όπου Ε2 το εμβαδό που περικλείεται ανάμεσα στον κύκλο και το οκτάγωνο και Ε3 το εμβαδό που περικλείεται ανάμεσα στο οκτάγωνο και το τετράγωνο.

Χρήστος Λαζαρίδης Το παρακάτω άρθρο ασχολείται με το κεφάλαιο των κωνικών τομών και έχει επαναληπτικό χαρακτήρα. Αποτελείται από 8 θέματα, τα οποία αποσκοπούν να συνδυάσουν όλες τις κωνικές τομές. Αν κάποιος ασχοληθεί, καλό είναι να γνωρίζει πολύ καλά την θεωρία και, αν είναι δυνατόν, τις ασκήσεις

του σχολικού βιβλίου.

Θ Ε Μ Α l χ2 y2

Δίνεται η έλλειψη C : 2 + 2 = 1 , α > β > Ο , με α β

εστίες Ε·, Ε (Ε είναι εκείνη με θετική τετμημένη).

Επίσης Α', Α είναι τα άκρα του μεγάλου της άξο­να και Β, Β· τα άκρα του μικρού της άξονα. ί ) Ν α αποδείξετε ότι:

α) ..!!. = Jι - ε2 , όπου ε είναι η εκκεντρότη­α τα της C.

ΕΥΚΛΕΙΔΗΣ Β ' λθ ' τ3/42

Page 45: Ευκλειδης Β 59

Μαθηματικά Β ' Λυκείου -------------β) Το εμβαδόν του τριγώνου ΒΕΑ ισούται

α2 (ι - ε)� με

2 •

ii) Θεωρούμε την παραβολή με κορυφή Ο και εστία την εστία Ε της έλλειψης C. Αν το σημείο Μ ( 6β,2β.J6) ανήκει στην

παραβολή, να αποδείξετε ότι: J2 α) ε = -2

β) ο κύκλος διαμέτρου ΕΕ ', διέρχεται από τις κορυφές Β, Β ' της έλλειψης C.

ΛVΣΗ Υ

Β '(Ο,-β) ί ) α) ε = l . α

Έχουμε: β2 2 β2 β2 = α2 - γ2 =>- = l -L =>- = l - ε2 => α2 α2 α2

l!_ = Jk! . α β ) (ΒΕΑ) =� (ΕΑ) · (ΒΟ) = � ( α - γ) · β =

= .!.α2 (� -1.) . Ι!_ = .!.α2 (1 - ε )�1 - ε2 . 2 α α α 2 i ί ) Το Ε ( γ,Ο) είναι εστία της παραβολής, άρα,

Ε. = γ ή p = 2γ . 2 Η παραβολή έχ.,ει εξίσωση y2 = 2px ή y2 = 4γχ . Το Μ ανήκει στην παραβολή άρα θα επαληθεύ­ει την εξίσωσή της. ( 2βJ6)2 = 4γ6β => 24β2 = 24γβ => β = γ .

α) β = γ => l!_ = 1. => �1 - ε2 = ε => l - ε2 = ε2 => α α 2ε2 = ι => ε2 = _!. => ε = J2 . 2 2

β) Ο κύκλος έχει κέντρο Ο και ακτίνα {ΕΕ' ) ρ = -2- = γ = β , άρα θα έχει εξίσωση,

χ2 + y2 = β2 . Ο κύκλος προφανώς διέρχεται από τα Β (Ο, β) και Β' (0, -β) .

ΘΕΜΑ 2

Δίνεται η παραβολή C : y2 = 4χ και το τυχαίο σημείο της Α(χι , Υι ) , όπου Χι > 0 . Η εφαπτο­μένη ε της παραβολής C στο Α(χι ,Υι ) τέμνει

τους άξονες χ' χ και ψ'ψ στα Β, Γ αντίστοιχα. ί ) Να αποδείξετε ότι Β ( -χυΟ) , Γ( Ο, Υ; ) και

ότι το Γ είναι το μέσο του ευθυγράμμου τμήματος ΑΒ.

ί ί ) Ν α αποδείξετε ότι η απόσταση d της εστίας Ε της παραβολής από την εφαπτομένη ε, ισούται με d = �χι + ι και ότι η απόσταση των σημείων Β, Γ ισούται με (BΓ) = Jxi + Χι .

ί ί ί ) Να υπολογίσετε τις συντεταγμένες του ση­μείου Α, αν το εμβαδόν του τριγώνου ΕΓΒ ισούται με 5.

i ν) Α ν Α (ι, 2) , να υπολογίσετε την εξίσωση

της έλλειψης με εστίες τα σημεία Ε, Β, η οποία διέρχεται από το σημείο Γ.

ΛVΣΗ

Α� / Β Ο� E( l ,O)

Α (χι , Υι ) ε C , άρα, y� = 4χι . Η εφαπτομένη ε έχει εξίσωση, ε : ΥΥι = ρ ( χ + Χ ι ) <=> ΥΥι = 2 (χ + χ ι ) <=> 2χ - yι y + 2Χ ι = 0 .

i) Αν χ = 0 : 2χ 4χ y2 Υ ΥΥι = 2χι => y =-ι => y = -ι = -ι = __!_ , Υι 2yι 2yι 2

άρα, r( ο, i ) .

Αν y = Ο : Ο = 2 ( χ + Χ ι ) => χ = -χ ι , άρα,

ΕΥΚΛΕΙΔΗΣ Β' λθ ' τ3/43

Page 46: Ευκλειδης Β 59

Μαθηματικά Β ' Λυκείου

Β (-χ1 , 0) .

Το μέσο του ΑΒ είναι: ( Χ ι ; Χ ι , Υ ι; 0 ) δηλα-δή (Ο, i} το οποίο προφανώς είναι το Γ.

ii) Ε (� , Ο} άρα, Ε (1 , 0) .

d = d(E, ε) = l2 + 2xι l = 2l 1 + xι l = 2l l 1 + xι l χ�ο �4+ y� �4+ y�

�4+4χ1

2 ( 1 + Χ ι ) ,-;-;--::-=

2Fx; =ν1 + χ ι .

(ΒΓ)2 = (0 + χ 1 )2 + (i - οΥ = x� +yl = χ� + χι ,

άρα, (ΒΓ) =�χ� + χ 1 •

iii) (ΕΓΒ) = _!_(ΒΓ) · d = _!_�χ� + χ 1 Fx; = 2 2

= _!_�χ ι ( χ ι + 1)Fι+J =_!_Fι · (χ ι + 1) .

2 2 (ΕΓΒ) = 5 .

Άρα, _!_ Fι · (χ ι + 1) = 5 � Fι · ( χ ι + 1 ) = 10 . 2

Θέτουμε Fι = y > Ο , άρα έχουμε την εξίσω-ση : Υ ( / + 1) = 10 � y3 + y - 1 0 = Ο �

� (y - 2) ( y2 + 2y + 5) = 0 (Με τη βοήθεια Homer) � y = 2 . Άρα, Fι = 2 � χ 1 = 4 , οπότε, y� = 4 · 4 ή Υι = ±4 . Τελικά, Α(2, 4) ή Α(2, -4) .

ίv) Αν Α(1, 2) τότε Β (- 1, 0) και Ε (1, 0) . Η εστια­κή απόσταση είναι 2γ = ( ΒΕ) = 2 , άρα, γ = 1 . Το σημείο Γ(Ο, 1 ) είναι σημείο της έλλειψης, άρα, 2α = (ΓΒ) + (ΓΕ) = J2 +J2 = 2J2 ή α = J2 .

Ισχύει: β2 = α2 - γ2 = 2 - 1 = 4 . 2 2 Τελικά η έλλειψη έχει εξίσωση , � + L = 1 . 2 1

ΘΕΜΑ 3 Δίνεται η έλλειψη C1 : χ2 + 2y2 = 2κ2 και η πα­

ραβολή c2 : y2 = 4κχ ' όπου κ > ο .

i) Να υπολογίσετε την εκκεντρότητα της έλ­λειψης C1 και να αποδείξετε ότι μία εστία της έλλειψης C1 συμπίπτει με την εστία της παραβολής c2 •

ii) Να αποδείξετε ότι η διευθετούσα δ της πα­ραβολής c2 τέμνει την έλλειψη c. στα

σημεία Α ( -κ, κf) και +κ, -

κf) .

ίίί) Αν η εφαπτομένη της έλλειψης C1 στο ση-

, Α ( κfi ) 'ζ ' ξ μειο -κ,-

2- σχηματι ει με τους α ο-

νες τρίγωνο με εμβαδόν 4J2 , να υπολογί­

σετε την τιμή του κ. ΛΥΣΗ

χ2 y2 Cι :-2 +---τ = 1

2κ κ C2 : y2 = 2 (2κ) χ .

ί) β2 = α2 - γ2 , άρα, γ2 = α2 - β2 = 2κ2 - κ2 = κ2 ή γ = κ .

γ κ J2 ε = - =--=-α κJ2 2 ·

p = 2κ , άρα η εστία της C2 είναι Ε ( κ, Ο) η οποία προφανώς συμπίπτει με μία εστία της Cι .

ί ί) δ : χ = _Ε. ή χ = -κ . Αντικαθιστούμε τις συ-2

ντεταγμένες των Α, Β στις εξισώσεις των δ και C1 , διαπιστώνουμε ότι τις επαληθεύουν, άρα η δ τέμνει την C1 στα Α, Β .

iii) Η εφαπτομένη ε της C1 στο Α, είναι:

ΕΥΚΛΕΙΔΗΣ Β' λθ ' τ3/44

Page 47: Ευκλειδης Β 59

Μαθηματικά Β ' Λυκείου

κJ2 χ (-κ) y-

2- χ yJ2

ε :-- + = 1 <:::::> --+-- = 1 <:::::> 2κ2 κ2 2κ 2κ

-χ + yJ2 - 2κ = Ο .

Αν χ = Ο , τότε, y = � = κJ2 . Αν y = O , τό-

τε, χ = -2κ . Η ε τέμνει τους άξονες στα Κ( -2κ, Ο) και

Λ (ο, κJ2) . (ΟΚΛ) = � (ΟΚ) (ΟΛ) = i l-2κl lκJ2 1 = κ2J2 .

Έχουμε:

Θ Ε ΜΑ 4

Δίνεται ο κύκλος C1 : (χ - α )2 + y2 = 4α2 και η

παραβολή c2 : y2 = 4αχ ' όπου α > ο .

ι) Να αποδείξετε ότι ο κύκλος C1 έχει κέντρο

την εστία της παραβολής C2 και εφάπτε-

ται στη διευθετούσα της. ιι) Να αποδείξετε ότι ο κύκλος C1 και η πα-

ραβολή C2 τέμνονται στα σημεία Α (α, 2α) και Β( α, -2α) . (Εναλλακτικά να υπολογί­

σετε τα σημεία τομής του κύκλου και της

παραβολής).

ιιι) Να αποδείξετε ότι οι εφαπτόμενες της πα­ραβολής C2 στα Α, Β είναι κάθετες και τέ-

μνονται πάνω στη διευθετούσα της.

ΛΥΣΗ

χ

δ

i) Ο κύκλος έχει κέντρο Κ ( α, Ο) . Η παραβολή

έχει εστία, (� , Ο) ή (α, Ο) , η οποία προφανώς

συμπίπτει με το Κ. Η διευθετούσα της C2 , είναι,

δ : χ = _Ε. <:::::> χ = -α <:::::> χ + α = Ο . 2

Έχουμε d (Κ, δ ) = 2α , άρα, η δ είναι εφαπτομέ­νη του Cι .

ii) Αποδεικνύουμε ότι οι συντεταγμένες των Α, Β επαληθεύουν τις εξισώσεις των Cι , C2 , άρα Cι , C2 τέμνονται στα Α, Β.

iii) Οι εφαπτόμενες C2 στα Α, Β, αντίστοιχα είναι, ε ι : y · 2α = 2α ( χ + α) <:::::> y = χ + α ε2 : y (-2α) = 2α (χ + α) <:::::> y = -χ - α λε, · λε, = 1 · ( - 1) = -1 , άρα, ε ι l. ε2 . Λύνοντας το σύστημα των εξισώσεων των ε ι , ε2 , προκύπτει το κοινό τους σημείο (-α, Ο) , το οποίο προφανώς ανήκει στη διευθετούσα δ.

ΘΕΜΑ S

χ2 y2 Δίνεται η έλλειψη C1 : 2 + 2 = 1 , α > β > Ο , με

α β

εστιακή απόσταση 2γ.

i ) Να υπολογίσετε την εξίσωση του κύκλου με

διάμετρο τον μεγάλο άξονα έλλειψης.

i i ) Να βρείτε τα σημεία τομής Α, Β της έλλει­ψης με την ευθεία χ = γ .

iii) Θεωρούμε την εφαπτομένη της έλλειψης, στο σημείο τομής της με την ευθεία χ = γ ,

το οποίο έχει αρνητική τεταγμένη. Να απο­

δείξετε ότι η εφαπτομένη έχει συντελεστή

διευθύνσεως ίσο με την εκκεντρότητα της

έλλειψης.

iv) Να υπολογίσετε την εξίσωση της παραβο­

λής με κορυφή Ο, άξονα συμμετρίας χ'χ, η

οποία διέρχεται από τα Α, Β.

(Εναλλακτικά η έλλειψη να έχει εξίσωση 2 2

C : � + L = 1 , οπότε οι πράξεις θα είναι 25 9

ευκολότερες)

ΛΥΣΗ

(α,Ο) χ

ΕΥΚΛΕΙΔΗΣ Β ' λθ ' τ3/45

Page 48: Ευκλειδης Β 59

Μαθηματικά Β ' Λυκείου

i) Τα άκρα του μεγάλου άξονα της C1 είναι (-α, Ο) και (α, Ο) . Ο κύκλος έχει κέντρο Ο και ακτίνα α, άρα, θα έχει εξίσωση, χ2 + y2 = α2 .

ί ί ) Λύνουμε το σύστημα της εξίσωσης της C1 και της

χ =γ ,αιώtε�.ΟΟΟW +-�) και Β(Ύ·�} iii) Η εφαπτομένη της C1 , στο Α , είναι

y (- β2 ) χγ +

α = 1 �

α2 β2 γχ Υ 2 2 --- = l � γχ - αy = α � αy = γχ - α � α2 α

γ Υ =-χ - α � y = εχ - α . α

β) Αν ε η εκκεντρότητα της C2 , τότε,

ε < l � l. < l � .!..

< l � α > l . α α

γ) ε = .fi

� r = .fi

�.!_

= .fi

� α =.fi

. 2 α 2 α 2

β2 = α2 - γ2 = (.J2)2 - 12 = 1 , άρα, β = l .

2 Τελικά, C1 :� + y2 = 1 . 2

ΘΕΜΑ 7

Έστω οι ευθείες ε1 : λχ + λy = 2 και ε2 : χ - y = 2λ , όπου λ Ε IR* . α) Να αποδείξετε ότι το σημείο τομής των ευ­

θειών ανήκει σε μία ισοσκελή υπερβολή C. β) Ν α υπολογίσετε τις εστίες Ε ', Ε της C.

ίv)Έστω c2 : y2 = 2ρχ Α Ε c2 άρα, γ) Να εξετάσετε αν υπάρχει σημείο Μ που να β4 β4 z = 2pγ � 2ρ =-

2 . α γα

τ λ . c 2 β4 ε ικα, 2 : y = -2 χ . γα

ΘΕΜΑ 6

Η εστία της παραβολής C1 : y2 = 4χ συμπίπτει με 2 2

μία εστία της ω...ειψης c2 : χ2 +

y2 = 1 α > β > ο .

α β α) Να αποδείξετε ότι το σημείο Μ (α,β) κι-

νείται σε μία ισοσκελή υπερβολή. β) Να δείξετε ότι α > 1 .

γ) Α ν επιπλέον η εκκεντρότητα της ω...εtψης είναι

J2 β , εξ' .ι:.� � -2 , να ρειτε την ισωση της �ιψης.

ΑΥΣΗ

Η C1 έχει εστία ,�,ο} όπου ρ = 2 , άρα, E (l, O) .

Η C2 έχει εστίες (-γ, Ο) , (γ, Ο) . Συμπεραίνουμε ότι: γ = 1 . α) Έστω M (x, y) τυχαίο σημείο, τότε:

χ = α}� χ2 = α2 }� χ2 - y2 = α2 - β2 �

y = β y2 = β2

� χ2 - y2 = γ2 � χ2 - y2 = 1 ' που είναι εξίσωση ισοσκελούς υπερβολής.

ανήκει στην ευθεία y = -χ και {ΜΕ) - {ΜΕ' ) = 8 .

ΛΥΣΗ

α) D = Ιλ λ I = -λ - λ = -2λ * Ο . 1 -1

Dx = lξλ �� Ι = -2 - 2λ2 = -2(λ2 + 1)

οΥ = I� ξλ l = 2λ2 - 2 = 2(λ2 - ι)

X = � j<c>

Χ = λ': J !

Dy λ2 - 1 y =o y = --λ-

. 2 2 ( λ2 + 1 )2 ( λ2 - 1 )2 Εχουμε: χ - y = -λ - --

λ λ4 + 2λ2 + 1 - λ4 + 2λ2 - 1 = 2 = 4 .

λ Το σημείο τομής επαληθεύει την εξίσωση χ 2 - y2 = 4 , η οποία παριστάνει ισοσκελή υ-περβολή .

β) β2 = γ2 - α2 � γ2 = α2 + β2 � . � γ2 = 4 + 4 � γ = 2.J2 Άρα, E ' {-2.J2,o) και E (2.J2,o) .

ΕΥΚΛΕΙΔΗΣ Β ' λθ ' τ3/46

Page 49: Ευκλειδης Β 59

Μαθηματικά Β ' Λυκείου

γ) Έστω ότι υπάρχει σημείο Μ, τότε {ΜΕ) - (ΜΕ

' ) = 8 = 2α , άρα το Μ ανήκει στον

ένα κλάδο της C. Το Μ θα είναι το σημείο το­μής της C με την ευθεία y = -χ . Η y = -χ είναι ασύμπτωτη της C, η οποία δεν τέμνει τη C, άρα, δεν υπάρχει τέτοιο σημείο.

Θ Ε ΜΑ 8 ' ' χ2 y2 Εστω η υπερβολη C : 2 - --τz = 1 , λ, α > Ο με

α λ α εκκεντρότητα ε και η παραβολή C ' : y2 = 2λχ . α) Να βρείτε τις εξισώσεις των ασυμπτώτων της C. β) Να αποδείξετε ότι λ = �ε2

- 1 .

γ) Α ν η C είναι ισοσκελής να δείξετε ότι οι C και C ' τέμνονται σε δύο σημεία Α, Β.

δ) Α ν το μέσο Μ του ΑΒ έχει συντεταγμένες ( 3, Ο) να υπολογίσετε την εξίσωση της C.

ΛΥΣΗ

α) Οι ασύμπτωτες θα έχουν εξισώσεις, λα y = ±-x <::::> y = ±λx . α

γ �α2 + β2 �α2 + λ2α2 β) ε = - = = =

α α α

= α� = �λ2 + Ι α

ε = �λ2 + Ι =? ε2 = λ2 + Ι =? =? λ2 = ε2 - Ι =? λ = �ε2 - Ι .

γ ) Αν η C είναι ισοσκελής α = β =? α = λα =? λ = Ι . Θεωρούμε το σύστημα:

� -i_ = 1} χ2 - 2χ - α2 = 0 ( 1)} α2 α2 <::::> 2 2 y2 = 2χ Υ = χ

τότε,

Η ( 1 ) έχει Δ = 4 + 4α2 = 4 (α2 + 1) > 0 , άρα θα έχει δύο λύσεις Χ ι , χ2 ετερόσημες αφού Χ ι χ2 = 1 = -α2 < Ο . Δεκτή είναι μόνο η θετική.

α Τελικά το σύστημα έχει δύο λύσεις οπότε οι C, C ' τέμνονται σε δύο σημεία Α, Β.

δ) Αν Χι > 0 τότε Α (χι ,�) , Β (χι , -2Fι) , χ + χ άρα, ι ι = 3 <::::> Χ ι = 3 . 2

Από ( 1 ) : α2 = χ2 - 2χ = 32 - 2 · 3 = 3 , άρα, α = .J3 .

' χ2 y2 Τελικα, C :--- = I . 3 3

ΔΥΟ ΣΗ ΜΑΝΤΙΚΕΣ ΠΑΡ ΑΛΕΙΨΕΙΣ ΤΟΥ ΤΕΥΧΟΥΣ 58 . . . μt: την αμέριστη βοήθεια του «Δαίμονα του Τυπογραφείου»

I . Δεν αναφέρθηκε ότι το μεγαλύτερο μέρος της ύλης του περιοδικού γράφτηκε από τους συναδέλφους του παραρτήματος της Ε.Μ.Ε. Λαμίας, ύστερα από πρόταση που τους έκανε η Συντακτική Επιτρο­πή. Από τη θέση αυτή τους ευχαριστούμε. Αξίζει να σημειωθεί ότι το παράρτημα της Ε.Μ.Ε. Λαμίας οργάνωσε με επιτυχία το τελευταίο συνέδριο της Ε.Μ.Ε.

2. Οι συγγραφείς του άρθρου της Άλγεβρας της Α ' Λυκείου είναι οι: Γιάννης Γιαννόπουλος, Σπύρος Παναγιωτόπουλος και Σπύρος Τζινιέρης και όχι μόνο ο Σπύρος Τζινιέρης όπως γράφτηκε.

ΕΛΛΗΝΙΚΉ ΜΑΘΗΜΑΤΙΚΉ ΕΤ ΑΙΡΕΙΑ

• Ημερίδα 2 Μαϊου 2006 στο Πανεπιστήμιο Αθήνας με θέμα την επιστημονική και παιδαγωγική κατάρτιση των πτυχιούχων εκπαιδευτικών, οι οποίοι θέλουν να προσληφθούν στη δημόσια εκπαί­δευση και αλλού μέσα από τους διαγωνισμούς του ΑΣΕΠ.

ΕΥΚΛΕΙΔΗΣ Β' λθ ' τ3/47

Page 50: Ευκλειδης Β 59

LΧ!aΙιιιι•••• "� l'fl' r ��lι ,_, Α•eΙ-ιι

ΑΣΚΗΣΕΙΣ ΑΝΑΛ ΥΣΗΣ

ι . Θεωρούμε ένα μη μηδενικό πολυώνυμο Ρ( χ) με πραγματικούς συντελεστές και έναν αριθμό ρ Ε IR . Να δείξετε ότι το Ρ( χ)

έχει παράγοντα το {χ - ρ )2 αν, και μόνο αν:

Ρ{ρ) = Ο και p' (ρ) = Ο .

Στη συνέχεια να δείξετε ότι το πολυώνυμο: Χ Χ2 Χν

f {x) = 1 + - + - + ••• + - , 1 ! 2 ! ν !

όπου ν Ε Ν* , δεν έχει διπλή πραγματική ρίζα. ΛΥΣΗ

1) i) Έστω ότι το Ρ( χ) έχει παράγοντα το (χ - ρ )2 • Τότε, υπάρχει πολυώνυμο Π(χ) με:

Ρ (χ ) = ( χ - ρ)2 · Π (χ ) ( 1 ) Από την ( 1 ) με χ = ρ βρίσκόυμε Ρ (ρ) = Ο . Παραγωγίζοντας τα μέλη της ( 1 ), βρίσκουμε: p' ( χ ) = 2 ( χ - ρ ) Π (χ ) + (χ - ρ )2 π' ( χ ) .

Απ' αυτή με χ = ρ βρίσκουμε p' (ρ) = Ο . ii) Αντιστρόφως. Έστω ότι Ρ (ρ) = Ο και

p' (ρ) = Ο . Ονομάζουμε Π (χ ) και αχ + β

το πηλίκο και το υπόλοιπο αντιστοίχως, της διαίρεσης Ρ( χ ) : ( χ - ρ )2 , οπότε:

Ρ (χ ) = (χ - ρ)2 Π (χ ) + αχ + β . (2) Από την (2) με χ = ρ και επειδή Ρ(ρ) = Ο , βρίσκουμε ότι αρ + β = Ο . Παραγωγίζοντας τα μέλη της (2), βρίσκουμε: p' ( χ ) = 2 (χ - ρ )Π ( χ ) + (χ - ρ )2 π' ( χ ) + α .

Απ' αυτή με χ = ρ και επειδή p' (ρ) = Ο ,

βρίσκουμε ότι α = Ο . Και επειδή αρ + β = Ο ,

της Χρυσταλλένης Κυβερνήτου

έπεται ότι β = Ο . Αντικαθιστώντας στη (2), έχουμε: Ρ (χ ) = (χ - ρ)2 Π (χ ) . Άρα, το

Ρ{ χ) έχει παράγοντα το (χ - ρ )2 . 2) Έστω ότι το f (χ ) έχει μία διπλή πραγματική

ρίζα ρ. Τότε, το f (χ ) θα έχει παράγοντα το (χ - ρ )2 , οπότε ν � 2 . Εξάλλου, τότε σύμφωνα με τα προηγούμενα, θα έχουμε f (ρ) = Ο και ( (ρ) = Ο . Έχουμε:

, Χ Χ 2 Χ ν-1 f (χ ) = 1 + - +-+ . . . +

( ) �

1 ! 2 ! ν - 1 ! Χ ν

� r ' (x ) = f (x ) -- . (3) ν ! Από την (3) με χ = ρ και επειδή f (ρ) = Ο και

ν ( (ρ) = Ο , έπεται ότι Ε_ = Ο και συνεπώς

ν ! ρ = Ο . Έτσι, θα έχουμε: f {O) = O και επειδή f (Ο) = 1 , θα έχουμε Ο = 1 , άτοπο.

2. Μία συνάρτηση f είναι ορισμένη στο IR , παραγωγίσιμη στο Ο και για κάθε x, y Ε IR , ισχύει:

f {x + y) = f {x)f {y) - ημxημy . (1)

Να δείξετε ότι η f είναι παραγωγίσιμη στο IR •

ΛΥΣΗ

Από την ( 1 ) με χ = y = Ο , βρίσκουμε ότι: f {O) = f 2 (Ο) � f (O) [f (0) - 1] = Ο �

� [f (O) = O ή f (O) = l] . - Έστω ότι f (Ο) = Ο . Από την ( 1 ) με χ Ε IR και

y = Ο , βρίσκουμε ότι: f (χ ) = f (χ ) · f (Ο) - ημχημΟ � f (χ ) = Ο .

ΕΥΚΛΕΙΔΗΣ Β ' λθ ' τ.3/48

Page 51: Ευκλειδης Β 59

Μαθηματικά για την Γ Λυκείου

Άρα, τότε: f ( χ ) = Ο , για κάθε χ Ε IR. . Έτσι, τό­τε, από την ( 1 ) θα έχουμε για κάθε χ, y Ε IR. : Ο = Ο · Ο - ημχημy => ημχημy = Ο , άτοπο.

- Άρα f (Ο) = 1 . Επειδή η f είναι παραγωγίσιμη

στο Ο, έχουμε: f ( x ) - f (O) . lim = f (O) E !R. => χ �Ο Χ - 0

f (x ) - 1 , => lim = f (Ο) (2)

χ �Ο Χ Έστω ένας αριθμός χ0 Ε 1R. . Έχουμε:

' ( ) . f ( x0 + h ) - f ( x0 ) ( I ) f χ0 = lιm =

h�O h . f ( x0 ) f (h ) - ημx0ημh - f ( x0 ) = hm =

h�O h

= lim [f (xo ) f (h ) - 1_ ημxo

ημh ] = h�O h h

= f (χ0 ) r ' (Ο) - ημχ0 ,

λόγω της (2) και επειδή lim ημh = Ι . h �O h

Άρα, η f είναι παραγωγίσιμη στο IR. και για �άθε χ Ε IR. , ισχύει: f ' ( χ ) = f ' (Ο) · f ( χ ) - η μχ .

3. Να δείξετε ότι για κάθε ν ε Ν* και για κά­θε χ > Ο , ισχύει:

ν 1 χ - ln x � - . (1) ν

ΛΥΣΗ

Θεωρούμε έναν αριθμό ν Ε Ν* και τη συνάρτηση : f ( Χ ) = Χ ν - Ι η χ .

Η f είναι ορισμένη και παραγωγίσιμη στο διάστη­μα (Ο, +οο) με:

( ( ) v-1 I Χ = νΧ -- = χ

χ ο

f'

νχ ν - 1 χ

-f �

> 0, αν Χ >� = 0, αν Χ =� < 0, αν χ <�

να +ω

6 + ί

εGχ /

Σ , , f , ελά .fl , υμπεραινουμε οτι η εχει χιστο στο χ= ν� , ισο με:

1 1 1 1 1 1 = - -- ln - = - -- ( - ln ν) = -( Ι + ln ν) = ν ν ν ν , ν ν

=.!.Ιη (eν) �.!. , γιατί eν � e και άρα ln (eν) � 1 . ν ν

Έτσι, για κάθε χ > Ο , έχουμε:

r (ψ r ( �) + c> f (ψ �ο> (Ι)

4. Να λυθεί η εξίσωση: 3χ + sx = 2χ + 6χ . (1)

ΛΥΣΗ

Έστω ότι ένας αριθμός ρ Ε IR. είναι ρίζα της εξί-σωσης ( 1 ), οπότε:

3ρ + 5ρ = 2ρ + 6ρ => 3ρ - 2ρ = 6ρ - 5ρ =>

3ρ - 2ρ 6ρ - 5ρ => (2) . 3 - 2 6 - 5

Θεωρούμε τη συνάρτηση : f ( χ ) = χ ρ , χ Ε (Ο, +οο) .

Η f είναι παραγωγίσιμη στο (Ο, +οο) με: ( ( Χ ) = ρ χ ρ-ι _

Είναι φανερό ότι η f πληροί τις υποθέσεις του θε­ωρήματος μέσης τιμής σε καθένα από τα διαστή­ματα [2, 3] και [5, 6] . Άρα, υπάρχουν αριθμοί ξ1 και ξ2 με: 2 < ξ1 < 3 < 5 < ξ2 < 6 και !( (ξ, ) = f (3�=�(2) {ρξj-1 = f (3) - f ( 2) (2) ( (ξ, ) = f ( 6l=:(5)

ο> ρξ\-' = f (6) - f {5) ο>

=> ρξΓ' = ρξ�-ι => ρ ( ξf-' _ ξ�-ι ) = 0 =>

=> (ρ = ο ή ξj-1 = ξ�-1 ) =>

=> [ρ = Ο ή (ρ - l ) Ιη ξ1 = (ρ - 1) 1η ξ2 ] =>

=> (ρ = Ο ή ρ = 1) . Άρα, τότε: ρ = Ο ή ρ = 1 .

Α ντιστρόφως Όπως βρίσκουμε εύκολα οι αριθμοί Ο και 1 επαληθεύουν την εξίσωση ( 1 ) και άρα αυ­τοί είναι οι μοναδικές ρίζες της εξίσωσης αυτής.

5. Να βρείτε τα διαστήματα μονοτονίας και τα ακρότατα της συνάρτησης:

ΕΥΚΛΕΙΔΗΣ Β ' λθ ' τ.3/49

Page 52: Ευκλειδης Β 59

Μαθηματικά για την Γ Λυκείου

ι f (χ) = χ� ' χ Ε (Ο, +α:>) •

Μετά, να δείξετε ότι, για κάθε ν Ε Ν* , ι­σχύει: � � � .

ΛΥΣΗ

α) Για κάθε χ > Ο , έχουμε: � _!_ In χ , _!_ In χ ( ln χ ) ' f {x ) = eln x = e x :::::> f (x ) = e x · --;- :::::>

ι {> 0, . - 1 - ln x f (χ ) = χ χ • χ 2 = Ο,

< 0,

χ ο 1 2 e

f ' + b f � Λ

μεγ.

αν αν αν

3

-

x < e x = e x > e

4 +οο

Συμπεραίνουμε ότι: fλ (Ο, e] και Π [ e, +οο) . Επίσης ότι η f στο e έχει μέγιστο, ίσο με

ι f ( e) = e� .

I β) Με ν = 1, 2, 3, . . . , έχουμε: f (ν) = ν; = r.Γv . Λό-

6.

γω των παραπάνω, έχουμε: f (1) < f (2) ( 1 )

και f (3) > f (4) > f (5) > . . . (2) Εξάλλου, έχουμε: f (3) - f (2) = � - J2 = � - � > 0 :::::>

:::::> f ( 3) > f ( 2) (3) Από τις ( 1 ), (2) και (3) συμπεραίνουμε ότι, για

' θ �τ* , κα ε ν ε 1'1 , ισχuει: f (ν) � f (3) :::::> r.Γv � � -

Nα δείξετε ότι, με χ > Ο , y > Ο και χ + y = ι , ισχύει:

χ Υ ι χ · Υ � 2 (ι) ΛΥΣΗ

Έχουμε: y =1 - χ και επειδή y > Ο , έπεται ότι 1 - χ > Ο , οπότε χ < 1 . Άρα Ο < χ < 1 . Έτσι, για να δείξουμε την ( 1 ) αρκεί να δείξουμε ότι, για κάθε

(ο 1) ' χ (1 )ι-χ 1 ' χ ε , ισχuει: χ · - χ � - , αρκει: 2

1 χ ln χ + (1 - χ ) ln ( 1 - χ ) � Ι η-2 Θεωρούμε τη συνάρτηση :

f (χ ) = χ ln χ + ( 1 - χ ) ln ( 1 - χ) , χ ε (Ο, 1) . Στο διάστημα (Ο, 1) έχουμε:

f · (χ ) = Ι η χ + 1 - ln ( 1 - χ) - 1 = > 0, αν χ

= I η χ - ln (1 - χ ) = 0, αν χ

< 0, αν χ

χ ο 1 Ι 2

f ' - b +

f � !--ελαχ �

1 > -2 Ι = 2 1 < -2

(2)

Σ ' ' f ' ελά Ι ' υμπεραινουμε οτι η εχει χιστο στο "2 , ισο με:

f (.!.) = .!. ιη.!. + .!. ιη.!. = In.!. . 2 2 2 2 2 2 Συνεπώς, για κάθε χ ε (Ο, 1) , έχουμε:

f ( x ) � r (�)=> (2) .

7. Να βρείτε τις παραγωγίσιμες συναρτήσεις f : JR � JR και g : {Ο, +οο) � JR , για τις ο-ποίες για κάθε χ > Ο , ισχύουν: r (g (x)) = x (ι) και r ' (g (x)) = x (2).

ΛΥΣ Η

Έστω ότι δύο παραγωγίσιμες συναρτήσεις f : JR � JR και g : (Ο, +οο) � JR πληρούν τις ισότη-τες ( 1 ) και (2) . Από την ( 1 ) έχουμε για κάθε χ > Ο :

' ( ) ' (2) ' ' 1 f g ( χ ) · g (χ ) = 1:::::> xg (χ ) = 1 <=> g (χ ) = - :::::> χ :::::> g ' ( x ) = ( ln x )' :::::> g (x ) = ln x + c (c ε JR) . Έτσι, από την ( 1 ) έχουμε για κάθε χ > Ο :

f ( In x + c) = x . (3)

Για κάθε χ ε JR έχουμε ex-c > Ο . Θέτοντας στην (3) όπου χ το ex-c έχουμε, για κάθε χ ε JR :

r (ιn ex-c + c) = ex-c :::::> f (x - c + c) = ex-c :::::> :::::> f (χ ) = ex-c

ΕΥΚΛΕΙΔΗΣ Β' λθ ' τ.3/50

Page 53: Ευκλειδης Β 59

Μαθηματικά για την Γ Λυκείου

Βρίσκουμε εύκολα ότι οι συναρτήσεις f (χ ) = e χ -c και g (χ ) = Ι η χ + c , όπου c Ε IR , πληρούν τις δοσμέ­

νες συνθήκες και άρα αυτές είναι οι ζητούμενες.

8 . Να βρείτε τις συναρτήσεις f, οι οποίες είναι ορισμένες και δύο φορές παραγωγίσιμες στο IR και για τις οποίες ισχύουν: f {0) = 2 , r' (o) = O και r" (x) = f {x) ,

για κάθε χ Ε JR •

ΛΥΣ Η

Έστω ότι μία συνάρτηση f πληροί τις δοσμένες συνθήκες. Έτσι, έχουμε για κάθε χ Ε IR :

( (x ) = f ( x ) � ( (χ ) + { (χ ) = { ( x ) + f (x ) �

(r'( x ) + f ( x ))

' = r

'( x ) + f ( x ) . ( 1 )

Η συνάρτηση g (χ ) = { (χ ) + f (χ ) είναι ορισμένη και παραγωγίσιμη στο IR και λόγω της ( 1 ), έχουμε για κάθε χ Ε IR :

g'( x ) = g (x ) � g

'( x ) e-x - g (x ) e-x = 0�

g' (χ ) e-x + g ( χ) ( e-x )

'= ο � (g ( χ ) e-x )

' = 0 �

g (x ) e-x = c(c E IR) � g (x ) = c · ex �

{ ( χ ) + f (χ ) = c · ex . (2) Από την (2) με χ = Ο βρίσκουμε:

r'(O) + f (O) = c � 0 + 2 = c � c = 2 .

Έτσι, από τη (2), έχουμε για κάθε χ Ε IR : f'(x) + f ( x) = 2ex �f

'(x)ex + f (x) (ex )

' = 2e2x �

( f (χ )ex ) ' = ( e2x )' � f (χ ) ex = e2x + c1 ( c1 Ε IR) (3) .

Από την (3) με χ = Ο βρίσκουμε: f (Ο) = 1 + c1 � 2 = 1 + c1 � c1 = 1 .

Έτσι, από την (3), έχουμε για κάθε χ Ε IR : f ( x ) ex = e2x + 1 � f (x ) = ex + e-x (4)

Όπως βρίσκουμε εύκολα, η συνάρτηση (4) πληροί τις δοσμένες συνθήκες και άρα είναι η μοναδική ζητούμενη. 9. Ν α βρείτε τους αριθμούς χ Ε {Ο, +οο) και

y Ε IR , για τους οποίους ισχύει:

2χ..Γχ + 1 = 3χσυνy . (1)

ΛΥΣ Η

Έστω ότι για δύο αριθμούς χ Ε (Ο, +οο) και y Ε IR ισχύει η ισότητα ( 1 ), οπότε θα έχουμε:

2.Jx +.!. = 3συνy . (2) χ

Θεωρούμε τη συνάρτηση :

f {x ) = 2.Jx +.!_ , χ Ε (Ο,+οο) . χ

Η f είναι παραγωγίσιμη στο (Ο, +οο) με: . 1 1 1 1 ..Γχ 1 f (χ ) = 2 ·

2..Γχ -� = ..Γχ -� = -;:-� =

π {> 0, ν χ- - 1 - - ο - 2 - '

χ < 0,

αν αν αν

χ > 1

χ = 1

χ < 1

Συμπεραίνουμε ότι η f έχει ελάχιστη τιμή, την ο­ποία λαμβάνει μόνο για χ = 1 , ίση με f ( 1 ) = 3 .

χ ο 1 +οο

f' - b +

f � f(ί)=3 � ελαχ

Έτσι, έχουμε: f (χ ) � 3 , για κάθε χ Ε (Ο, +οο) , με το = μόνο αν χ = 1 . Έτσι, από τη (2), έχουμε:

3συνy = 2 .Jx +.!. � 3 � 3συνy � 3 � συνy � 1 .

χ Και επειδή συνy � 1 , έχουμε συνy = 1 και άρα:

y = 2κπ , κ Ε Ζ . Επειδή συνy = 1 , από τη (2), έχουμε:

2.Jx +.!. = 3 � f (x ) = 3 � χ = 1 . χ

Άρα, �ότε: (χ, y ) = ( 1, 2κπ) , Κ Ε Ζ . (3) Α ντιστρόφως. Όπως βρίσκουμε εύκολα τα ζεύγη (3) επαληθεύουν την ( 1 ) .

Άρα, τα ζεύγη (3) είναι τα ζητούμενα.

I Ο. Μία συνάρτηση f είναι ορισμένη και δύο

φορές παραγωγίσιjιη στο IR και για κάθε Χ Ε R ισχύει: f2 {3x + 1) + 4 � 4f (2x2 + χ + 1) . (1)

ΕΥΚΛΕΙΔΗΣ Β ' λθ ' τ.3/51

Page 54: Ευκλειδης Β 59

Ν α δείξετε ότι: α) Η f δεν αντιστρέφεται. β) η ( δεν αντιστρέφεται. γ) Η εξίσωση: f" {χ) = Ο έχει μία τουλάχι-

στον ρίζα στο JR . ΛΥΣ Η

α) Από την ( 1 ) με χ = Ο και χ = 1 βρίσκουμε α-ντιστοίχως:

{f2 (1) + 4 � 4f (1) => {( f (1) - 2 )2 � ο

=> {f (1) = 2

f2 (4) + 4 � 4f (4) (f (4) - 2)2 � ο f (4) = 2

Έτσι, έχουμε f ( 1) = f ( 4) και επειδή 1 :;e 4 , έ-πεται ότι η f δεν είναι 1 - 1 και συνεπώς δεν α­ντιστρέφεται.

β) Θεωρούμε τη συνάρτηση : g ( χ ) = f 2 (3χ + 1 ) + 4 - 4f (2x2 + χ + 1 ) .

Η g είναι ορισμένη και παραγωγίσιμη στο JR με: g. ( χ ) = 2f {3x + 1) · r ' (3χ + 1 ) · (3χ + 1)' -

-4( ( 2χ 2 + χ + 1) . ( 2χ 2 + χ + 1 ) ' => g' ( χ ) = 6f (3x + 1) · r ' (3χ + 1) -

-4( 4χ + 1) . ( ( 2χ2 + χ + 1) .

Εξάλλου, έχουμε: g (Ο) = f 2 ( 1) + 4 - 4f ( 1) = Ο και g (1) = f2 (4) + 4 - 4f (4) = 0 , αφού f (1) = 2 και f ( 4) = 2 . Έτσι, λόγω της ( 1 ), έχουμε για κάθε χ ε JR : g ( χ ) � g (Ο) και g (χ ) � g ( 1) .

Άρα, σύμφωνα με το θεώρημα του Fennat, έχουμε: {g' (o) = o {6f {1) · r ' ( 1 ) - 4r ' ( 1 ) = 0 g ' ( 1) = ο

=> 6f ( 4) . ( ( 4) - 20( ( 4) = ο

=>

{{ (1) = 0 => .

( ( 4) = ο

Έτσι, έχουμε r ' ( 1 ) = r ' (4) και επειδή 1 :;e 4 , έπεται ότι η f ' δεν είναι 1 - 1 και συνεπώς δεν αντιστρέφεται.

γ) Η συνάρτηση ( πληροί τις υποθέσεις του θεω­ρήματος του Rolle στο διάστημα [1 , 4] . Άρα, η εξίσωση : ( ( χ ) = Ο έχει μία τουλάχιστον ρίζα στο διάστημα (1 , 4) και επομένως στο JR .

Ασκήσεις στις nαpαγώγοuς Δημήτριος Ρέγκλης

Η παράγωγος γεννήθηκε μεσ ' από προβλήματα (της εφαπτομένης καμπύλης και της στιγμιαίας ταχύτητας)

και επιλύει προβλήματα θεωρητικά και πρακτικά. Εδώ παρουσιάζουμε μερικές υποδειγματικά λυμένες α­

σκήσεις για να γίνει πιο κατανοητή η σημασία της παραγώγου ως εργαλείου στην αντιμετώπιση συγκεκριμέ­

νων θεμάτων της Πραγματικής Ανάλυσης.

Ασκή σεις Διαφορικού Λογισμού

Άσκηση 1 Δίνεται παραγωγίσιμη στο JR συνάρτηση με f συνεχή πρώτη παράγωγο, η οποία στρέφει τα κοίλα άνω στο JR •

Αν η γραφική παράσταση της f έχει aσύμπτω­τες τις ευθείες ( ε1 ) : ψ = λ χ + β και

( ε2 ) : ψ = -λ χ + β στο +οο και στο -οο αντίστοι­χα, λ > Ο και υπάρχουν στο JR τα όρια lim f' (χ) , lim f' (χ) aποδείξτε ότι:

Χ-Η·ΟΟ X�-«J

α) Η f παρουσιάζει ακριβώς ένα ακρότατο το είδος του οποίου να προσδιορίσετε.

β) f (χ) > λχ + β και f (χ) > -λχ + β για κάθε χ ε JR .

ΕΥΚΛΕΙΔΗΣ Β ' λθ ' τ.3/52

Page 55: Ευκλειδης Β 59

Μαθηματικά για την Γ Λυκείου

Λύση

α) Θεωρούμε την συνάρτηση g ( χ ) = f ( χ ) - λχ - β , x E IR .

Είναι lim g ( χ ) = Ο αφού η Χ -Η«>

ευθεία ψ = λχ + β είναι ασύμπτωτη στο +οο της γραφι­κής παράστασης της f . Οπότε:

lim f ( χ ) = lim ( g (χ ) + λ + �) = λ . Χ-Η«> Χ Χ-Η«> Χ Χ

Και επειδή lim f (x ) = lim (g ( χ ) + λχ+β) =

χ �+«> χ -++«>

= lim g ( χ ) + lim (λχ+β) = +οο Χ -+00 Χ -++«>

εφαρμόζοντας τον κανόνα De L' Hospital έ­

χουμε lim f ( χ ) = lim f 1 ( χ ) . χ -++<Χ> χ χ -++οο

Συνεπώς lim f 1 ( χ ) = λ > Ο . Χ -ΗΟΟ

Άρα (όριο και διάταξη) υπάρχει ξι σε περιοχή του +οο ώστε f 1 (ξ ι ) > Ο . Με όμοιο τρόπο θεωρώντας την h ( χ ) = f ( χ ) + λχ - β, χ Ε IR αποδεικνύεται ότι υπάρχει ξ2 σε περιοχή του -οο ώστε f 1 ( ξ2 ) < Ο . Και επειδή η f 1 είναι συνεχής στο [ ξι _ξ2 J με f 1 (ξ ι ) f 1 (ξ2 ) < 0 σύμφωνα με το θεώρημα Bol­zano υπάρχει ξ Ε (ξ ι , ξ2 ) τέτοιος ώστε f 1 (ξ) = Ο . Όμως η f 1 είναι γνησίως αύξουσα στο IR (αφού η f είναι κυρτή στο IR ), άρα το ξ είναι μοναδικό στο IR . Επιπλέον: για χ<ξ είναι f 1 ( x ) < f 1 (ξ) <=:> f 1 ( x ) < O , για χ>ξ είναι f 1 ( χ ) > f 1 (ξ) <:::::> f 1 ( χ ) > Ο , που

αύξουσα (αποδεικνύεται εύκολα) και επιπλέον lim g 1 ( x ) = lim (f 1 ( χ ) - λ) = Ο .

Χ -++«> Χ-+-tα:ι

Άρα το σύνολο τιμών της g1 θα είναι {}!_�,g � ( x ) , o) , δηλαδή g l ( x ) < O για κάθε

χ Ε IR που σημαίνει ότι η g είναι γνησίως φθί­νουσα στο IR και επομένως το σύνολο τιμών της g θα είναι

{Δ� g ( χ ) , }i� g ( χ )) δηλαδή {Ο, }i�g ( χ )) . . Άρα g ( χ ) > ο <:::::> f ( χ ) - λχ - β > ο <:::::> f ( χ ) > λχ + β ' για κάθε χ Ε IR . Με όμοιο τρόπο θεωρώντας την h ( χ ) = f ( χ ) + λχ - β χ Ε IR αποδεικνύεται η δεύτερη ανισότητα.

Άσκηση 2 Δίνεται η συνάρτηση f με

f (χ) = 2α2χ3 - 3 (α2 + l)x2 + 6χ - 6 .

Να βρείτε τις τιμές του α Ε IR* ώστε η f να πα­ρουσίαζει στο χ0 = 1 τοπικό μέγιστο.

Λύση

Η f είναι παραγωγίσιμη στο IR με f 1 ( χ ) = 6α2χ 2 - 6 ( α2 + Ι ) χ+6=6 ( α2χ - Ι ) ( χ - Ι) .

Είναι f 1 ( l ) = Ο για όλα τα α Ε IR * . Έτσι η ανα­γκαία σύμφωνα με το θεώρημα το Fermat συνθήκη f 1 ( 1 ) = Ο , για να είναι το χ = I τοπικό ακρότατο, δίνει ταυτότητα ως προς α . Οι θέσεις πιθανών α-

, f ' 1 ' 1 κροτατων για την ει ναι χ= η χ = -2 • α

Διακρίνουμε τις παρακάτω περιπτώσεις για το σημαίνει ότι η f παρουσιάζει στο ξ τοπικό α Ε JR * . ελάχιστο, που είναι προφανώς ολικό. • α2 = 1 <:::::> α = 1 ή α = - 1 . Τότε:

χ ξ ....ω +οο f' (χ) - b +

f(x) � ε�χ- � β) Είναι g 1 ( x ) = f 1 ( x ) - λ , x E IR . Επειδή η f 1 εί­

ναι γνησίως αύξουσα θα είναι και gl γνησίως

f 1 ( χ ) = 6( χ - 1 γ η οποία διατηρεί θετικό πρόσημο εκατέρωθεν του χ = 1 και συνεπώς η f δεν παρουσίαζει ακρότατο στο χ = 1

• α2 < 1 <=:> α Ε (-1, 0) υ (Ο, 1) . Τότε: Το πρόσημο της f 1 δίνεται στον παρακάτω πίνακα μεταβολών:

ΕΥΚΛΕΙΔΗΣ Β ' λθ ' τ.3/53

Page 56: Ευκλειδης Β 59

Μαθηματικά για την Γ Λυκείου

Ι χ Ι --00 2 α +οο

f' (x) + - +

f(x) � τ μ. � τ ε. � απ' όπου και προκύπτει ότι η f παρουσιάζει το­πικό μέγιστο για χ = 1 .

• α2 > 1 <=:> α Ε (--οο, -1)υ ( 1, -tοο) .Τότε Το πρόσημο της f 1 δίνεται στον παρακάτω πίνακα μεταβολών

Ι χ 2 Ι --00 α +οο

f' (x) + - +

f(x) � τ μ. � τ ε. � απ' όπου προκύπτει ότι η f παρουσιάζει στο χ = 1 τοπικό ελάχιστο. Επομένως η f παρουσιάζει στο χ = 1 τοπικό μέγιστο, όταν α Ε ( -1, 0) υ (Ο, 1)

Άσκηση 3

Να μελετήσετε ως προς την μονοτονία την συ­νάρτηση f (χ) = χ - πlnx και να δείξετε ότι eeΠπ > eΖπ

Λ\)ση

Η f είναι παραγωγίσιμη στο (Ο, +οο) με f 1 (x ) = 1 - π .

χ

Είναι: f 1 (χ ) > Ο <=:> 1 - � > Ο <=:> χ > π και χ

f 1 (x ) = O<=:> χ = π Επομένως η f είναι γνησίως φθίνουσα στο (Ο, π] αφού είναι συνεχής σ αυτό και f 1 (χ ) < Ο στο εσωτερικό του. Η f είναι γνησίως αύξουσα στο [π, +οο) αφού είναι συνεχής σ' αυτό και f 1 (χ ) > Ο στο εσωτερικό του. Έχουμε e, π Ε (Ο, π] στο οποίο η f είναι γνησίως φθίνουσα και επειδή είναι e<π έχουμε:

f ( e) > f (π)<=:> e - π > π - πlηπ <=:> <=:> lne• + ln ππ > 2π <=:> <=:> ln ( e•ππ ) > ln e2π <=:> <=:> e•ππ > e2π

Άσκηση 4

α) Αν η συνάρτηση f είναι δύο φορές παραγω­γίσιμη στο JR τότε να αποδείξετε ότι δεν υ­πάρχει ξ Ε JR , στο οποίο η f να παρουσιάζει ταυτόχρονα τοπικό ακρότατο και σημείο κα­μπής.

β) Να αποδείξετε ότι η συνάρτηση z

f (χ) = ex - � - χ - 2006 δεν παρουσιάζει το-2

πικό ακρότατο στο χ = Ο •

Λύση

α) Με άτοπο: Υποθέτουμε ότι υπάρχει ξ Ε JR στο οποίο η f παρουσιάζει ταυτόχρονα τοπικό ακρότατο και σημείο καμπής. Τότε θα ισχύει f 1 (ξ ) = f " (ξ) = Ο και λόγω του σημείου καμπής η γραφική παράσταση της f αλλάζει κυρτότητα εκατέρωθεν του ξ . Ας υποθέσουμε (χωρίς βλάβη της γενικότητας) ότι η f είναι κυρτή σε διάστημα (ξ - δ, ξ) και κοίλη σε διάστημα (ξ, ξ + δ) , δ > Ο . . Τότε εξ ορισμού η f 1 θα είναι γνησίως αύξουσα στο [ξ - δ, ξ ] και γνησίως φθίνουσα στο [ξ, ξ + δ] . Άρα Για χ Ε {ξ-δ, ξ) είναι

χ<ξ <=:> f 1 ( χ ) < f 1 (ξ) <=:> f 1 ( χ ) < ο

Για χ Ε (ξ, δ+ξ) είναι χ>ξ <=:> f 1 (x ) < f 1 (ξ) <=:> f1 ( χ ) < Ο

Δηλαδή η f 1 ( χ ) διατηρεί σταθερό (αρνητικό) πρόσημο εκατέρωθεν του ξ και επομένως δεν παρουσιάζει ακρότατο στο ξ το οποίο έρχεται σε αντίθεση με την αρχική υπόθεση .

β) Η f είναι δύο φορές παραγωγίσιμη στο JR με f 1 (x ) = e' - χ - 1 και f " (x ) = e' - 1 . Είναι f " (x ) > O <=:> χ > 0 και f"(x) < O<=:>x < O . Άρα η f παρουσιάζει σημείο καμπής για χ = Ο οπότε σύμφωνα με το πρώτο ερώτημα της άσκησης δεν παρουσιάζει τοπικό ακρότατο στο χ = Ο , αν και ισχύει η συνθήκη f 1 (0) = 0 . (Δεν ισχύει το αντίστροφο του θεωρήματος Fermat) .

ΕΥΚΛΕΙΔΗΣ Β ' λθ ' τ.2/54

Page 57: Ευκλειδης Β 59

Άσκηση 5 Δίνεται παραγωγίσιμη στο IR συνάρτηση f με συνεχή πρώτη παράγωγο, η οποία στρέφει τα κοίλα άνω στο IR και υπάρχει ξ Ε IR τέτοιο ώστε f ' (ξ) = λ . α) Αν f (ξ) > λξ + β , λ, β Ε IR aποδείξτε ότι η

γραφική παράσταση της f δεν έχει με την ευθεία (ε) : ψ = λχ + β κοινά σημεία.

β) Αν f (ξ) < λξ + β , λ, β Ε IR aποδείξτε ότι η γραφική παράσταση της f και η ευθεία (ε) : ψ = λχ + β έχουν δύο τουλάχιστον κοινά ση­μεία.

γ) Αποδείξτε ότι e• > ex --1- για κάθε χ Ε IR •

2006 Λύση

Θεωρούμε την συνάρτηση g(χ) = f ( χ) -λχ -β , x E IR .

Είναι g ' ( x ) = f ' ( x ) - λ για κάθε x E IR , g' (ξ) = Ο και g' γνησίως αύξουσα στο IR αφού f ' γνησίως αύξουσα στο IR ( f στρέφει τα κοίλα άνω στο IR ) . Επομένως: Για χ<ξ είναι g ' ( χ ) < g ' (ξ) � g ' ( χ ) < Ο ( 1 ) . Για χ>ξ είναι g ' ( x ) > g ' (ξ ) � g ' ( x ) > O (2). Άρα η συνάρτηση g παρουσιάζει στο ξ ολικό ε­λάχιστο και επομένως ισχύει g ( χ ) � g (ξ) για κάθε χ E IR . α) Επειδή g (ξ) = f (ξ) - λξ - β > Ο θα είναι

g ( χ ) > Ο � f (χ ) > λχ+β για κάθε X E IR και έτσι η γραφική παράσταση της f και η ευθεία ψ = λχ+β δεν έχουν κοινά σημεία.

β) Η συνάρτηση g ( t ) = f ( t ) - λt - β ικανοποιεί τις προϋποθέσεις του Θεωρήματος Μέσης τιμής σε καθένα από τα διαστήματα [ χ,ξ] και [ξ,χ ] για κάθε χ -:;:. ξ . Επομένως υπάρχουν ξ2 Ε ( ξ,χ ) τέτοια ώστε

ξ, Ε (χ,ξ)

g( χ ) = g ' (ξ , ) ( χ - ξ) + g (ξ) και g ( χ ) = g' ( ξ2 ) ( χ - ξ) + g (ξ)

και

επειδή λόγω των ( 1 ) , (2) είναι g' (ξ , ) < Ο ,

g ' (ξ2 ) > Ο θα είναι: Δ�g(χ ) = }i�

[g ' (ξ, ) ( χ - ξ) + g (ξ)] = +οο = = lim [g ' (ξ2 ) ( x - ξ) + g (ξ)] = lim g (x )

x � +co χ -Η-οο

Επομένως υπάρχουν γ , δ σε περιοχή του -οο

και +οο αντίστοιχα με g (γ) > Ο και g (δ) > Ο και επιπλέον g (ξ) = f (ξ) - λξ - β < Ο . Έτσι έχουμε g ( γ )g (ξ) < Ο και g (ξ)g (δ) < Ο και εφαρμόζο­ντας το θεώρημα Bolzano στα διαστήματα (γ,ξ] και [ξ,δ] για την συνεχή συνάρτηση g παίρνουμε ότι η εξίσωση g (χ ) = Ο � f ( χ ) = λχ+β έχει δύο τουλάχιστον ρίζες στο !R μία στο ( γ, ξ) και μία στο (ξ, δ) . Άρα η γραφική παράσταση της f και η ευθεία ψ = λ χ + β έχουν δύο τουλάχιστον κοινά σημεία.

γ) Εφαρμογή του α) για την συνάρτηση f ( χ ) = e' και για ξ = 1 δίνει το ζητούμενο. Πράγματι η f στρέφει τα κοίλα άνω στο !R αφού f " ( x ) = e' > Ο και f ' ( 1) = e ,

Άσκηση 6

Ι f (1) = e > e · l -- . 2006

Α ν η γραφική παράσταση μιας παραγωγίσιμης στο IR συνάρτησης f τέμνει τη γραφική παρά­σταση της g (χ) = e• σε δύο σημεία, aποδείξτε ότι υπάρχει ξ Ε IR τέτοιο ώστε f ' (ξ) = f (ξ) .

Λ\Jση

Υποθέτουμε ότι τα σημεία στα οποία η γραφική παράσταση της f τέμνει την γραφική παράσταση της g είναι Α( α, e• ) και Β (β, eβ ) με α < β . Τότε θα είναι f (α) = ea και f (β) = eβ . Για την συνάρτη-

ση h ( χ ) = f ��) με χ Ε [α, β] ισχύουν οι προϋπο­

θέσεις του Θεωρήματος Rolle αφού είναι συνεχής στο [α, β] και παραγωγίσιμη στο (α, β) ως πηλίκο των παραγωγίσιμων στο IR συναρτήσεων f ( χ ) και e' και επιπλέον

h (α) = f��) = h (β) = f��) = 1 .

Επομένως υπάρχει ξ Ε (α, β) τέτοιο ώστε να είναι ΕΥΚΛΕΙΔΗΣ Β ' λθ ' τ.3/55

Page 58: Ευκλειδης Β 59

Μαθηματικά για την Γ Λυκείου

h 1 {ξ) = Ο � f l {ξ) �f {ξ) = 0 � f 1 {ξ) = f {ξ) .

e

Άσκηση 7 Θεωρούμε την δύο φορές παραγωγίσιμη συνάρ­τηση f : [α, β] � JR. με α > Ο με συνεχή δεύτερη παράγωγο και f (α) = f (β) = Ο . Να δείξετε ότι: α) Υ π άρχει ένα τουλάχιστον ξ Ε (α, β) τέτοιο

ώστε ξf 1 (ξ) = f (ξ) . β) Α ν f" (χ) * Ο για κάθε χ Ε (α, β) , τότε το ξ εί­

ναι μοναδικό. γ) Η εφαπτομένη της γραφικής παράστασης

της f στο σημείο Μ (ξ,f (ξ)) περνά από την

αρχή των αξόνων. Λύση

α) Η συνάρτηση g ( χ ) = f (χ ) είναι ορισμένη και χ

συνεχής στο [α, β] ως πηλίκο συνεχών και πα-ραγωγίσιμη στο (α, β) με

Επιπλέον

g 1 ( x ) = xf 1 ( x ) - f ( x ) χ2

g (α) = g (β) και σύμφωνα με το Θεώρημα Rolle υπάρχει τουλάχιστον ένα ξ Ε (α, β) ώστε

g l (ξ) = O � ξf l {ξ�; f {ξ) = 0 � ξf 1 (ξ) = f {ξ)

β) Η συνάρτηση h (χ ) = χf 1 ( χ ) - f ( χ ) έχει ρίζα το ξ Ε (α, β) , είναι παραγωγίσιμη στο (α, β) με h 1 ( x ) = f " ( x ) * O για κάθε χ Ε (α, β) . Επειδή h 1 είναι συνεχής και h 1 ( χ ) * Ο για κάθε χ Ε (α, β) έπεται ότι η h 1 θα διατηρεί σταθερό πρόσημο στο (α, β) οπότε η h θα είναι γνησίως μονότονη στο (α, β) και επομένως το ξ είναι μοναδικό.

γ) Η εφαπτομένη της γραφικής παράστασης της f στο Μ (ξ, f (ξ)) είναι

(ε) : ψ - f {ξ) = f 1 ( ξ) ( χ - ξ) �

� ψ = f 1 {ξ) χ - ξf 1 {ξ) + f {ξ) και επειδή f {ξ) - ξf 1 {ξ) = Ο έπεται ότι η (ε) εί-

ναι της μορφής ψ = λχ οπότε διέρχεται από την αρχή των αξόνων.

Άσκηση 8 Οι συναρτήσεις f και g είναι παραγωγίσιμες στο JR. με f1 (x) = κf (x) + g (x) και g1 (x) = κg (x) - f (x) , f (O) = O και f 1 (0) = 1 με

κ Ε JR. • Ν α δείξετε ότι η συνάρτηση h ( χ) = [ f2 (χ) + g2 (χ) J e-zκx είναι σταθερή και

ότι ισχύει f2 (χ) + g2 (χ) = e2κχ •

Λύση

Η h είναι παραγωγίσιμη στο JR. αφού προκύπτει από άθροισμα και γινόμενο παραγωγίσιμων συ­ναρτήσεων. Επιπλέον: h 1 ( x ) = 2[f 1 ( x ) f { x ) + g1 ( x ) g ( x )J e-2κx --2κ [ f 2 (χ) + g2 (χ) J e-2κx = 2[f1( x)f( χ) +g1(x) g( χ) -κf2 (χ) -κg2 ( χ) Je-2κx =

= 2[ κf 2 ( x ) + f ( x ) g ( x ) + κg2 ( x ) ­

-f ( x ) g ( x ) - κf 2 ( x ) - κg2 ( x )J e-2κx = 0 για κάθε χ Ε JR.. Επομένως h { χ ) = c . ( 1 ) Όμως η δεδομένη f 1 ( χ ) = κf ( χ ) + g (χ ) για χ = Ο δίνει f 1 (0) = κf (Ο) + g (O) � g (O) = 1 Οπότε η ( 1 ) για χ = Ο δίνει

[f 2 (0) + g2 (0)] e0 = c � c = 1 .

Έτσι τελικά [f 2 ( χ ) + g2 ( χ )] e-2κχ = 1 � f 2 (χ ) + g2 ( χ ) = e2κχ

Άσκηση 9

Δίνεται η παραγωγίσιμη στο [α, β] συνάρτηση f. Αποδείξτε ότι υπάρχει ξ Ε (α, β) τέτοιο ώστε

f l (ξ) + f (ξ)

+ f (ξ) = ο ξ - β ξ - α

ΛίJση

Η ζητουμένη ισότητα για ξ Ε (α, β) ισοδύναμα γί-νεται: (ξ - α ) (ξ - β ) f 1 {ξ) + (ξ - α ) f {ξ) + {ξ - β)f (ξ) = Ο .

Το πρώτο μέλος της είναι η παράγωγος της συνάρ­τησης g ( x ) = ( χ - α) ( χ - β) f ( χ ) για χ = ξ για την

ΕΥΚΛΕΙΔΗΣ Β' λθ ' τ.2/56

Page 59: Ευκλειδης Β 59

Μαθηματικά για την Γ Λυκείου

οποία ισχύουν οι προϋποθέσεις του θεωρήματος Rolle στο διάστημα [α, β] αφού είναι γινόμενο πα-ραγωγίσιμων (άρα και συνεχών) συναρτήσεων στο [α, β] και g ( α) = g (β) = Ο . Επομένως υπάρχει ξ Ε (α, β) έτσι ώστε g' (ξ) = Ο απ' όπου προκύπτει η ζητουμένη ισότητα.

Λσκηση 1 0 α ) Δίνεται μία συνάρτηση f παραγωγίσιμη στο

[0, 1] με f (O) = f (1) και f (x) * O για κάθε χ Ε [0, 1] . Να δείξετε ότι η εξίσωση f ' (x)f (1 - x) + f (x)f ' (1 - x) = O έχει μία τουλάχιστον ρίζα στο (Ο, 1) .

β) Δίνεται συνάρτηση f παραγωγίσιμη στο JR με f (χ) * Ο για κάθε χ Ε IR .

Αποδείξτε ότι η εξίσωση f' (α + χ) f' (α - χ) , , ( )

= ( )

, με α > Ο , εχει μια του-f α + χ f α - χ

λάχιστον ρίζα στο JR •

. \ ω Για χ Ε (Ο, 1) και f ( χ ) * Ο , είναι: f ' ( χ ) f ( l - χ ) + f ( x)f ' ( l - x) = O<c> [ f�l�ω

· = 0

Θεωρώ τη συνάρτηση g ( χ ) = ; ( χ ) ) , η οποία

f 1 - χ είναι παραγωγίσιμη στο [Ο, 1] [ αφού προκύπτει από σύνθεση και πηλίκο παραγωγίσιμων συ­ναρτήσεων.] Επιπλέον

f (O) f ( 1) g (O) = f (1)

= f (O)

= g (l ) .

Σύμφωνα με το Θεώρημα Rolle η εξίσωση g ' ( χ ) = 0 <=:> f ' ( χ ) f ( 1 - χ ) + f ( χ ) f ' ( 1 - χ ) = 0 έχει μία τουλάχιστον ρίζα στο (Ο, 1) .

β ) Για χ Ε JR η εξίσωση γράφεται ισοδύναμα: f ' ( α + χ ) f ( α - χ ) - f ( α + χ ) f ' ( α - χ ) = Ο <=:>

<=> [f (α + x ) f (α - x )J' = 0

Έτσι θεωρούμε την συνάρτηση h ( χ ) = f (α + χ ) f (α - χ ) η οποία ικανοποιεί τις

προϋποθέσεις του θεωρήματος Rolle στο [-α, α] αφού είναι παραγωγίσιμη στο JR (άρα και συνεχής) και επιπλέον

h (-α) = f (Ο) f ( 2α) = h (α) . Έτσι η εξίσωση

h' ( χ ) = ο <=:> f ' ( α + χ ) f ( α - χ) - f ( α + χ ) f ' ( α - χ ) = 0

έχει μία τουλάχιστον ρίζα στο (-α, α) οπότε και στο JR

Άσκηση 1 1 Δίνεται η παραγωγίσιμη στο JR συνάρτηση f. Α ν υπάρχει α Ε (Ο, +οο) τέτοιος ώστε f (α+ 1)

__

f (α) + 1 (1) δ 'ξ , , , απο ει τε οτι υπαρχει α+1 α

τουλάχιστον ένα ξ Ε ( α,α + 1) τέτοιο ώστε

f ' (ξ) = ξ + f (ξ)

ξ

Λύση

Η ζητουμένη ισότητα για ξ Ε (α, α + 1) ισοδύναμα γίνεται:

ξf ' (ξ) - f (ξ) = ξ2 <=:> ξf ' (ξ�; f (ξ) = 1 <=:>

<=:> ξf ' (ξ) - f (ξ) - 1 = 0 ξ2

αφού ξ Ε (α, α + 1) και α > Ο δηλαδή ξ * Ο . Το πρώτο μέλος της είναι η παράγωγος g ' (ξ) με

g ( χ ) = f ( χ ) - χ για την οποία ισχύουν οι προϋ-χ

ποθέσεις του θεωρήματος Rolle . Πράγματι η g εί-ναι συνεχής στο [α, α + 1] και παραγωγίσιμη στο ( α.α + 1) αφού η f είναι παραγωγίσιμη στο JR και επιπλέον

f (α+ 1) f (α) g (α+ 1) = - α - 1 =--α = g (α) α + 1 α

λόγω της ( 1 ) . Άρα υπάρχει ξ Ε (α, α + 1) , τέτοιο ώστε g ' (ξ) = Ο , από την οποία προκύπτει η ζητούμενη ισότητα.

ΕΥΚΛΕΙΔΗΣ Β' λθ ' τ.3/57

Page 60: Ευκλειδης Β 59

ΓΕΝΙΚΗΣ ΠΑΙΔΕΙΑΣ

Πιθανότητες του Θανάση Χριστόπουλου - Παναγιώτη Χριστόπουλου

Η ανάγκη του ανθρώπου να μελετήσει και να κατανοήσει το «τυχαίο» οδήγησε στη θεωρία Πιθανοτή­

των. Η έννοια του τυχαίου βρίσκεται στην αδυναμία μας να προβλέψουμε με βεβαιότητα το αποτέλεσμα ε­

νός πειράματος, σε αντίθεση με τα αιτιοκρατικά φαινόμενα στα οποία τα αποτελέσματα υπακούουν σε γνω­

στούς νόμους. Η πιθανότητα ενός ενδεχομένου είναι επομένως ένα μέτρο της τυχαιότητας, (υποκειμενικό ή

αντικειμενικό).

Για την ανάπτυξη των πιθανοτήτων έχουν συμβάλλει λίγο ή πολύ αρκετοί μαθηματικοί.

Ενδεικτικά αναφέρουμε κατ ' αρχήν τους: Pascal (1623-1662) και Fermat (1608-1665) προβληματίζονται

πρώτοι με τα τυχερά παιχνίδια και έβαλαν τις βάσεις της θεωρίας των πιθανοτήτων. ( Ο Fermat βέβαια ή­

ταν ερασιτέχνης Μαθηματικός αφού ήταν δικαστικός ! ). Η έννοια της πιθανότητας ορίστηκε από τον Laplace (1812) με τον γνωστό κλασικό ορισμό

Ρ( Α) = Ν( Α) /Ν( Ω), για την περίπτωση που ο δειγματικός χώρος του πειράματος τύχης αποτελείται από ι­

σοπίθανα απλά ενδεχόμενα και το πλήθος τους να είναι πεπερασμένο (όχι άπειρο).

Αργότερα από τον Richard von Mises όρισε την πιθανότητα ενός ενδεχομένου ως το όριο της σχετικής

συχνότητας του ενδεχομένου σε άπειρες επαναλήψεις του πειράματος . .

Τελικά από τον Kolmogorov (Κολμογκόρωφ) έχουμε τον αξιωματικό ορισμό που διόρθωσε πολλά μειο­

νεκτήματα των προηγούμενων.

Ασκήσεις 1. Χαρακτηρίστε σαν Σωστές ή Λάθος τις παρα­

κάτω προτάσεις. Για κάθε ζεύγος ενδεχομέ­νων Α, Β και για κάθε συνάρτηση πιθανότη­τας ισχύει ότι:

'

i) Ρ (Α' ) = 1 - Ρ(Α) .

ii) Αν Α ς Β τότε Ρ(Α Π Β) = Ρ(Α) .

iii) Αν Ρ(Α) = Ρ(Β) τότε Α =Β

ίν ) Αν Ρ(Α) :::;; Ρ (Β) τότε Α ς Β

ν) Αν Ρ(Β) = 1 - Ρ(Α) τότε Β = Α·

(Απάντηση : Σ - Σ - Λ - Λ - Λ)

2) Έστω Ω ο δ. χ. ενός πειράματος τύχης και Α, Β ςΩ. Αν Ρ(Α) = 0,6 και P(B) = O,S

δείξτε: i ) Τα Α,Β δεν είναι ασυμβίβαστα. ί ί ) 0,1 :::;; Ρ ( Α Π Β) :::;; 0,5 .

Απάντηση

i) Αν τα Α, Β ήταν ασυμβίβαστα τότε θα ίσχuε:

P(A U B) = P(A) + P(B)

Άρα: P(A U B) = 1, 1 άτοπο ! Αφού P(A U B) :::;; 1 .

Άρα τα Α, Β δεν είναι ασυμβίβαστα. ii ) Αφού Α Π Β ς Β ισχύει: Ρ(Α Π Β) :::;; Ρ (Β)

Άρα: Ρ(Α Π Β) :::;; Ο, 5

Επίσης: P(AU B) :::;; 1 � Ρ(Α) + Ρ(Β) -Ρ(Α Π Β) :::;; 1 �

0, 6 + 0, 5 - Ρ(Α Π Β) :::;; 1 � 1, 1 - 1 :::;; Ρ ( Α Π Β) � 0, 1 :::;; Ρ(Α Π Β) .

Άρα: 0, 1 :::;; Ρ (Α Π Β) :::;; ο, 5 .

3) Α ν Α, Β ενδεχόμενα ενός δειγματικού χώ­ρου Ω πειράματος τύχης (π. τ.) για τα οποία ισχύει:

P(A U B) = �� ' Ρ(Α' ) + Ρ(Β' ) = � ·

Να βρεθούν οι πιθανότητες των ενδεχομέ-νων: i) Α Π Β και ii) A' U B' .

ΕΥΚΛΕΙΔΗΣ Β ' λθ ' τ.3/58

Page 61: Ευκλειδης Β 59

Μαθηματικά για την Γ Λυκείου

Απάντηση

i) Από τα δεδομένα προκύπτει ότι:

Ρ( Α, ) + Ρ ( Β' ) = � � 1 - Ρ( Α) + 1 - Ρ( Β) = � � 7 7

2 -% = Ρ(Α) + Ρ(Β) � Ρ(Α) + Ρ(Β) = % ·

P (A U B) = Ρ( Α) + Ρ(Β) - Ρ(Α Π Β) �

Ρ( Α Π Β) = P(A) + P(B) - P(A U B) �

Ρ(Α Π Β) = � - 33 = .!2. . 7 35 35

ii) Ρ (Α' U Β' ) = Ρ(Α Π Β)' = 1 - Ρ(Α Π Β) = .

= 1 - .!2. = li 35 35

(Ισχύει: Α, U Β. = (Α Π Β).)

4 ) Αν Ω = {ι, 2, 3,4, 5} δειγματικός χώρος ενός πειράματος τύχης και Ρ ( 2) = 2Ρ (ι) επίσης

Ρ (κ} = .!. για κάθε κ Ε Ω με κ > 2 •

κ i) Ποιες οι πιθανότητες των απλών ενδε­

χομένων του Ω. i i ) Α ν Ε= {λ ε Ω/ λ θέση τοπικού αιφοτάτου της

f (χ) = χ3 - 6χ2 + 9χ + 2} να βρεθεί η

Ρ(Ε) . Απάντη ση

Ισχύει: Ρ ( 1 ) + Ρ (2) + Ρ (3) + Ρ( 4) + Ρ(5) = 1

Ρ ( 1 ) + 2Ρ(1) + _!. + ..!_ + _!. = 1 � 3 4 5

3Ρ ( 1 ) = 1 -20 + 15 + 12

� 60

47 1 3 1 3 3Ρ(1) = 1 -60

� 3Ρ(1) = 60

� Ρ(1) = 1 80

. ( 1 3 26 1 Άρα: Ρ 1) = 1 80

, Ρ (2) = 1 80

, Ρ (3) = 3 ,

Ρ(4) =� , Ρ (5) = � .

ii) Για κάθε χ Ε JR ισχύει: ( (χ) = 3χ2 - 1 2χ + 9 ,

Έχουμε: ( ( χ ) = Ο � χ2 - 4χ + 3 = 0 � χ = 1, χ = 3

Το πρόσημο της παραγώγου και η μονοτονία της f φαίνεται στο σχήμα:

χ ι 3 Γ( χ) + - +

f / � / Άρα: η f παρουσιάζει στα 1 και 3 τοπικά ακρό-τατα. Επομένως: Ε = { 1, 3} . ( )

1 3 1 73 Άρα: Ρ Ε = Ρ(1) + Ρ(3) = - + - = - . 1 80 3 1 80

5) Το 36% των μαθητών ενός σχολείου δεν μιλούν καλά Αγγλικά, το 42% μιλούν καλά Γαλλικά, ενώ το 24% δε μιλούν καλά ούτε Αγγλικά ούτε Γαλλικά. Ποια η πιθανότητα ένας μαθητής του σχολείου α) να μιλά καλά και τις δύο γλώσσες β) Να μιλά καλά μόνο Γαλλικά.

Απάντη ση

Θεωρούμε τα ενδεχόμενα: Α: «0 μαθητής μιλάει καλά Αγγλικά» Β : «0 μαθητής μιλάει καλά Γαλλικά».

- Το 36% των μαθητών δε μιλούν καλά Αγγλι­κά. Άρα: Ρ ( Α' ) = 36% = 0, 36 άρα

Ρ( Α) = 1 - 0, 36 = 0, 64 .

- Το 42% των μαθητών μιλούν καλά Γαλλικά Ρ(Β) = 0, 42 .

- Το 24% των μαθητών δε μιλούν καλά ούτε Αγ­γλικά ούτε Γαλλικά.

Άρα: P(A U B). = 0, 24

� P(A U B) = 1 - 0, 24 = 0, 76 .

α) Ρ(Α U Β) = Ρ(Α) + Ρ(Β} - Ρ(Α Π Β) �

Ρ(Α Π Β) = P (A) + P(B) - P(A U B)

Άρα: Ρ(Α Π Β) = 0, 64 + 0, 42 - 0, 76 = 0, 30 ή

30% β) Ρ(Β - Α) = Ρ( Β) - Ρ( Α Π Β} =

= 0, 42 - 0, 30 = 0, 1 2 = 1 2 %.

6) Μια κάλπη περιέχει 30 σφαιρίδια αριθμημένα από το ι έως το 30. Α ν επιλέξουμε ένα σφαιρίδιο τυχαία, ποια η πιθανότητα α) η ένδειξή του να είναι πολ/σιο του 4 όχι

ΕΥΚΛΕΙΔΗΣ Β' λθ ' τ.l/59

Page 62: Ευκλειδης Β 59

Μαθηματικά για την Γ Λυκείου

όμως και του 3. β ) Η ένδειξη να είναι πρώτος αριθμός.

Απάντη ση

α) Α: «πολ/σιο του 4» Άρα Α = {4, 8, 1 2, 1 6, 20, 24, 28} Β: "πολ/σιο του 3" Άρα Β = {3, 6, 9, 1 2, 1 5, 1 8, 2 1, 24, 27, 30} Άρα: Α Π Β = { 1 2, 24} Έχουμε: Ν (Ω) = 30 , Ν (Α) = 7 , Ν (Α Π Β) = _!_

1 5

Άρα: Ρ(Α) =_2_

, Ρ(Α Π Β) =2

και 30 30

Ρ(Α - Β) = Ρ(Α) - Ρ(Α Π Β) = ;Ο -3� = :ο = i

β) Γ: «ένδειξη, πρώτος αριθμός» Γ = { 2, 3, 5, 7 , 1 1, 1 3, 1 7, 1 9, 23, 29} , Ν {Γ) = 10

Άρα: Ρ (Γ) = _!_2_ = .!_ . 30 3

7) Αν Α, Β ενδεχόμενα ενός δειγματικού χώ­ρου Ω πειράματος τύχης (π. τ.). Αν Ρ(Β) = Ρ(Α' nB) και Ρ(Β' ) = Ρ(ΑΠΒ' ) τότε τα ενδεχόμενα Α, Β είναι συμπληρω­ματικά.

Απόδειξη

Αρκεί να δείξουμε ότι Α U Β = Ω και Α Π Β = 0 . Πράγματι: Ρ(Β) = Ρ ( Α' Π Β ) => Ρ(Β) = Ρ(Β - Α) <=>

<=> Ρ(Β) = Ρ ( Β) - Ρ( Α Π Β) Άρα: Ρ (Α Π Β) = Ο ή Α Π Β = 0 . Επίσης: Ρ ( Β' ) = Ρ ( ΑΠ Β" ) <=> 1 - Ρ(Β) = Ρ ( Α - Β) <=>

1 - Ρ( Β) = Ρ ( Α) - Ρ(Α Π Β) <=> Ρ( Α) + Ρ(Β) - Ρ(Α Π Β) = 1 P(A U B) = 1 άρα A U B = Ω .

8 ) Αν Ω δειγματικός χώρος ενός πειράματος τύχης με ισοπίθανα απλά ενδεχόμενα, με

( ) ( χ2 + 4 χ

Ν Ω = 30 και Ν Α ) = -2- , Ρ {Β } = 6

με Α, Β συμπληρωματικά ενδεχόμενα να βρεθούν τα Ρ { Α)

και Ρ {Β ).

Απάντη ση

χ 2 + 4 Ρ( Α) + Ρ(Β) = 1 <:::> -

2- + � = 1 <=>

30 6 χ 2 + 4 χ 2 -- +- = 1 <=> χ + l Ox - 56 = Ο <=> 60 6

<=> (χ = 4 ή χ = -14)

- Α ν χ = 4 τότε Ρ (Α) = 4 2 + 4 = 20 = .!_ 60 60 3

Ρ(Β) =_±

=�

. 6 3

και

- Αν χ = -14 τότε Ρ( Β) < Ο άτοπο. Άρα χ = 4.

9) Μια κάλπη περιέχει 3 σφαίρες, μια άσπρη (Α), μία Μαύρη (Μ) και μια κόκκινη (Κ). Επιλέγουμε τυχαία μία σφαίρα, καταγρά­φουμε το χρώμα της και την επανατοποθε­τούμε στην κάλπη, στη συνέχεια επιλέγουμε πάλι μια σφαίρα και καταγράφουμε επίσης το χρώμα της. α) Ποιος είναι ο δειγματικός χώρος του

Πειράματος τύχης; β) Ποια η πιθανότητα να επιλέξουμε τις

δύο σφαίρες με ίδιο χρώμα; γ) Ποια η πιθανότητα μια εκ των δύο του-

λάχιστον να είναι άσπρη ; Απάντηση

α) Ω = { ΑΑ,ΑΜ,ΑΚ,ΜΑ,ΜΜ,ΜΚ.,ΚΑ,ΚΜ,ΚΚ} .

β) Α = {ΑΑ,ΜΜ,ΚΚ} άρα Ρ(Α) =�

= .!_ . 9 3

γ) Β = {ΑΑ,ΑΜ ,ΑΚ,ΜΑ, ΚΑ} άρα Ρ(Β) =�

. 9

1 0) Σε ένα σχολείο το 40% των μαθητών είναι αγόρια και το 30% από αυτά έχει ξανθά μαλλιά ενώ από τα κορίτσια του σχολείου το 70% έχει ξανθά μαλλιά. Ποια η πιθανότητα επιλέγοντας τυχαία ένα μαθητή του σχολείου να έχει ξανθά μαλλιά;

Απάντηση

ΕΥΚΛΕΙΔΗΣ Β ' λθ ' τ.l/60

Page 63: Ευκλειδης Β 59

Μαθηματικά για την Γ Λυκείου

Έστω Α: «Το ενδεχόμενο ο τυχαία επιλεγόμε-νος μαθητής να είναι αγόρι» και

Β : «Το ενδεχόμενο ο τυχαία επιλεγόμενος μα­θητής να έχει ξανθά μαλλιά»

Από τα δεδομένα του προβλήματος προκύπτει ότι: Ρ (Α ) = 0, 4 και Ρ (Α' ) = 0, 6

Επίσης: Ρ (Α Π Β ) = Ο, 1 2 (αγόρια με ξανθά μαλλιά) και Ρ (Β Π Α') = 0, 42 (κορίτσια με ξανθά μαλλιά) .

Όμως: Β = (Α Π Β) U (Β Π Α') και (Α Π Β) Π (Β Π Α') = 0

Άρα: Ρ(Β) = Ρ(Α Π Β) + Ρ(Β Π Α') = 0, 12 + 0, 42 = 0, 54

ι 1 ) Αν Α, Β συμπληρωματικά ενδεχόμενα και 25Ρ2 (Α ) + 8 ::; 29Ρ {Α} - Ρ {Β ) , (1) να βρε-θούν οι Ρ {Α} , Ρ {Β } .

ΛΥΣ Η Ρ (Β ) = ι - Ρ ( Α) άρα η ( 1 ) γράφεται 25Ρ2 (Α ) - 29Ρ (Α ) + Ρ (Β) + 8 ::; Ο � 25Ρ2 (Α ) - 30Ρ (Α) + 9 ::; Ο � (5Ρ (Α) - 3)2 ::; Ο � 5Ρ (Α) - 3 = 0 � Ρ( Α) =� . 5

1 2 ) Αν Ω = {ω1 ,ω2 , • • •

,ω6 } και Ρ ( ω1 ) = Ρ2 ( ω2 ) = Ρ ( ω3 ) = = 1 - 4Ρ (ω4 ) = Ρ ( ω5 ) = Ρ ( ω6 ) i) Να βρεθούν οι πιθανότητες των απλών

ενδεχομένων του Ω. ii) Αν Α = {ω2 ,ω3 ,ω4 } .

ΛΥΣΗ i ) Έστω Ρ ( ω1 ) = Ρ2 ( ω2 ) = Ρ ( ω3 ) = 1 - 4Ρ { ω4 ) =

= Ρ (ω5 ) = Ρ {ω6 ) = χ2 Τότε Ρ (ω1 ) = Ρ (ω3 ) = Ρ (ω5 ) = Ρ (ω6 ) = χ2 ,

1 - χ2 Ρ (ω2 ) = χ , Ρ (ω4 ) = --4 6 1 χ2 ΣΡ (ωί ) = 1 � 4χ2 + χ + --- = 1 �

i=l 4 � 15χ2 + 4χ - 3 = 0

1 1 2 6 2 Ρ (Α) = - + - + - = - = - . 3 9 9 9 3

δεκτή

απορ.

13) Αν P {A U B ) = Ρ { Α Π Β ) τότε Α = Β . ΛΥΣ Η

Ρ (Α U Β ) = Ρ (Α Π Β ) � Ρ (Α ) + Ρ (Β ) - Ρ (Α Π Β ) = Ρ ( Α Π Β ) � [Ρ ( Α) - Ρ ( Α Π B )J + [P (B ) - Ρ ( Α Π Β )J = 0 � Ρ (Α - Β ) + Ρ (Β - Α) = Ο Όμως, Ρ (Α - Β ) � Ο. Ρ (Β - Α) � Ο Άρα: Ρ(Α-Β) = Ρ(Β-Α) =Ο

A - B = 0 � A c B} Άρα: - Αρα : Α = Β . Β - Α = 0 =:> Β ς Α Σχόλιο :

Α, Β είναι ενδεχόμενα δ · χ με ισοπίθανα τα απλά ενδεχόμενα.

14) Αν το μικρό ποντίκι του σχήματος δεν πη­γαίνει πίσω και σε κάθε διασταύρωση πη­γαίνει δεξιά ή αριστερά ή ευθεία με ίδια πι­θανότητα να βρεθεί η πιθανότητα να φτάσει στο τυρί.

I I I I τυρί

ο ι I I I I I

ποντίκι

Σ' αυτό το πρόβλημα θα βοηθήσει το δενδρο­διάγραμμα (Δ: δεξιά, Α: αριστερά, Ε: ευθεία)

ΕΥΚΛΕΙΔΗΣ Β' λθ ' τ.l/61

Page 64: Ευκλειδης Β 59

Μαθηματικά για την Γ Λυκείου

Δ

Δ�� �Α�� Ε�Α Ε Ω = { ΔΔ,ΔΑ, ΔΕ, ΑΔ,ΑΑ, ΑΕ, ΕΔ, ΕΑ, ΕΕ} ,

Ν (Ω) = 9 .

Ευνοϊκές περιπτώσεις { ΑΔ, ΕΑ} .

Ά θ ' ' ' ' 2 ρα η πι ανοτητα να φτασει στο τυρι ειναι - . 9

1 5) Δίνεται η εξίσωση (λ - 1}χ2 - λχ + 1 = 0 (1)

Ρίχνουμε ένα αμερόληπτο ζάρι και aντικα­θιστούμε το λ με την ένδειξη του ζαριού. Ποια η πιθανότητα α) Να έχει η εξίσωση μια διπλή ρίζα β) Να έχει 2 ρίζες πραγματικές και άνισες γ) Να έχει 1 ρίζα

Απάντηση

Α ν λ * 1 η διακρίνουσα της ( 1 ) είναι : Δ = λ2 - 4(λ - 1) = λ2 - 4λ + 4 = (λ - 2)2

α) Για να έχει η εξίσωση ( 1 ) μια διπλή ρίζα πρέπει Δ = 0 <=> λ = 2 . Αν Α: «Το ενδεχόμενο η εξίσωση ( 1 ) να έχει διπλή ρίζα», θα είναι: Α = {2} ,οπότε:

Ρ (Α) = _!. . 6

β) Για να έχει η εξίσωση ( 1 ) μια διπλή ρίζα πρέπει Δ > Ο <::> λ > 2 . Αν Β : «Το ενδεχόμενο η εξίσωση ( 1 ) να έχει 2 ρίζες πραγματικές και άνισες», θα είναι Β = {3, 4, 5, 6} οπότε : Ρ(Β) =

�.

6 γ) Η εξίσωση ( 1 ) έχει μία ρίζα για λ= 1 .

Αν Γ : «Το ενδεχόμενο η εξίσωση ( 1 ) να έχει μία ρίζα », θα είναι : Γ = {1} οπότε: Ρ (Γ) = _!. .

6

z ι χ - χ + -4

'

3 - - χ 7

Να βρεθεί η τιμή του χ. ΛVΣΗ

i � 3

4 � i � ll

i ;::: 12

Πρέπει: Ρ( ω1 ) + Ρ( ω2 ) + . . . + Ρ( ω1 8 ) = 1 <=>

3χ3 + 8( χ2 - χ +�) + 7 (% - χ) = 1 <=>

3χ3 + 8χ2 - 1 5χ + 4 = 06

( χ = 1 ή χ = -4 ή χ = �) . Η τιμή χ = 1 απορρίπτεται διότι Ρ ( ω1 2 ) =

� - 1 = -� άτοπο. 7 7 Επίσης απορρίπτεται και η τιμή χ = -4 διότι Ρ( ω1 ) = -64 < 0

Γ 1 ' Ο ια χ = "3 ισχυει, � Ρ( ω) � 1, i = 1, 2, . . . , 1 2 ο-

πότε: Ρ ( ω1 ) = Ρ( ω2 ) = Ρ ( ω3 ) =-1 27 1 Ρ( ω4 ) = Ρ ( ω5 ) = . . . = Ρ( ω1 1 ) =-36

2 Ρ ( ω1 2 ) = . . . = Ρ( ω1 8 ) = - . 2 1

ΑΣΚΗΣΕΙΣ ΠΡΟΣ Λ ΥΣΗ

ΑΣ ΚΗ ΣΗ lη

Έστω Ω ο δειγματικός χώρος στη ρίψη ενός ζα­ριού. Αν Ε το ενδεχόμενο Ε = {λ ε ΩΙ IΡ(Α) - 21 + 13Ρ (Α) + 11 = λ - 1} , Α * 0

Ν α βρεθεί η πιθανότητα Ρ (Ε) .

ΑΣΚΗΣΗ 2'1

(Απ : Ρ(Ε) = �

) 6

Οι εργάτες ενός εργοστασίου φτάνουν σ' αυτό χρησιμοποιώντας λεωφορείο, τρένο ή αυτοκίνητο. Αυτοί που χρησιμοποιούν αυτοκίνητο δεν χρησι­μοποιούν άλλο μέσο. Η πιθανότητα να μη χρησι-

16) Αν Ω = {ω1 ,ω2 , ••• ,ω18 } ο δειγματικός χώ­

ρος ενός πειράματος τύχης μοποιούν 2 μέσα είναι 80% ενώ η πιθανότητα να

ΕΥΚΛΕΙΔΗΣ Β' λθ ' τ.l/62

Page 65: Ευκλειδης Β 59

Μαθηματικά για την Γ Λυκείου

χρησιμοποιούν μόνο λεωφορείο ή μόνο τρένο είναι 55%. Ποια η πιθανότητα ένας εργαζόμενος να φτάνει στο εργοστάσιο με αυτοκίνητο;

(Απ: 25%) ΑΣ ΚΗΣΗ 3'1

Αν Ω = {1, 2, 3, 4, 5, 6} ο δειγματικός χώρος ενός πειράματος τύχης και έστω λ, 2λ, 3λ, . . . , 20λ, λ ε Ω οι παρατηρήσεις μιας μεταβλητής Χ. Α ν Α το ενδεχόμενο η μέση τιμή χ είναι μεγαλύ­τερη από 50 και Β : το ενδεχόμενο οι 5 πρώτες παρατηρήσεις να έ­χουν διακύμανση μεγαλύτερη από 30 Να βρεθούν οι πιθανότητες:

Ρ (Α) , Ρ (Β) , Ρ (Α - Β) , Ρ (Β - Α) .

ΑΣ Κ Η Σ Η -111

1 1 1 (Απ: - , - , Ο, - ) . 3 2 6

Σε μια πόλη το 10% των ανδρών και το 20% των

ου λάβαμε:

γυναικών, καπνίζουν. Αν στο σύνολο ανδρών και γυναικών οι καπνίζοντες είναι 1 6%. Να βρεθεί το ποσοστό των ανδρών.

(Απ: 40%). Αl: Κ ΗΣ Η 5' 1

Ρίχνουμε διαδοχικά 2 ζάρια και σημειώνουμε τις ενδείξεις τους σε ένα διατεταγμένο ζεύγος. Α ν Ω ο δειγματικός χώρος αυτού του πειράματος τύχης, θεωρούμε τα ενδεχόμενα

Χ = {(χ , y ) ε Ω/ το σημείο M (x , y) } ανήκει στην ευθεία y = 2χ - 1

Υ={(χ,y) εΩ!το�Μ(χ,y) �σrηγραφική } nοprοταση της σmφtηση:; Υ = xz

Να βρεθούν οι πιθανότητες: Ρ(Χ) , Ρ (Υ) , Ρ (Χ - Υ) .

Οι Αναδρομικές Ακολουθίες και ο Υπολογισμός του Αν με Α Τετραγωνικό Πίνακα και v ε Ν , Κωνσταντίνου Μιχ. Τσιλικούδη .

23° Συνέδριο Μαθηματικής Παιδείας Πάτρα, 24 - 26 Ν ο βρίου 2006

"Τα μαθηματικά ως πολιτισμός στο σύγχρονο κόσμο, προεκτάσεις στην κριτική σκέψη, στην επι­χειρηματολογία και στην αισθητική ."

ΕΥΚΛΕΙΔΗΣ Β ' λθ ' τ.l/63

Page 66: Ευκλειδης Β 59

προτεiνε� Εuκλεiδn

«Η καρδιά των μαθηματικών είναι τα προβλήματα και οι λύσεις και ο κύριος λόγος ύπαρξης του μαθηματικού είναι να λύνει προβλήματα».

P. R. HALMOS

Την ευθύνη της διατύπωσης και της ορθότητας της λύσης

κάθε άσκησης την έχει ο συνάδελφος που στέλνει τη λύση.

Επιμέλεια: Α. Κυριακόπουλος, Γ. Στρατής, Γ. Τριάντος, Ν. Αντωνόπουλος ΑΣΚΗΣΕΙΣ Γ ΙΑ Λ ΥΣ Η

65. Αν μ και ν είναι ακέραιοι θετικοί αριθμοί πρώτοι προς αλλήλους, αποδείξτε ότι και οι αριθμοί:

μ3ν2 + μ2ν3 + μ3ν + 2μ2ν2 + μν3

και μ 2ν + μν2 + μν + μ + ν είναι επίσης πρώτοι προς αλλήλους. ( Επροτάθη από τον

Ακαδημαϊκό κύριο Νικόλαο Αρτεμιάδη ).

Λύση (Από τον ίδιο) • Κατ' αρχάς δείχνουμε, με δυο διαφορετικούς

τρόπους, την πρόταση : Αν α, β ε Ν* με (α, β) = 1 , τότε (αβ, α+β) = 1

1 "ς τρόπος

Έστω (α+β, αβ) = δ, δ > 1 . Τότε υπάρχει, ένας τουλάχιστον πρώτος διαιρέτης του δ, έστω ρ, οπότε: (ρ/(α+β) και ρ/αβ) � (ρ/( α + β) και (ρ/α ή ρ/β)). - Αν ρ/α και ρ/α+β, τότε ρ/β, οπότε ρ/(α, β)= 1 ,

άτοπο. - Αν ρ/β και ρ/α+β, τότε ρ/α, οπότε ρ/(α,β)= 1 ,

άτοπο. Άρα (α+β, αβ) = 1

2"ς τρόπος

Έστω (αβ, α+β) = δ. Τότε δ/αβ και δ/α+β, οπότε δ/α(α+β)-αβ και δ/β(α+β)-αβ που σημαίνει ότι δ/α2 και δ/β2• Επειδή (α, β) = 1 , ως γνωστόν και οι αριθμοί α2, β2 είναι σχετικά πρώτοι. Άρα δ/ 1 , οπότε δ = 1 .

• Για την άσκηση θέτουμε: χ = μ 3ν2 + μ 2ν3 + μ 3ν + 2μ 2ν2 + μν3 = = μ2ν2 (μ + ν) + μν(μ + ν)2 και y = μ 2ν + μν2 + μν + μ + ν =

= μν(μ + ν) + (μν + (μ + ν)] ( 1 ) Θέτουμε: κ = μν και λ = μ+ν. Τότε (κ, λ) = 1 και οι συνθήκες ( 1 ) γίνονται:

χ = κ2λ + κλ2 = κλ(κ + λ) και y = κλ + (κ + λ) (2) Αν τώρα θέσουμε κλ = ρ και κ+λ = σ, τότε (ρ, σ)=1 και οι συνθήκες (2) γίνονται χ = ρσ και y=ρ+σ, οπότε (χ, y) = 1 , που είναι το ζητούμενο. Λύσεις με παρόμοια προσέyyιση λάβαμε και από

τους συναδέλφους Ηλιόπουλο Γιάννη - Κιι.λυ.μά­τα και Σταματόγιαννη Γ ιίη'\'1] -Δροmά Αττικής.

Λύσεις έστειλαν επίσης ο καθηγητής Μπόλης

Θόδωρος, ο κύριος ' !πόρης Ρ ο - Δάφνη Αττι­

κής, οι συνάδελφοι Κυ.ραβό·ως Ληι.11rιτρως -Κάτω Αχαί'α, Κο(φτης Χρυσόστομος - Λάρι­

σα, Τσοπέλας i ω (ι.ννης - Αμαλιάδα, Ηλίας

Κων/νος - Αλιβέρι, Τσαπακίδης Γιιί)ιηος -Αγρίνιο και ο χημικός Δη μ. Καρβ:':λας - Πεύκη.

66. Δίνεται τρίγωνο ΑΒΓ. Αποδείξτε ότι στο εσωτερικό του τριγώνου ΑΒΓ υπάρχει ση­μείο Ρ τέτοιο ώστε: ΡΙ'Β = Ρ ΑΒ και ΡΒΓ = Ρ ΑΓ τότε, και μόνο τότε, όταν το τρίγωνο ΑΒΓ είναι οξυγώνιο (Επροτάθη από τον Ακαδημαϊκό κύριο Ν ικόλαο Αρτεμιάδη ).

ΕΥΚΛΕΙΔΗΣ Β ' λθ ' τ.3/64

Page 67: Ευκλειδης Β 59

Ο Ευκλείδης προτείνει ... Ευκλείδη ..• και Διόφαντο

Λύση (Από τον ίδιο) Έστω ότι οι προεκτάσεις των ΑΡ, ΒΡ, ΓΡ τέ­μνουν τον περιγεγραμμένο κύκλο του τριγώνου ΑΒΓ στα σημεία Α ' , Β ' , Γ αντίστοιχα. Τότε ΒΑ' = ΒΓ' , οπότε η ΒΒ ' είναι διχοτόμος της γωνίας Α'Β 'Γ ' . Ομοίως η ΓΓ ' είναι διχοτό­μος της γωνίας Α'Γ 'Β ' .

Α� Β '

Γ

Α ' Επομένως το σημείο Ρ είναι το έγκεντρο του τριγώνου Α 'Β 'Γ, οπότε και η ΑΑ ' είναι διχο­τόμος της γωνίας Β Ά 'Γ, πράγμα που σημαίνει ότι: ΡΒΑ = PfA . Η γωνία που σχηματίζουν οι ΑΓ και ΒΒ ' είναι: !crn· + Ar13) = _!_(ffiι + Α? + FB) = 2 2 = ΓΒΒ' + ΑfΓ' + ΒΓΓ' = 90° Άρα ΒΒ' l_ ΑΓ , οπότε η ΒΒ ' είναι ύψος του τριγώνου ΑΒΓ. Ομοίως προκύπτει ότι οι ΑΑ ' και ΓΓ ' είναι ύψη του τριγώνου ΑΒΓ. Επομέ­νως το σημείο Ρ είναι το ορθόκεντρο του τρι­γώνου ΑΒΓ που σημαίνει ότι το Ρ είναι στο εσωτερικό του τριγώνου ΑΒΓ, τότε και μόνο τότε, όταν το τρίγωνο είναι οξυγώνιο. Λύση με τριγωνομετρική προσέγγιση πήραμε από

τον κύριο Α νδρή Ιωάννη, Πολιτικό Μηχανικό ­

Αθήνα. Λύση έστειλαν επίσης ο κύριος Μπόρης

J>. - Δάφνη Απικής, ο κύριος Λουκάς Χυτή ρης

- Αθήνα και οι συνάδελφοι Τσαπακίδης Γιώρ­

γος - Αγρίνιο και Ηλίας Κων/νος - Αλιβέρι.

67. Ένα πολυώνυμο Ρ(χ), με πραγματικούς συ­ντελεστές, έχει όλες τις ρίζες του πραγματι­κές. Ν α δείξετε ότι για κάθε χ Ε JR ισχύει: {Ρ'(χ) γ � Ρ(χ) · Ρ"(χ) . ( Επροτάθη από τον Ακαδημαϊκό κύριο Ν ικό­

λαο Αρτεμιάδη).

Λύση (Από τον ίδιο) Έστω ότι το πολυώνυμο είναι ν βαθμού. Τότε ισχύει:

P(x) = αv (x-p1 )(x - p2 ) . . . (x - pv ) με αν :;t: O Η αποδεικτέα για χ = Pi , i Ε {1, 2 . . . , ν} γίνεται (P'(p) )

2 � Ο , ( 1 ) που ισχύει. (Η ισότητα όταν η ρίζα Pi του πολυώνυμου Ρ(χ) είναι και ρίζα του Ρ'(χ) , δηλαδή όταν η ρίζα Pi είναι πολλα­πλότητας τουλάχιστον δυο για το πολυώνυμο Ρ(χ) ). Για όλους τους αριθμούς χ, που δεν εί-ναι κάποιο από τις ρίζες του πολυώνυμου Ρ(χ) έχουμε: Rn iP(x) l = = Rn lαv l + Rn l x - p1 1 + Rn l x - p2 1 + . . . + Rn l x - Pv l οπότε: Ρ'(χ) 1 1 1 , -- = -- +-- + . . . + -- για καθε χ, Ρ(χ) Χ - Ρι Χ - p2 Χ - pv

με x :;t: pi , i Ε {1 , 2, . . . , ν} . Άρα: ( Ρ'(χ) )' = Ρ( χ)

-1 -1 -1 = 2 + 2 + . . . + 2 < 0 ( χ - Ρ ι ) ( χ - Ρ2 ) ( χ - Pv )

Ρ"(χ)Ρ(χ) - (Ρ'(χ) )2

οπότε 2 < 0 , (Ρ( χ))

απ' όπου προκύπτει ότι για κάθε χ με χ :;t: Pi , i Ε {1, 2, . . . , ν} ισχύει (Ρ'(χ) )2 > Ρ(χ)Ρ"(χ) (2) Από τις ( 1 ) και (2) προκύπτει το ζητούμενο. 1δια λύση πήραμε και από το συνάδελφο Ηλιό­

πουλο Γιάννη - Καλαμάτα. Λύσεις έστειλαν ε­

πίσης ο συνάδελφος Γιάννης Σταματογιάννης

- Δροσιά Απικής, Καραβότας Δημήτρης -Κάτω Αχαία, ο κύριος Μπόρης Ρ. - Δάφνη Ατ­

τικής, ο καθηγητής Μπόλης Θόδωρος και οι

συνάδελφοι Κούρτης Χρυσόστομος, Λάρισα

και Τσαπακίδης Γιώργος, Αγρίνιο.

68. Δίνεται ορθογώνιο τρίγωνο ΑΒΓ με υποτεί­νουσα ΒΓ και ΑΓ>ΑΒ. Στην πλευρά ΓΑ παίρνουμε τμήμα Γ Δ = ΑΒ και στην πλευρά ΒΑ τμήμα ΒΕ = ΑΔ. Να αποδείξετε ότι η οξεία γωνία των ευθειών ΒΔ και ΓΕ είναι ίση με 45°. Επροτάθη από το συνάδελφο Μπάμπη

Στεργίου - Χαλκίδα)

Λύση (Από τον ίδιο) Α ν Ζ είναι το συμμετρικό του σημείου Α ως προς το μέσο της υποτείνουσας ΒΓ, τότε το τετρά-

ΕΥΚΛΕΙΔΗΣ Β ' λθ ' τ.3/65

Page 68: Ευκλειδης Β 59

Ο Ευκλείδης προτείνει .•. Ευκλείδη ••• και Διόφαντο

πλευρο ΑΓΖΒ είναι ορθαyώνιο. Έστω Η σημείο της πλευράς ΒΖ ώστε να ισχύει ΒΗ = Γ Δ.

Β Η Ζ - - - - - - - - - - - - ::: :7\ _ _ _ _ _ _ _

\ \

\ \

\ \

\ \

\ \

\ \

\ \

\ \

\ "

Α Γ Από την ισότητα των ορθογωνίων τριγώνων ΒΕΗ και ΖΗΓ, προκύπτει ότι ΗΕ = ΗΓ, οπότε το τρίγωνο ΕΗΓ είναι ισοσκελές. Το τετρά­πλευρο ΔΓΗΒ είναι παραλληλόγραμμο, οπότε: ΒΗΓ + ΗΓΔ = ι 8Οο => ΒΗΕ + ΕΗΓ + ΗΓΔ = ι 8Οο => ΕΗΓ + ΖΓΗ + ΗΓΔ = ι 80° => ΕΗΓ + 90° = ι 80° => ΕΗΓ = 90° Άρα το τρίγωνο ΕΗΓ είναι ορθογώνιο και ισο­σκελές, οπότε η γωνία ΗfΈ , που είναι ίση με την οξεία γωνία των ΒΔ και ΓΕ, θα είναι 45° . Λύσεις μας έστειλαν επίσης οι συνάδελφοι Γιάννης

Σταματογιάννης - Δροσιά Ατrικής, Αποστολό­

πουλος Γιώργος - Μεσολόyγι, Κοίφτης Χρυσό­

στομος, Λάρισα ο κύριος Ρ. Μπόρης - Δάφνη Ατ­

τικής και οι μαθητές Νάκος Βασίλης - jD Ε.Λ.

Ηρακλείου Ατrικής και Νίκος Μαραβίτσας - (f Λύκειο Περιστερίου, ο καθηγητής Μπόλης Θό­

δωρος και οι συνάδελφοι Ηλίας Κων/νος - Αλι­

βέρι και Τ σαπακίδης Γιώργος - Αγρίνιο.

69. Δίνεται τρίγωνο ΑΒΓ και έστω Ρ τυχαίο

εσωτερικό σημείο του τριγώνου. Α ν D, Ε, F είναι προβολές του σημείου Ρ, να υπολογί­

σετε το μέγιστο της παράστασης PD · PE · PF

(Επροτάθη από το συνάδελφο PA · PB · PC Σωτή ρη Σκοτίδα, Καρδίτσα)

Λύση (Από τον ίδιο) Α

Β c

, PD ΡΕ Ισχυει: ημΑ1 = - , ημΑ2 = -ΡΑ ΡΑ

Αλλά ημα · ημβ $ η μ 2 ( α;β ) , (είναι ισοδύνα-

μη με την συν( α-β) ::; ι η οποία ισχύει), οπότε:

ημΑι · ημΑ2 S ημ2 ( Αι � Α2 )=>

PD · PE < 2 (Α ) ΡΑ2 - ημ

2 , PE · PF 2 (Β ) Ομοιως 2 $ ημ - και

ΡΒ 2 PD · PF 2 (C ) , -

P-C

-:-2- $ η μ l , οποτε:

PD · PE · PF Α Β C , , ---- S ημ-ημ-ημ- (Η ισοτητα οταν PA · PB · PC 2 2 2 οι ΡΑ, ΡΒ, ΡΒ είναι διχοτόμοι των γωνιών του

τριγώνου) . Εξάλλου η συνάρτηση f(χ) = ημ� 2

είναι κοίλη στο [Ο,�] , οπότε από την ανισότη-

τα Jensen, προκύπτει: Α Β C

ι ( Α Β c ) 2 + 2 + 2 ι - ημ- + ημ- + ημ- S ημ = -3 2 2 2 3 2

(Η ισότητα όταν Α = Β = C). Τέλος από την ανισότητα Cauchy έχουμε: [ημ% + ημ� + ημ� ]3 Α Β C

3 � ημ2ημ2ημ2 =>

Α Β C ι ημ2ημ2ημ2 $ 8

PD · PE · PF ι Άρα $ - με την ισότητα να ισχύει

PA · PB · PC 8 μόνο όταν Α = Β = Γ και το Ρ είναι το έγκεντρο του τριγώνου, οπότε το μέγιστο της παράστα-

σης PD · ΡΕ · PF

είναι _!_ •

PA · PB · PC 8 Λύση με τη βοήθεια συναρτήσεων περισσότερων

μεταβλητών, πήραμε από τον κύριο Ανδρή Ιω­

άννη , Πολιτικό Μηχανικό, Αθήνα.

Λύσεις έστειλαν επίσης ο συνάδελφος Αποστο­

λόπουλος Γιώργος - Μεσολόγγι, ο κύριος Ρ. Μπόρης - Δάφνη Ατrικής και ο καθηγητής

Μπόλης Θοδωρής.

ΕΥΚΛΕΙΔΗΣ Β' λθ ' τ.3/66

Page 69: Ευκλειδης Β 59

Ο Ευκλείδης προτείνει •.. Ευκλείδη ... και Διόφαντο

χ2

y2

72. Δίνεται η έλλειψη (c) : - + - = 1 και μια α

2 β

2

εφαπτομένη της που τέμνει τους άξονες χ ' χ

και y 'y στα σημεία Α, Β αντίστοιχα. Να α­

ποδείξετε ότι η ελάχιστη τιμή της απόστα­

σης ΑΒ είναι ίση με το άθροισμα των ημια­

ξόνων της έλλειψης, δηλαδή min(AB) = α + β .

(Επροτάθη από το συνάδελφο Θανάση Κυρια­

κόπουλο, Αθήνα)

Λύση (Από τον ίδιο)

Η εξίσωση της εφαπτομένης (ε) στο σημείο , . χχσ ΥΥο

_ P(x0 , y0 ) ειναι. -2 + -2 - 1 ( 1 ) α β Προφανώς χ0 · y0 � Ο (διαφορετικά η (ε) δεν τέμνει τους άξονες) . Η ευθεία (ε) τέμνει τον άξο-να χ 'χ στο σημεiο Α [ :>) και τον άξονα y 'y

στο σημεiο Β [ Ο,�: ) . Η απόσταση (ΑΒ) δίνεται

ωtό τον τύπο (ΑΒ) = [:: )' + [σ . οπότε

2 α4 β4 (ΑΒ) = 2 + 2 . Το σημείο Ρ ανήκει στην Χο Υο

έλλειψη, οπότε χ2 y2 β2 α2 -f + -f = 1 <=> z = 2 2 , O < lxo l < α α β Υ ο α - Χο

Επομένως 2 α4 2 α2 2 [ α2 β2 ) (ΑΒ) = 2 + β -2--2 = α 2 + 2 2 χ0 α -χ0 χ0 α - χ0

= 2 βχ� + α(α2 - χ� ) . (β + α)χ� - α3 2 2 2 2 , οπότε: χ (α - χ0 ) χ0

d'(x0 ) � O <c> (β + α)χδ � α3 <ο> χ0 �� , αν χ0 > 0 και

d'(x0 ) � Ο <c> (β + α)χ� � α3 <c> χ0 � -� α' α + β

γκροτούμε τους παρακάτω πίνακες μεταβολών. χ � αiβ ο α

d'(x0) - +

d (Χο) � Τ Ε �

χ � αiβ α ο d'(Xo) - +

d (Χο) +οο � / +οο

ΤΕ

Εύκολα πλέον βρίσκουμε ότι και στις δυο περι-( β)2 , , . d α + πτωσεις ισχυει: nun. 2 α

Επομένως η ελάχιστη τιμή της απόστασης είναι: (ΑΒ) = α · (α + β) = α + β α

2η Λύση (Από τους μαθητές του τμήματος Bl Τεχνολογικής Κατεύθυνσης του 3ου Ε.Λ.

Χαϊδαρίου). Υ

ο

Μ( ασυνφ,βημφ) α Α(συνφ •Ο)

Έστω η έλλειψη χ2 y2 - + - = 1 <::> β2χ2 + α2y2 _ α2β2 = Ο α2 β2

χ

Στο σημείο της Μ (ασυνφ, βημφ), φ ε [0,2π) έ­χει εφαπτομένη την ευθεία με εξίσωση:

ΕΥΚΛΕΙΔΗΣ Β ' λθ ' τ.3/67

Page 70: Ευκλειδης Β 59

Ο Ευκλείδης προτείνει ... Ευκλείδη •.• και Διόφαντο

β2χασυνφ + α2yβημφ - α2β2 = Ο Η εφαπτομένη τέμνει τους άξονες στα σημεία Α και

Β με συντεταγμένες Α (-α-,ο) και Β (ο.-β-)

συνφ ημφ

α2 β2 και η απόσταση ΑΒ είναι: ΑΒ = --2- + -2-συν φ ημ φ

Για να είναι min(AB) = α+β αρκεί να αποδει-α2 β2

χθεί ότι --+ -- � α + β με την ισότη­συν2φ ημ2φ

τα να ισχύει σε μια τουλάχιστον περίπτωση . Πράγματι έχουμε:

α2 β2 -- + -- � α2 + 2αβ + β2 συν2φ ημ2φ

α2ημ 2φ + β2συν2φ � � α2ημ 2φσυν2φ + 2αβημ 2φσυν2φ + +β2ημ2φσυν2φ

α2ημ2φ + β2συν2φ _ α2ημ2φσυν2φ _

-2αβημ 2φσυν2φ - β2ημ 2φσυν2φ � Ο α2ημ2φ(l - συν2φ) + β2συν2φ(l - ημ2φ) ­

-2αβημ2φσυν2φ � Ο α2ημ4φ + β2συν4φ - 2αβημ2φσυν2φ � Ο (αημ2φ - βσυν2φ)2 � Ο που ισχύει. Το ίσον ισχύει όταν:

με φ Ε [0, 2π) . Λύσεις επίσης πήραμε από τους συναδέλφους

Γιάννη Σταμματόγιαννη - Δροσιά Απικής,

Ηλιόπουλο Γιάννη - Καλαμάτα, Κούρτη

Χρυσόστομο - Λάρισα και τον κύριο Ρ. Μπόρη

- Δάφνη Απικής, από τον καθηγητή Μπόλη

Θεόδωρο και από τους συναδέλφους Τσοπέλα Ι ωάννη - Αμαλιάδα, Ηλία Κων/νο - Αλιβέρι

και Τσαπακίδης Γιώργος - Αγρίνιο.

τέλος ο κύριος Ανδρής Ιωάννης, Πολιτικός Μηχανικός - Αθήνα, αποδεικνύει επίσης ότι το τμήμα ΑΒ της εφαπτομένης ελάχιστου μήκους, χωρίζεται από το σημείο επαφής με την έλλει­ψη, σε δυο τμήματα από τα οποία το μεγαλύτε­ρο είναι ίσο με το μεγάλο ημιάξονα και το μι-κρό είναι ίσο με το μικρό ημιάξονα.

74. Δίνεται το πολυώνυμο Ρ( κ, λ) = (2κ + 1)8 - 2λ2 - 1 κ, λ e .Ζ .

Να βρείτε τις τιμές του λ, ώστε για οποια­δήποτε τιμή του κ, οι τιμές του πολυώνυ­μου να είναι πολλαπλάσια του 32. (Επροτάθη από τον χημικό Δη μήτριο Καρβε­

λά, Πεύκη)

Λί>ση (Από τον ίδιο) Ο αριθμός 2κ+ 1 είναι περιττός, οπότε το τετρά­γωνό του είναι της μορφής 8μ+ 1 , μ Ε .Ζ . Έτσι έχουμε: (2κ + 1)2 - 1 = 8μ , όπου αν θέ­σουμε 2κ + 1 = α , παίρνουμε: α2 - 1 = 8μ . Ισχύει: (2κ + 1)8 - 1 = α8 - 1 = (α4 + 1)(α2 + 1)(α2 - 1) , όπου οι αριθμοί α4 + 1 και α2 + 1 είναι άρτιοι, οπότε (2κ + 1)8 - 1 = πολ.32 , έστω (2κ + 1)8 - , -1 = 32p , p Ε Ζ . Επομένως: Ρ( κ, λ) = πολ.32 <::::> 2λ2 = πολ.32 <::::>

2λ2 = 32ν <::::> λ2 = 4 2 ν , ν Ε Ζ απ' όπου προκύπτει ότι ο αριθμός ν πρέπει να είναι τετράγωνο ακέραιου, έστω ν = σ2 , σ Ε Ζ , οπότε λ = ±4σ , σ Ε Ζ . Εύκολα πλέον διαπιστώνουμε ότι με λ = ±4σ , προκύπτει ότι Ρ( κ, λ) = πολ.32 Άρα, οι τιμές του πολυώνυμου είναι πολλαπλά­σια του 32, αν και μόνο αν, ο αριθμός λ είναι πολλαπλάσιο του 4. Λύσεις έσrειλαν επίσης ο κύριος Ανδρής Ιωάν­

νης, Πολιτικός Μηχανικός - Αθήνα, ο συνάδελ­

φος Καραβότας Δη μήτρης - Κάτω Αχαία και

ο κύριος Ρ. Μπόρης - Δάφνη Απικής, ο καθη­

γητής Μπόλης Θόδωρος και ο συνάδελφος

Κούρτης Χρυσόστομος, Λάρισα.

75. Να βρείτε όλα τα ζεύγη των θετικών ακέ­ραιων χ, y ώστε το καθένα από τα αθροί­σματα χ2 + 3y και y2 + 3χ να είναι τέλειο τετράγωνο. ( Επροτάθη από το συνάδελφο Α­

ντώνη Ιωαννίδη , Λάρισα)

Λί>ση (Από τον ίδιο) Έστω ότι: χ2 + 3y = α2 . Τότε 3y = α2 - χ2

απ' όπου προκύπτει ότι α> χ, οπότε μπορούμε να γράψουμε: χ2 + 3y = (χ + n)2 , η Ε Ν* .

ΕΥΚΛΕΙΔΗΣ Β ' λθ ' τ.3/68

Page 71: Ευκλειδης Β 59

Ο Ευκλείδης προτείνει ... Ευκλείδη ... και Διόφαντο

Επομένως: χ2 + 3y = χ2 + 2χη+η2 � � -2χη + 3y = η2 ( 1 ) Ομοίως: y2 + 3χ = ( y + m)2 � y2 + 3χ = = y2 + 2ym + m2 � 3χ - 2ym = m2 , m ε Ν* (2) Οι εξισώσεις ( 1 ), (2) σχηματίζουν σύστημα με: D = 4mn - 9 :;t: Ο , διότι m, η ε Ν* , Dx = -2mn2 - 3m2 , Dy = -2ηm2 - 3η2

, 2mn2 + 3m2 2ηm + 3η2 οποτε: χ = ----- και y = 9 - 4mn 9 - mn Ισχύει: χ, y ε Ν* , οπότε 9 - 4mn > Ο � mη < 2 4 Άρα (m, η) ε { ( l , 1 ) ,( 1 , 2), (2, 1 )} • Αν m = η = 1 , τότε Χ = y = 1 και

χ 2 + 3y = y2 + 3χ = 1 + 3 = 4 = 22 • Αν m=l και η=2 , τότε χ= 1 1 , y= 16 και

χ2 + 3y = 1 12 + 3 · 1 6 = 1 2 1 + 48 = 169 = 1 32 y2 + 3χ = 1 62 + 3 · 1 1 = 256 + 33 = 289 = 172

• Αν m = 2 και η=1 , τότε χ=16, y= 1 1 και x2 + 3y = l 72 , y2 + 3x = 132

Λύσεις έστει�αν επίσης ο συνάδελφος Αποστολό­

πουί.ο.; Γιι:)ργος - Μεσολόγγι, ο κύριος Ρ. Μπό­

ρη.; - Δάφνη Απικής, ο καθηγητής Μπόλης Θο­

δι·ψtj .; και οι συνάδελφοι Κούρτης Χρυσόστο­

μο.; - Λάρισα και Ν. Καραγκιόλη - Σέρρες. 77 . Αν ισχύει (x2 +y2 )2 + 4x2y2 = 2(x + y) ,

x,y ε R να βρείτε τη μέγιστη τιμή της δια-φοράς x-y ( Επροτάθη από το συνάδελφο

.\ρtj στο �εμιρτζόγλου, Δράμα).

. \ί1ση (Από τον ίδιο) Είναι: (χ2 + y2 )2 + 4x2y2 = 2(χ + y) � χ4 + 6x2y2 + y4 = 2(χ + y) � � (χ - y)4 + (χ + y)4 = 4(χ + y) � � (χ - y)4 = 4(χ + y) - (χ + y)4 (προφανώς χ + y � Ο ) Θεωρούμε τη συνάρτηση f (t) = 4t - t4 , t � Ο . Αναζητούμε τα ακρότατα της συνάρτησης αυτης. Έχουμε: f '(t) = 4 - 4t3 = 4(1 - t3 ) , οπότε f '(t) > O για κάθε t ε (0, 1) και f '(t) < O για κάθε t > 1 . Άρα η συνάρτηση παρουσιάζει ολι­κό μέγιστο για t = 1 , το f (1) = 3 οπότε 4t - t4 :::;; 3 για κάθε t � Ο . Είναι: (χ - y)4 = 4(χ + y) - (χ + y)4 :::;; 3 , οπότε

(x - y)4 :::; 3 � !x - y! :::; � Επειδή x - y :::; ! x - y! � x - y :::; � Άρα η μέγιστη τιμή της διαφοράς χ - y είναι � και προκύπτει μόνο όταν χ = �( � + 1) και y =�(1 - �) -

Λύσεις έστειλαν επίσης οι συνάδελφοι Αποστολό­

πουλος Γιώργος - Μεσολόγγι, Κούρτης Χρυσό­

στομος - Λάρισα, ο κύριος Ρ. Μπόρης - Δάφνη

Απικής και ο καθηγητής Μ πόλης Θόδωρος.

78. Να βρείτε όλα τα ορθογώνια τρίγωνα των οποίων τα μήκη των πλευρών είναι ακέραι­οι αριθμοί και η περίμετροι είναι αριθμητι­κά ίση με το εμβαδόν τους ( Επροτάθη από

τον συνάδελφο Σωτήρη Σκοτίδα, Καρδίτσα).

Λ \Jση (Από τον ίδιο) Α ν α είναι η υποτείνουσα του τριγώνου, τότε έχουμε: α + β + γ = β · γ � α = β · γ - β - γ � 2 2 � α = β · γ - (β + γ) � 2

β2 2 � α2 = -γ- - βγ(β + γ) + β2 + γ2 + 2βγ � 4 β2 2

� 2βγ - βγ(β + γ) + -γ- = 0 � 4 � 2 - β - γ + βγ = 0 � 8 - 4β - 4γ + βγ = 0 � 4 � 4β - βγ + 4γ - 16 = -8 � � β( 4 - γ) - 4( γ - 4) = -8 � (β - 4 )(γ - 4) = 8 απ' όπου προκύπτει ότι β - 4 ε { 1, - 1, 2, -2, 4, -4, 8, -8} οπότε β = 2, 3, 5 , 6, 8, 1 2 Εύκολα τώρα βρίσκουμε ότι υπάρχουν δυο τέτοια τρίγωνα με μήκη πλευρών (6, 8, 10) και (5, 12, 13). Λύσεις έστειλαν επίσης ο κύριος Ανδρής Ι ωάν­

νης, Πολιτικός Μηχανικός, Αθήνα, οι συνάδελ­

φοι Κούρτης Χρυσόστομος, Λάρισα, Καραβό­

τας Δη μήτρης - Κάτω Αχαί'α, ο κύριος Ρ. Μπόρης - Δάφνη Απικής, ο καθηγητής Μπό­

λης Θόδωρος, οι συνάδελφοι Τσοπέλας Ιωάν­

νης - Αμαλιάδα, Ηλιόπουλος Γιάννης - Καλα­

μάτα, Τσαπακίδης Γιώργος - Αγρίνιο και ο

κύριος Λουκάς Χ υτή ρης - Αθήνα.

ΕΥΚΛΕΙΔΗΣ Β' λθ ' τ.3/69

Page 72: Ευκλειδης Β 59

π I I

Επιμέλεια:Γιάννης Στρατής - Βαyytλης Ευσταθίου

ΑΚΟΜΑ ΜΙΑ ΓΕΝΙΚΕΥΣΗ "ΔΙΟΦΑΝΤΙΚΗ ΛΥΣΗ ΤΗΣ ΕΞΙΣΩΣΗΣ χ2 + y2= z2"

Σ το βιβλίο «Μαθηματικά Β ' Λυκείου θετι­κής και Τεχνολογικής Κατεύθυνσης στο Κεφάλαιο Θεωρία των αριθμών δίνεται ως

γνωστή Διοφαντική λύση της εξίσωσης χ2 + y2 = z2 η εξής:

χ = μ 2 - λ2 ' Υ = 2λμ ' z = μ 2 + λ2 . Με αφορμή όμως το 4° θέμα του Μαθηματικού

Διαγωνισμού «Ευκλείδης», της Β ' Λυκείου, κατέλη­ξα στο συμπέρασμα ότι μόνο τα οξυγώνια τρίγωνα μετατρέπονται σε ορθογώνια τρίγωνα όταν και οι τρεις πλευρές ελαττωθούν κατά το ίδω μήκος.

Ένα βήμα παρακάτω ! ! ! Ας βρούμε όλα τα οξυ­γώνια τρίγωνα με πλευρές α, β, γ όπου α :::;; β :::;; γ , τα οποία μετατρέπονται σε ορθογώνια τρίγωνα με πλευρές χ < y < z όπου x, y, z Ε Ν , δηλαδή λύση της εξίσωσης χ 2 + y2 = z2 ακεραία.

Συμπέρασμα:

{χ = ν(2λ + 1) y = 2λν(λ + 1) z = ν[2λ(λ + 1) + 1]

η οποία είναι μια νέα Γενικευμένη ακεραία λύση. ΑΠΟΔΕΙΞΗ

Θέμα 4° Β' Λυκείου Δίνεται τρίγωνο ΑΒΓ με πλευρές

ΒΓ = α < ΓΑ = β < ΑΒ = γ . Να εξετάσετε αν είναι δυνατόν να ελαττωθούν και οι τρεις πλευρές κατά το ίδιο μήκος, έτσι ώστε να γίνουν πλευρές ορθο­γωνίου τριγώνου. ΛΥΣΗ

Έστω τρίγωνο ΑΒΓ με πλευρές α :::;; β :::;; γ και χ0 Ε (Ο, α) , διότι έχουμε μήκη πλευρών, τότε:

του Νίκου Στολάκη

(α - χσ )2 + (β - χο )2 = (γ - χο )2 �

2 2 2 β2 2β 2 2 2 2 α - αχ0 + χ0 + - χ0 + χ0 = γ - γχ0 + χ0 ή χ� - 2χ0 (α + β - γ) + α2 + β2 - γ2 = 0 και

2( α+β-γ) ±�4( α+β-γ)2 -4{ α2 +β2 -γ2 ) (χο)ι,

2 =

2 (Χ ο λ 2 = α + β - γ ±

ή �· --��----------�--�� ±�α2 + β2 + γ2 2αβ - 2αγ - 2βγ - α2 - β2 + γ2

ή (χολ,2 = α +β - γ ± �2γ2 + 2αβ - 2αγ - 2βγ .

Τέλος (χ0 )1,2

= α + β - γ ± �2(γ - α)(γ - β) και

χ0 Ε (Ο, α) . Άρα έχουμε δύο περιπτώσεις:

α) 0 < α + β - γ + �2(γ - α)(γ - β) < α

β) 0 < α + β - γ - �2(γ - α)(γ - β) < α .

α) 0 < α + β - γ + �2(γ - α)(γ - β) είναι σαφές διότι α + β > γ τώρα α + β - γ + �2(γ - α)(γ - β) < α ή

�2(γ - α)(γ - β) < γ - β . Επειδή και τα δύο μέλη είναι θετικά

=> 2(γ - α)(γ - β) < (γ - β)2 ή 2(γ - α) < γ - β ή

γ + β < 2α , άτοπο διότι β � α} άρα β + γ � 2α . γ � α

Άρα η α) απορρίπτεται. β) α + β - γ - �2(γ - α)(γ - β) < α ή

-�2(γ - α)(γ - β) < γ - β είναι σαφές διότι γ - β > Ο . Τώρα από 0 < α + β - γ -�2(γ - α)(γ - β) ή

ΕΥΚΛΕΙΔΗΣ Β' λθ' τ.3ΠΟ

Page 73: Ευκλειδης Β 59

Το Βήμα του Ευκλείδη

�2(γ - α)(γ - β) < α + β - γ και τα δύο μέλη είναι θετικά. Άρα 2(γ - α)(γ - β) < (α + β - γ)2 ή

2(γ2 -αγ-βy+αβ) < α2 + β2 +γ2 + 2αβ - 2αγ-2βy Τέλος γ2 < α2 + β2 . Άρα το τρίγωνο ΑΒΓ είναι

οξυγώνιο. Συνεπώς μόνο τα οξυγώνια τρίγωνα με­τατρέπονται σε ορθογώνια όταν και οι τρεις πλευ­ρές τους ελαττωθούν κατά το ίδιο μήκος.

χ0 = α + β - γ -�2(γ - α)(γ - β) και οι πλευρές του ορθογωνίου τριγώνου θα είναι:

( 1){� = ;=:: z = γ - χο

Το βήμα παρακάτω !

Ας βρούμε όλα τα οξυγώνια τρίγωνα με πλευ­ρές α $ β $ γ τα οποία μετατρέπονται σε ορθογώ-νια τρίγωνα με πλευρές χ, y, z όπου x, y, z ε Ν και χ2 + y2 = z2 . Αφού x, y, z ε Ν από ( 1 ) θα έχουμε υποχρεωτικά: 2{ γ - α) { γ - β) = μ2 .

Άρα αν 2 (γ - α) = κ τότε γ - β = κλ2 ή αν (γ - β) · 2 = κ τότε γ - α = κλ2 , που είναι το ίδιο διότι χ 2 + y2 = y2 + χ 2 .

Έστω 2 (γ - α) = κ και γ . β = κλ2

' κ β κλ2 η α = γ -- και = γ - . 2 Τότε

χ0 = γ -� + γ - κλ2 - γ - κλ = γ - (; + κλ2 + κλ) .

Συνεπώς ο ( 1 ) γίνεται: χ = α- χ0 = γ -� -γ +� + κλ2 + κλ = κλ(λ+ l) 2 2

2 κ 2 κ κ(2λ+ 1) (2) y = β - χ0 = γ-κλ -γ +2" + κλ + κλ =2" + κλ = --'--2-'-

κ κ (2λ2 + 2λ+ 1) z = γ - χ0 = γ - κ+- + � + κλ = ----"-----'-2 2

Τώρα από χ = κλ2 + κλ και z = κλ2 + κλ + κ , 2 θ ' Κ ' lM ' 2 α εχουμε z = χ +- , οπου χ, z ε 1'1 αρα κ = ν . 2

Τότε η (2) γίνεται:

!χ = 2λν(λ + 1) (3) y = ν {2λ + 1)

z = ν ( 2� + 2λ + 1) με λ, ν ε Ν .

Επειδή έχουμε χ 2 + y2 = z2 , άνετα μπορούμε να πάρουμε λ, ν ε Ν και έτσι επιτυγχάνουμε τη ΓΕΝΙΚΕΥΜΕΝΗ ακεραία Λ ΥΣΗ της εξίσωσης. Π Ο Ρ ΙΣΜΑ I

Οι αρχικές λύσεις της εξίσωσης χ 2 + y2 = z2 δίνονται από τους τύπους {χ = 2λ{λ + 1)

(4) y = 2λ + 1 z = 2λ{λ + 1) + 1

Πράγματι από την (3) έχουμε: [ 2λν (λ + 1)]2 + [ ν (2λ + 1)]2 = [ν ( 2λ2 + 2λ + 1 )Τ

ή ν2 [2λ{λ + 1)]2 + ν2 (2λ + 1)2 = ν2 (2λ2 + 2λ + 1)2 ,

για ν :;e Ο ή [2λ{λ + 1)]2 + {2λ + 1)2 = (2λ2 + 2λ + 1)2 που εί­ναι η (4)

Π Ο ΡΙ Σ ΜΑ 2

Σε κάθε ορθογώνιο τρίγωνο η μεγαλύτερη κά­θετη πλευρά και η υποτείνουσα είναι διαδοχικές και η κάθετη πλευρά είναι άρτια όταν χ, y, z ε Ν και (x , y, z) = l . Απόδειξη

Από (4) έχουμε z - χ = ι ' άρα Χ, z - διαδοχικοί και από χ = 2[λ(λ + ι)] έχουμε χ - άρτιο.

Συμπέρασμα

Η ( 4) είναι ισοδύναμη με τη λύση του βιβλίου. Α ν στην ( 4) θέσουμε λ + 1 = μ τότε θα έχουμε: χ = 2λμ

y = 2λ + l = λ2 + 2λ + 1 - λ2 = (λ + 1)2 - λ2 = μ2 - λ2 z = 2λ{λ + 1) + 1 = 2λ2 + 2λ + 1 = λ2 + 2λ + l + λ2 = = (λ + 1 )2 + λ2 = μ 2 + λ2 .

ΕΥΚΛΕΙΔΗΣ Β ' λθ ' τ.3Π1

Page 74: Ευκλειδης Β 59

τιjλιι TfJU

lιf 11 Β IJT ιj Επιμέλεια: Αντώνης Κυριακόπουλος - Γιάννης Στρατής

ΜΑΣ ΓΡΑΦΕΤΕ . . . ΣΑΣ ΑΠΑΝΤΆΜΕ

• Απάντηση στο γράμμα του μαθητή Τάσου Πασχαλίδη (2° Ενιαίο Λύκειο Κοζάνης).

Αγαπητέ Τάσο. α) Στην εικασία του Γκολντμπάχ δεν έχει δοθεί

απάντηση μέχρι σήμερα. 1

β) Παράγωγος της συνάρτησης f (x) = � , όπου ν Ε Ν* και ω Ε JR •

Διακρίνονται δύο περιπτώσεις: ί) Έστω ότι το ω είναι ένας ακέραιος άρτιος

αριθμός. Στην περίπτωση αυτή, η συνάρτηση f είναι ορισμένη και παραγωγίσιμη στο JR* . Έχουμε,

' ΠΙ> * τοτε, στο .1!'!. : I

f (x ) = (x(l) ) ν , οπότε:

I ν� f ' (χ) = -.!.(Χω )- � -Ι (Χω )' = -.!.ψιω )- --;-ωχω-1 =

ν ν

ν+ I ω ι ι-(1)- ω-1 . = -- Χ ν · Χ = ν

ω --χ ν ω+ ν ν

ω+ ν ω - --(-χ) ν ' ν

αν χ > Ο

αν χ < Ο

ίί) Έστω ότι ω Ε (1R - {2κ / κ Ε Ζ} ) . Τότε, η συνάρτηση f είναι ορισμένη και παραγωγίσιμη στο (Ο,+οο) . Έχουμε, τότε, στο (Ο,+οο) :

(1) f (x) = χ ν ' οπότε:

ω - � -1 ω - ω+ν f '(x) = --Χ ν = - - Χ ν

ν ν

Βλέπουμε ότι σε κάθε περίπτωση που η f είναι ορισμένη και παραγωγίσιμη, η παράγωγός της εί­ναι:

ν+l , ω ι ι-ω·- ω Ι f (χ) = - - Χ ν · Χ - . ν

Λοιπόν Τάσο. Όταν ζητάς την παράγωγο μιας συνάρτησης, πρώτα θα βρίσκεις το σύνολο ορισμού της, μετά θα βρίσκεις που είναι παραγω­γίσιμη και μετά θα εφαρμόζεις τους κανόνες παρα­γώγισης. Την παράγωγο της παραπάνω συνάρτη­σης δεν είναι ανάγκη να τη θυμάσαι απ' έξω. • Απάντηση στο γράμμα του μαθητή Διιμι1 -

τρη Σπυρ()ποιιλου (Λύκειο Νέας Πεντέλης). Αγαπητέ Δημήτρη. Σε ευχαριστούμε για τα

καλά σου λόγια σχετικά με το περιοδικό μας. Οι παρατηρήσεις σου θα ληφθούν υπόψη. Τις ασκή­σεις που μας έστειλες πρέπει να τις επεξεργαστείς περισσότερο (εκφωνήσεις και λύσεις) .

Λ ΥΣΕΙΣ ΑΣΚΗΣΕΩΝ

33. θεωρούμε, ένα τρίγωνο ΑΒΓ με . Γ = 2Β , ένα εσωτερικό του σημείο Μ και υποθέτουμε ότι ΑΜ = ΑΓ και ΜΒ = ΜΓ . Να δείξετε

- l Λ ότι: ΜΑΒ = -Α •

3 ΕΥΚΛΕΙΔΗΣ Β ' λθ ' τ.3Π2

Page 75: Ευκλειδης Β 59

(Επροτάθη από το μαθητή Γιάννη Σούμπλη,

Αρσάκειο Ψυχικού, Αθήνα). Λύση (από τον ίδιο μαθητή) Θεωρούμε τον περιγεγραμμένο κύκλο στο τρί­

γωνο ΑΒΓ και τη χορδή του ΑΑΊ/ΒΓ. Επειδή ABf = ΑΥΒ και Arn = 2ABr , έχουμε:

-ΑΓΒ = 2Α'ΓΒ . Άρα, η Γ Α' είναι διχοτόμος της γωνίας Γ. 'Ετσι, έχουμε Ar=BA'=i\'A . Άρα Ar = 1\'Α και συνεπώς ΑΓ=ΑΑ' . Εξάλλου, επει­δή ΜΒ = ΜΓ , ΒΑ'=Γ Α και ΜΒΑ' = MrA (δια­φορές ίσων γωνιών) , τα τρίγωνα Α 'ΒΜ και ΑΓΜ είναι ίσα και άρα ΜΑ' = ΜΑ. Έτσι, έχουμε:

ΜΑ' = ΜΑ = ΑΓ = ΑΑ'

Άρα, το τρίγωνο ΜΑΑ' είναι ισόπλευρο. Έ­χουμε, λοιπόν:

ΜΑΒ=ΜΑΑ' - w' = 6οο - Β . ( 1 ) . Εξάλλου, έχουμε: Α + Β + Γ = 1 80° => Α + Β + 2Β = 1 80° =>

λ

1 80° - 3Β = Α => 60° - Β = Α (2) . 3 Από τις ( 1 ) και (2) έπεται ότι: ΜΑΒ=_!..Α . 3

• Λύσεις έστειλαν και οι μαθητές: Ν ίκος Μαρα­

βίτσας, Στέλιος Ιωάννου, Ν ίκος Ζήσης, Νι­

κολέτα Φεσσά.

34. Με α, β και γ πραγματικούς αριθμούς και ν Ε Ν* να δείξετε ότι, αν η ισότητα:

(α + β + γ)2ν+1 = α2ν+1 + β2ν+1 + γ2ν+1 (l)

ισχύει για ν = 1 , τότε ισχύει και για κάθε ν Ε Ν* . (Επροτάθη από τη μαθήτρια Εβίτη Αντωνί­

ου, Ιταλική Σχολή Αθηνών) Λύση (από το μαθητή Νίκο Μαραβίτσα. 6°

Λύκειο Περιστερίου)

Έστω ότι η ισότητα (Ι) ισχ6ει για ν = 1 , δηλαδή ότι:

(α + β + γ)3 = α3 + β3 + γ3 => [(α + β) + γ]3 = α3 + β3 + γ3 => (α + β)3 + 3(α + β)2 γ + 3(α + β)γ2 + γ3 = = (α + β)3 - 3αβ(α + β) + γ3 => 3(α + β) [ (α + β)γ + γ2 + αβ ] = 0 => (α + β)(αγ + βγ + γ2 + αβ) = Ο => (α + β) [ α(β + γ) + γ(β + γ)] = Ο => (α + β )(β + γ)( γ + α) = Ο => ( α = -β ή β = -γ ή γ = -α )

Έστω ότι α = -β . Για να δείξουμε την ( 1 ) (με ν Ε Ν* ), αρκεί να δείξουμε ότι:

(-β + β + γ)2ν+Ι = ( -β)2ν+Ι + β2ν+l + γ2ν+Ι ' αρκεί 2ν+Ι 2ν+Ι , 'Ο ζ , γ = γ , ισχυει. μοια εργα ομαστε αν

β = -γ ή γ = -α . • Λύσεις έστειλαν και οι μαθητές: Νίκος Ζήσης,

Γιάννης Σούμπλης, Μαίρη ΓεωργιοποίJJ.ου,

Νικολέτα Φεσσά.

35. Να βρείτε τους ακέραιους θετικούς αριθ­μούς χ και y, για τους οποίους ισχύει:

2x+y+3-FY - 5� - 2../Υ - 3 = Ο (1)

(Επροτάθη από τη μαθήτρια Νικολέτα Φεσσά,

Λύκειο Νέας Χαλκηδόνας) Λύση (από το μαθητή Γ ιάννη Σοίφπλη . Αρσά­

κειο Ψυχικού, Αθήνα) Έχουμε: (1) � (2χ + 2_[r:Y - 6..Γχ) + (.{r:Y + y - 3JY) +

+( .rx + JY - 3) = ο � 2..Γχ(..Γχ + JY - 3) + JΥ(.,Γχ + JY - 3) +

+(..Γχ + JY - 3) = 0 � ( .rx + JY -3)(2..Γχ + JY + 1) = ο � ..Γχ + JY - 3 = Ο (γιατί χ>Ο και y>O) � .rx + JY = 3 � JY = 3 - ..Γχ � {3 - ..Γχ > 0 {χ < 9 �

y = (3 - ..Γχ)2 �

y = 9 - 6..Γχ + χ Με χ = 1 , βρίσκουμε y = 4 . Με χ = 2 , βρίσκουμε: y = 1 1 - 6.J2

ΕΥΚΛΕΙΔΗΣ Β ' λθ ' τ.3/73

Page 76: Ευκλειδης Β 59

• • ν • •ι•w•ι • v v .1.9 a u.v ι ι • Ί

.· �2 1 1 - y , ( ' , ) Ό � ν L

= -6- , ατοπο αρρητος = ρητος . -μοια, με χ = 3 , 5 , 6, 7, 8 φθάνουμε σε άτοπο. Με χ = 4, βρίσκουμε y = 1 . Άρα, οι ζητούμενοι αριθμοί είναι (χ = 1 . y = 4) και (χ = 4, y = 1 ) . • Λύσεις έστειλαν και οι μαθητές: Εβίτα Αντω­

νίου, Σπύρος Λεονάρδος, Ν ίκος Αλεξίου, Ν ί­

κος Μαραβίτσας.

36. Έστω ένας φυσικός αριθμός με κ ψηφία: ιοκ-ι ν = Υκ-ιΥκ-2 • .. Υ2ΥιΥο = Υ κ-ι · +

οκ-2 02 10 +Υ κ-2 · 1 + • · • + Υ 2 · 1 + Υ ι · + Υ ο , όπου Υ κ-ι , Υκ.2 , . . . , y2 , yι ,Yo είναι τα ψη-φία του, δηλαδή αριθμοί του συνόλου: {0, 1,

2, • • • , 9}. Να δείξετε ότι οι δύο διαιρέσεις ν:4 και (2y ι + y 0 ) : 4 έχουν το ίδιο υπόλοιπο. Εφαρ�ιογή : Να βρείτε τον αριθμό:

z = i885307856725987 + i795055725176218

(όπου i η φανταστική μονάδα) (Επροτάθη από το μαθητή Βαγγέλη Π ρωτο­

παπαδάκη, 3° Λύκειο Χανίων Κρήτης) . Λύση (από τον ίδιο μαθητή)

Επειδή καθένας από τους αριθμούς: 1 02 , 1 03 , 1 04 . . . είναι πολλαπλάσιο του 4, έπεται ότι: ν = πολ4 + 10y1 + y0 = πολ4 + 8y1 + 2y1 + y0 �

ν = πολ4 + (2y1 + y0 ) . ( 1 ) Έστω π το πηλίκο και υ το υπόλοιπο της διαί­

ρεσης (2y1 + y0 ) : 4 , οπότε: 2y1 + y0 = 4π + υ , Ο � υ < 4 . Έτσι, από την ( 1 ), έχουμε:

ν = πολ4 + 4π + υ=πολ4+υ � ν = πολ4 + υ (2) . Επειδή Ο � υ < 4 , η (2) είναι η ισότητα της

διαίρεσης ν:4. Άρα και το υπόλοιπο της διαίρεσης αυτής είναι υ. Εφαρμογή. Αν ν είναι ένας φυσικός αριθμός και υ είναι το υπόλοιπο της διαίρεσης ν:4 (υ=Ο, 1 , 2, 3), τότε είναι γνωστό ότι i ν = i υ .

Στον εκθέτη του πρώτου προσθετέου είναι: Υ ο = 7 και Υ ι = 8 . Το υπόλοιπο της διαίρεσης του αριθμού 2y1 + y0 = 2 · 8 + 7 = 23 δια του 4 είναι 3 .

Άρα, σύμφωνα με την παραπάνω πρόταση, ο πρώ­τος προσθετέος του αθροίσματος είναι ίσος με i3 =-i .

Στον εκθέτη του δεύτερου προσθετέου είναι: Υ ο = 8 και y1 = 1 . Το υπόλοιπο της διαίρεσης του αριθμού 2y1 + y0 = 2 · 1 + 8 = 10 δια του 4 είναι 2. Άρα, ο δεύτερος προσθετέος του αθροίσματος εί­ναι ίσος με i2 =-1 .

Συνεπώς: z=-i+1 . • Λύσεις έστειλαν και οι μαθητές: Σπύρος Λεο­

νάρδος, Κων/νος Παπαγεωργίου, Γεώργιος

Σοίιμπλης, Αντώ\'ης Μιστριώτης.

37. Να δείξετε ότι η συνάρτηση:

f(x) = αχ4 + βχ2 + γχ + δ ,

όπου α, β, γ, δ ε 1R με α * Ο και α+β=- �δ , 2

έχει μία τουλάχιστο ρίζα στο διάστημα [-1, 1].

(Επροτάθη από το μαθητή Γεώργιο Σούμπλη .

Αρσάκειο Ψυχικού, Αθήνα) Λύση (από το μαθητή Πα\'αγιώτη Για\'νούλη,

3° Λύκειο Χίου) Η συνάρτηση f είναι ορισμένη και συνεχής στο

1R . Έχουμε: f (Ο) = δ , f (1) = α + β + γ + δ και f ( - 1) = α + β - γ + δ . Προσθέτοντας κατά μέλη βρίσκουμε: f (O) + f (1) + f (-1) = 2α + 2β + 3δ = 2(α + β) + 3δ =

=> f (O) + f (l) + f (-1) = Ο . ( 1 ) Αν f (O) = Ο , τότε η f έχει ρίζα τον αριθμό Ο.

Αν f (l) = Ο έχει ρίζα το 1 και αν f (-1) = Ο έχει ρίζα το -1 .

Έστω ότι f(O) * Ο , f( l ) * Ο και f(- 1 ) :;t; Ο . Τό-τε, από την ( 1) έπεται ότι δύο τουλάχιστο από τους αριθμούς αυτούς είναι ετερόσημοι (γιατί;) . Έστω ότι f(O) · f( l )<O . Τότε, από το θεώρημα του Bol-zano, για την f στο διάστημα [0, 1 ], έπεται ότι η f έχει μία τουλάχιστο ρίζα στο (0, 1 ) . Όμοια, αν f( 1 )f( -1 ) < Ο η f έχει μία τουλάχιστο ρίζα στο (-1 , 1 ) και αν� f(-1 )f(0)<0 έχει μία τουλάχιστο ρί-ζα το (-1 , 0).

Άρα, σε κάθε περίπτωση, η f έχει μία τουλάχι­στο ρίζα στο διάστημα [-1 , 1 ] .

ΕΥΚΛΕΙΔΗΣ Β ' λθ ' τ.3Π4

Page 77: Ευκλειδης Β 59

• Λύσεις έστειλαν και οι μαθητές: Κων/νος Πα­

παγεωργίου, Σπύρος Λεονάρδος, Αντώνης

Μιστριώτης, Αντώνης Μακρής.

38. Μία συνάρτηση f είναι ορισμένη και συνε­

χής στο διάστημα [0,5]. Να δείξετε ότι υ­πάρχει χ0 Ε (0,5) με:

8f(x0 ) = 3f(l) + 4f(3) + f(4) .

(Επροτάθη από το μαθητή Κων/νο Π απαγε­

ωργίου, Εκπαιδευτήρια «0 Πλάτων», Αθήνα). Λύση (από το μαθητή Αντώνη Μιστριώτη,

Γερμανική Σχολή Αθηνών) Έστω ότι f(4) :::; f( l ) :::; f (3) (ανάλογα εργαζό-

μαστε σε κάθε άλλη περίπτωση). Έχουμε: 3f(l) + 4f (3) + f(4) 2: 3f (4) + 4f (4) + f (4) = 8f (4)

και 3f (l) + 4f (3) + f ( 4) :::; 3f (3) + 4f (3) + f (3) = 8f (3)

Έτm, έχουμε: f(4) ::; 3f (1) + 4f (3) + f (4) ::; f (3) . 8

- 'Εστω ότι: f(4) = 3f (1) + 4f (3) + f(4) . Τότε 8 χ0 = 4 .

- 'Εστω ότι: f(3) = 3f (l) + 4f (3) + f(4) . Τότε 8 χ0 = 3 . 'Ε , �(4) 3f(1) + 4f (3) + f (4) f (3) - στω οτι: 1 ' < < . 8

Κολλέγιο Αθηνών). Λύση (από το μαθητή Κων/νο Παπαγεωργί­

ου, Εκπαιδευτήρια «0 Πλάτων», Αθήνα) Έστω ότι μία συνάρτηση f πληροί τις δοσμένες

συνθήκες. Από την ( 1 ) έχουμε για κάθε χ Ε � :

f 2 (x) - 2xf (x) + x2 = χ2 + ex =>

Θεωρούμε τη συνάρτηση : g(x) = f (x) - χ .

(2)

(3) Η g είναι ορισμένη και συνεχής στο � . Έχου­

με g( l ) = f (1) - 1 > Ο . Επίσης από τη (2), έχουμε για κάθε χ Ε � .

(4) Από την ( 4) έπεται ότι: g( χ) * Ο , για κάθε

χ Ε � και άρα η g έχει σταθερό πρόσημο στο � και επειδή g(l) > Ο, έπεται ότι: g(x) > Ο, για κάθε χ Ε � . Έτσι από την (4) έπεται ότι:

g(x) = �χ2 + e2 για κάθε χ Ε � . Άρα από την (3) έχουμε, για κάθε χ Ε � :

f (x) = x + �x2 + e2 • Όπως βρίσκουμε εύκολα, η συνάρτηση αυτή

πληροί τις δοσμένες συνθήκες και άρα είναι η μο­ναδική ζητούμενη .

Τότε, σύμφωνα με το θεώρημα ενδιαμέσων τι- • Λύσεις έστειλαν και οι μαθητές: Αντώνης

μών για την f στο διάστημα [3, 4] , υπάρχει Μιστριώτης, Γεώργιος Σούμπλης, Αντώνης

Χο Ε (3 ,4) με: Μακρής, Νίκος Αλεξίου, Νίκος Μαραβί-

f (xo ) = 3f (l) + 4f (3) + f (4) . 8 Άρα, σε κάθε περίπτωση, υπάρχει χ0 Ε (0,5)

με: 8f (x0 ) = 3f (l) + 4f (3) + f(4) . • Λύσεις έστειλαν και οι μαθητές: Σπί>ρος Λεο­

νάρδος, Γεώργιος Σούμπλης, Αντώ\•ης Μ α­

κρής, Νίκος Αλεξίου.

39. Να βρείτε τις συνεχείς συναρτήσεις f : � -+ 1R , για τις οποίες ισχύει f(l) > 1 και

f2

(x) - 2xf(x) - ex = Ο , για κάθε χ Ε 1R . (1)

(Επροτάθη από το μαθητή Σπύρο Λεονάρδο,

τσα ς.

ΑΣΚΗΣΕΙΣ ΓΙΑ Λ ΥΣΗ

40. Έστω ένα ορθογώνιο τρίγωνο ΑΒΓ (Α = 90° ) με ΑΒ > ΑΓ. Στην πλευρά του ΒΑ παίρνουμε τμήμα ΒΔ =ΑΓ. Ονομάζουμε Ε το μέσο του τμήματος ΔΑ και Ζ το μέσο της υποτείνουσας ΒΓ. Να βρείτε το μέτρο της γωνίας ΒΕΖ .

(Προτείνεται από τη μαθήτρια Νικολέτα Φεσσά,

Λύκειο Νέας Χαλκηδόνας) .

ΕΥΚΛΕΙΔΗΣ Β ' λθ ' τ.3Π5

Page 78: Ευκλειδης Β 59

•• " " • ι• "' • ι " " "' .ι. • • - v • ι • • ι 4 1 . Έστω ένα ορθογώνιο τρίγωνο ΑΒΓ

(Α = 90° ) . Στο εσωτερικό της γωνίας Β φέρ---- l A νου με μία ημιευθεία Β χ με ΑΒχ = -Β . Η Β χ 3

τέμνει την πλευρά ΑΓ στο Δ και την παράλ­ληλο από το Γ προς την ΑΒ, στο Ε. Να δείξε­τε ότι ΔΕ =2ΒΓ.

(Προτείνεται από τη μαθήτρια Εβίτα Αντωνίου,

Ιταλική Σχολή Αθηνών).

42. Έστω η εξίσωση : αχ 2 + β χ + γ = Ο , όπου αγ * Ο και lα l + lγ l < lβ l . Να δείξετε ότι οι ρί­ζες της εξίσωσης αυτής είναι πραγματικές και άνισες. Τις ονομάζουμε χ 1 και χ2• Στη συνέ­χεια να δείξετε ότι οι αριθμοί χ 1 και χ2 δεν εί­ναι δυνατόν και οι δύο να είναι ακέραιοι.

(Προτείνεται από τον μαθητή Γιάννη Σούμπλη,

Αρσάκειο Ψυχικού, Αθήνα) .

43. Οι γωνίες α, β και γ ανήκουν στο διάστημα (0, 1 80°) και έχουν άθροισμα 360° . Να δείξετε ότι:

1 + συνα + συνβ + συνγ < Ο .

(Προτείνεται από τον μαθητή Βασίλη Νάκο, 3° Λύκειο Ν. Ηρακλείου Αττικής) .

44. Θεωρούμε ένα κύκλο C, μία διάμετρό του ΑΒ και μία χορδή του ΓΡ l_ ΑΒ , η οποία τέμνει την ΑΒ σ' ένα σημείο Λ. Παίρνουμε ένα ση­μείο Κ του τμήματος Γ Λ και φέρνουμε τις ευ­θείες ΑΚ και ΒΚ, οι οποίες τέμνουν τον C στα σημείο Μ και Ν, αντιστοίχως. Η ευθεία ΝΛ τέμνει την ΑΜ στο σημείο Π. Φέρνουμε και τα τμήματα ΜΝ και ΜΛ. Να δείξετε ότι: ΜΝ · ΠΛ = ΜΛ · ΠΝ

(Προτείνεται από τον μαθητή Γιάννη Χαρδftλοi>­

πα, «Εκπαιδευτήρια Κωστέα - Γείτονα») .

45. Μία εταιρεία έχει 20 υπαλλήλους. Μελετή­σαμε τις ηλικίες τους και βρήκαμε: μέση τιμή χ = 36, 8 έτη και τυπική απόκλιση S = 6,6 έ-τη . Στη συνέχεια η εταιρεία απέλυσε ένα υ­πάλληλο 50 ετών και προσέλαβε έναν άλλο 24 ετών. Να βρείτε τη νέα μέση τιμή χα και τη νέα τυπική απόκλιση Sa των ηλικιών των υπαλλήλων της εταιρείας αυτής.

(Προτείνεται από τον μαθητή Κων/νο Παπαγεωρ­

γίου, Εκπαιδευτήρια «0 Πλάτων», Αθήνα) .

46. Να βρείτε το ολοκλήρωμα:

Ι (χ ) - f dx - x(l + x4 )

(Προτείνεται από τον μαθητή Αντώνη Μιστριώ­

τη, Γερμανική Σχολή Αθηνών).

47. Μία συνάρτηση f είναι ορισμένη και συνεχής στο διάστημα [α, β] , παραγωγίσιμη στο (α, β) και ισχύουν: f (α) = f (β) = Ο . Να δείξετε ότι, για κάθε λ Ε IR , υπάρχει ξ Ε (α, β) με:

f ' (ξ) = λf 2 (ξ) .

(Προτείνεται από τον μαθητή Γεώργιο Σούμπλη,

Αρσάκειο Ψυχικού, Αθήνα) .

48. Μια συνάρτηση f είναι ορισμένη και συνεχής στο διάστημα [Ο, +οο) και για κάθε χ > Ο, ισχύουν:

όπου κ ένας θετικός αριθμός. Να δείξετε ότι: f (χ ) = Ο , για κάθε χ Ε [Ο, +οο) .

(Προτείνεται από τον μαθητή Σπύρο Λεονάρδο,

Κολλέγιο Αθηνών) .

ΕΥΚΛΕΙΔΗΣ Β ' λθ ' τ.3Π6

Page 79: Ευκλειδης Β 59

Τ Α ΑΙΑΘ 11 ΑΙΑΤ Ι Ι( Α ΑΙΑ%

JJI Α% I( 8 JJAZ Ο Υ 11

Su Doku Στην Ιαπωνία χρόνια τώρα έπαιζαν με ένα δια-

φορετικό σταυρόλεξο, το σταυρόλεξο των αριθ-μών. Το όνομά του (Su Doku) σημαίνει θέση του αριθμού.

Πρωτοεμφανίστηκε στους Times στις 1 2 Νοέμ-βρη 2004 και σήμερα δημοσιεύεται σε όλα σχεδόν τα περιοδικά και εφημερίδες.

Την ιδέα αυτή είχε ο Euler ο οποίοςτο 1 8° αιώ-να είχε δημιουργήcrει ένα παρόμοιο παιχνίδι.

Το Su Doku είναι κοινό για όλο τον κόσμο α-φού δεν είναι γραμμένο σε καμία γλώσσα αλλά χρησιμοποιεί τα εννέα ψηφία ( 1 ,2,3,4,5,6,7,8,9) και γι' αυτό είναι δημοφιλές.

Πώς παίζεται; Το «σταυρόλεξο» έχει 9 γραμμές και 9 στή-

λες, χωρισμένο σε 9 μικρότερα τετράγωνα 3χ3 και μας δίνεται η θέση ορισμένων αριθμών.

Εμείς πρέπει να συμπληρώσουμε τις κενές θέ-σεις έτσι που να είναι μόνο μία φορά το κάθε ψη-φίο σε κάθε γραμμή και σε κάθε στήλη και μία φορά σε κάθε μικρό τετράγωνο.

Υπάρχουν από εύκολα μέχρι πολύ δύσκολα su dοku,ανάλογα πόσων αριθμών δίνεται η θέση. 0-ρίστε μερικά συμπληρώστε τα.

4 3 7 6 1 7

4 1 9 6 2 4 8

5 8 9 7 2 8 5 1 3

9 8 7 3

3 1 9 4 ΕΥΚΟΛΟ

Επιμέλεια: Παναγιώτης Χριστόπουλος

9 6 4

7 8

1 6 1 5

9

ΜΕΤΡ ΙΟ

1 4 8

1

5 1 9

2

ΔΥΣΚΟΛΟ

Η ισότητα

9

5

7

8

6

1

2 7 1 5 3 9

6 1 4

2 7 9 3 2 7 6

5 8 3 9 7 9 4 7

1 3 2 7 4 2

8 9

Έχουμε την ισότητα 3 + 8 - 2 19 = 1 29, όμως δεν είναι αληθής. Προσθέστε μία ευθεία γραμμή οπουδήποτε μέσα στην ισότητα για να γίνει ένας αληθής ισχυρισμός.

Τα 9 σημεία Χωρίς να σηκώσετε το

μολύβι από το χαρτί, σχεδιάστε τέσσερις ευθείες γραμμές που να περνούν και από τα εννέα σημεία που βλέπετε

• • •

• • •

• • •

Η Οικογένεια ΕΥΚΛΕΙΔΗΣ Β' λθ' τ.3Π7

Page 80: Ευκλειδης Β 59

Τα Μαθηματικά μας διασκεδάζουν

Ένας πατέρας και μία μητέρα, ένας γιος και μία κόρη, μια αδελφή και ένας αδελφός, μια θεία και ένας θείος, μια ανιψιά και ένας ανιψιός και δύο ξαδέλφια συζητούν και προσπαθούν να μοιράσουν μια κληρονομιά έξι εκατομμύρια Ευρώ Τελικά πή­ραν από ένα και μισό εκατομμύρια γιατί;

Οι πίτες Ο Πέτρος θέλει να ψήσει τρεις πίτες αλλά η

ψησταριά χωράει μόνο δύο. Η κάθε πίτα θέλει δύο λεπτά από κάθε πλευρά για να ψηθεί. Μπορείτε να τον βοηθήσετε να τις ψήσει σε έξι λεπτά.

Η παλiρ ροια κ<η το καράβι Από ένα καράβι στο λιμάνι κρέμεται μια σκάλα

και το νερό φτάνει μέχρι το τρίτο σκαλοπάτι και 8 εκατοστά ποιο πάνω. Τα σκαλοπάτια έχουν μεταξύ τους απόσταση 20 εκατοστά. Η παλίρροια αρχίζει στις 9.00 ' και στην περιοχή το νερό ανεβαίνει 30 εκατοστά σε μια ώρα. Πόσα σκαλοπάτια θα είναι μέσα στο νερό στις 10.00 ' .

Λ Π Α r ,Π Η )� Ε Ι Σ του 1 "" η:ύχους

Λι cί,.Jωγ{Η rυναικώ;ι Οι 3άδες των αριθμών που έ:χ,ουν γινόμενο 36 είναι: 1 + 1

+ 36 = 38, 1 + 2 + 1 8 = 2 1 , 1 + 3 + 1 2 = 16, 1+4+9=14,

2+3+6=1 1 , 3+3+4=10, 1+6+6=13, 2+2+9=13 . Η κ. Πλά­

τωνος που γνωρίζει τον αριθμό της διεύθυνσης του σπιτιού

της δεν κατάλαβε μετά την απάντηση που πήρε στην πρώ­

τη ερώτησή της διότι οι ηλικίες των παιδιών είναι μία από

τις 3άδες που έ:χ,ουν το ίδιο άθροισμα 13 , έτσι με την δεύ­

τερη ερώτησή της κατάλαβε ότι οι ηλικίες είναι 2,2,9.

Dl ρόβλημα με τις t<λεψίJδρες Αναποδογυρίζουμε και τις δύο κλεψύδρες ,ο χρόνος αρ­

χίζει να μετράει. Όταν η ροή της άμμου στην κλεψύδρα

των 7 λεπτών σταματά την αναποδογυρίζουμε θέτοντας

πάλι σε λεΊτουργία. Όταν σταματήσει η ροή της άμμου

στην άλλη κλεψύδρα των 1 1 λεπτών αναποδογυρίζουμε

και πάλι την κλεψύδρα των 7 λεπτών και συνεχίζουμε

να μετράμε το χρόνο μέχρι που να αδειάσει. Συνολικός

χρόνος 7 +4+4= 1 5 .

Μαντέψτε την ηλικία των φίλων σας

Αφού το αποτέλεσμα των πράξεων τους το πολλαπλα­

σιάζουν με 9 άρα το άθροισμα των ψηφίων στο αποτέ­

λεσμα είναι πάντα 9. π.χ. 32+47=79, 79-2=77,

77 .9=693 . 6+9+3=1 8, 1 +8= 9, 9+ηλικία =Χ, Χ-9 = η­

λικία.

Ο έξυπνος μαθητής Ο Μιχάλης είπε στη Βάσω την συμμαθήτριά

του να γράψει έναν αριθμό, η Βάσω έγραψε 7 .543, τότε ο Μιχάλης της είπε ότι σ' αυτόν τον αριθμό θα προσθέσουμε δυο ακόμη αριθμούς εσύ και δυο αριθμούς εγώ και το αποτέλεσμα θα είναι 27.54 1 .

Πώς το βρήκε; (Εγραψαν με τη σειρά Βάσω: 254 1 , Μιχάλης:

7458, Βάσω: 3607, Μιχάλης:6392)

Η γυναίκα Μια γυναίκα ζαλισμένη από το ποτό πέρασε σε

μια διάβαση τρένου, αγνόησε τη μονή κατεύθυνση σ' ένα δρόμο, κινήθηκε μέσα σε μια λωρίδα λεω­φορείων, μπήκε αντίθετα στην κίνηση σε τρεις άλλους δρόμους και δεν είχε δίπλωμα οδήγησης μαζί της. Ο αστυνομικός που την παρακολουθούσε δεν την σταμάτησε, γιατί;

Τα τετράγωνα Με 1 Ο σπίρτα φτιάξτε δυο τετράγωνα το ένα με

δυο σπίρτα · σε κάθε πλευρά και το άλλο με ένα σπίρτο σε κάθε πλευρά.

Το 2 είναι ίσο με το 3 - · , χ 2 2 , χ , χ .::.ερουμε οτι αν = α τοτε = + α η = - α, για

την περίπτωσή μας είναι (2 - � ) = - (3 - � ) δηλαδή 2 2

5 5 5 2- - = - -3 2+3 = 2. - .

2 2 ,

2

Το ψευτολίJ κειο

Κάθε μαθητής που λέει την αλήθεια απαντάει μια φορά

ΝΑΙ στις ερωτήσεις ενώ κάθε μαθητής που λέει ψέματα

απαντάει δύο φορές ΝΑΙ στις ερωτήσεις. Το σύνολο

των ΝΑΙ είναι 60+40+30= 1 30 άρα τα επιπλέον 30 ΝΑΙ

είναι από τούς μαθητές που λένε ψέματα. (Διαβάστε τη

στήλη Μαθηματική Λογική) .

Ο γεωμέτρης μαθητής

Ο κύκλος διέρχεται από την κορυφή Ο της γωνίας

ΧΟΨ.

Μαθη ματικό σταυρόλεξο ι 2 3 4 5 6

ι ο 8 9

ι 6 I 8 ο

2 ο I 8 ο

3 ο Ε Μ Ε

2 ο ο 4 3

I ο 2 3 2

2

4

6

ΕΥΚΛΕΙΔΗΣ Β' λθ ' τ.3Π8

Page 81: Ευκλειδης Β 59

ΑΝΑΚΟΙΝΩΣΗ Η Ελληνική Μαθηματική Εταιρεία στην ετήσια Γενική της Συνέλευση που πραγματοποιήθηκε την

Κυριακή 12 Μαρτίου 2006 αποφάσισε την αύξηση της ετήσιας συνδρομής των μελών από το �τ J , από 15€ σε 20€.

Σημειώνουμε ότι η συνδρομή είχε την ίδια τιμή τα τελευταία ογτ:ώ χρόνια και η αύξηση αυτή ήταv επιβεβλημένη μετά από τη σημαντική αύξηση των τελών αποστολής των περιοδικών στα μέλη από τα Ελληνικά Ταχυδρομεία, όπως διαπιστώσαμε στο τέλος Ιανουαρίου.

Επειδή η επιβάρυνση της αποστολής των περιοδικών καθώς και η συνεχής αύξηση του κόστους έκδοσης των περιοδικών είναι μεγάλη παρακαλούμε όπως r.ξοφλεί-rε τ συνδρο,ιή σ ι;; tγκαι α σ ν Ε

Για ενημέρωσή σας, επάνω στην ετικέτα με το όνομα σας αναγράφεται το συνολικό ποσό το οποίο οφείλετε στην Ε.Μ.Ε. Οι τρόποι που μπορείτε να πληρώσετε τη συνδρομή σας είναι:

Στα γραφεία της Ε.Μ.Ε. Στα γραφεία των παραρτημάτων της Ε.Μ.Ε. Με ταχυδρομική επιταγή σε διαταγή, ΕΛΛΗΝΙΚΉ ΜΑΘΗΜΑτΙΚΉ ΕΤΑΙΡΕΙΑ, ΤΑΧ. ΓΡΑΦΕΙΟ ΑΘΗΝΑ 54, Τ.Θ. 30044 Είτε με κατάθεση του αντιτίμου της συνδρομής στους παρακάτω λογαριασμούς: � Τράπεζα ΕΘΝΙΚΗ, λογαριασμός όψεως 080/48002300 � Τράπεζα ALPHA, λογαριασμός όψεως 10 100 200 20 19 988

I 2.. Υ : Στην περίπτωση που η πληρωμή θα γίνει σε τράπεζα πρέπει να αποστείλετε στο Fax της ΕΜΕ (2 1 0-364 1 025) την απόδειξη κατάθεσης συμπληρωμένη με τα πλήρη στοιχεία του καταθέτη (ονοματεπώνυμο, διεύθυνση, τηλέφωνο) ώστε να είναι εφικτή η ορθή αποστολή των περιοδικών.

Από το Δ.Σ. της Ε.Μ.Ε. ΕΝΗΜΕΡΩΣΗ ΤΟΥ A PXEIOY ΤΗΣ Ε.Μ.Ε.

Αγαπητοί Συνάδελφοι, Η ΕΜΕ για την καλύτερη επικοινωνία και συνεργασία με τα μέλη της, στο πλαίσιο αναβάθμισης και

εμπλουτισμού της ιστοσελίδας της αλλά και την πληρέστερη ενημέρωσή σας σε θέματα που αφορούν τόσο στις δραστηριότητες του Σωματείου μας, αλλά και σε γενικότερα θέματα αποφάσισε να ανανεώσει και να συμπληρώσει το αρχείο των μελών της.

Για το λόγο αυτό σας παρακαλούμε, αφού συμπληρώσετε με ιδιαίτερη προσοχή και με Κ Ε Φ Λ Λ Λ Ι Α γράμματα τη φόρμα που ακολουθεί, να την αποστείλετε στην ΕΜΕ το συντομότερο δυνατό με fax στο 2 1 0-364 1 025 ή με e-mail στο info @hms.gr ή ταχυδρομικά. Εάν μέχρι τέλος Ιουνίου δεν έχουμε λάβει το σχετικό έντυπο θα θεωρήσουμε ότι τα στοιχεία που έχουμε στο αρχείο μας είναι σωστά και θα εξακολουθήσουμε να σας αποστέλλουμε τα περιοδικά «Ευκλείδης Α ' και Β '» . Διευκρινίζεται ότι στη θέση « [ 'J> Γ Α Σ I Α » πρέπει να συμπληρωθούν τα πλήρη στοιχεία της υπηρεσίας της επιχείρησης, του οργανισμού κλπ. στην οποία εργάζεστε. Επίσης στη θέση « Π Ε ΡΙ Ο Δ Ι ΚΟ>> εάν στην ίδια οικογένεια υπάρχουν 2 άτομα που είναι μέλη της Ε. ΜΕ. έχετε τη δυνατότητα να επιλέξετε δύο από τα τρία περιοδικά (μικρός Ευκλείδης, Ευκλείδης Α ' , Ευκλείδης Β ' από τέσσερα τεύχη το χρόνο) ή ένα από τα τρία επιστημονικά περιοδικά (Ευκλείδης Γ' , Μαθηματική Επιθεώρηση, από δύο τεύχη το χρόνο. Δελτίο από ένα τεύχος το χρόνο) .

Είμαστε βέβαιοι για τη θετική ανταπόκρισή σας στην προσπάθεια της Εταιρείας να ενημερώσει το Χ αρχείο και να καλύψει τις δικές σας απαιτi1 σης, προς όφελος όλων, σας ευχαριστούμε εκ των προτέρων.

ΚΩΔΙΚΟΣ Μ ΕΛΟΥΣ [ • • • . • . • . • . . . . . . . ]

ΟΝΟΜΑ: [ . . . . . . . . . . . . . . . . . . . . . . . . . . . . . . . . . . . . . . . . . . . . . . . . . . . . . . . . . . . . . . . . . . . . . . . . . ] ΕΠΩΝΥΜΟ : [ . . . . . . . . . . . . . . . . . . . . . . . . . . . . . . . . . . . . . . . . . . . . . . . . . . . . . . . . . . . . . . . . . . . . . . . . . . . . . . . . . ]

ΟΝΟΜΑ ΠΑΤΡΟΣ: [ . . . . . . . . . . . . . . . . . . . . . . . . . . . . . . . . . . . . . . . . . . . . . . . . . . . . . . . . . . . . . . . . . . . . . . . . . . . . . . . . ] HM/NIA ΓΕΝΝΗΣΗΣ : . . . . . / . . . . / . . . . . . .

ΟΔΟΣ : [ . . . . . . . . . . . . . . . . . . . . . . . . . . . . . . . . . . . . . . . . . . . . . . . . . . . . . . . . . . . . . . . . . . . . . . . . . . . . . ΑΡ.: . . . . . . . . . . . . . . ]

ΠΟΛΗ I ΠΕΡΙΟΧΗ : [ . . . . . . . . . . . . . . . . . . . . . . . . . . . . . . . . . . . . . . . . . . . . . . . . . . . . . . . . . . . . . . . . . . . . . . . . . . . . . . . . . . . . . . . . . . . . . . . ] ΤΑΧ. ΚΩΔΙΚΟΣ: [ . . . . . . . . . . . . . . . ]

ΤΗΛΕΦΩΝΟ: [ . . . . . . . . . . . . . . . . . . . . . . . . . . . . . . . . . . . . . . . . . . . . . . . . . . . . . . . . . . . . . . . . . . . . . . . . . . . . . . . . . . . . . . . . . . . . . . . . . . . ΚΙΝ.: . . . . . . . . . . . . . . . . . . . . . . . . . . . . . . . . . . . . . . . . . . . . . . . . . . . . . . . . . . . . . . ]

FAX :. [ . . . . . . . . . . . . . . . . . . . . . . . . . . . . . . . . . . . . . . . . . . . . . . . . . . . . . . . . . . . . . . . . . . ] E-MAIL : [ . . . . . . . . . . . . . . . . . . . . . . . . . . . . . . . . . . . . . . . . . . . . . . . . . . . . . . . . . . . . . . ]

ΣΠΟΥΔΕΣ: Α ' ΠΤΥΧΙΟ [ . . . . . . . . . . . . . . . . . . . . . . . . . . . . . . . . . . . . . . . . . . . . . . . . . . . . . . . . . . . . . . . . . . . . . . . . . . . . . . . . . . . . . . . . . . . . . . . . . . . . . . . . . . . . . . . . . . . . . . . . . . . ]

ΜΕΤΑΠΤΥΧΙΑΚΆ [ . . . . . . . . . . . . . . . . . . . . . . . . . . . . . . . . . . . . . . . . . . . . . . . . . . . . . . . . . . . . . . . . . . . . . . . . . . . . . . . . . . . . . . . . . . . . . . . . . . . . . . . . . . . . . . . . . . . . . . . . . . ]

ΔΙΔΑΚΤΟΡΙΚΟ [ . . . . . . . . . . . . . . . . . . . . . . . . . . . . . . . . . . . . . . . . . . . . . . . . . . . . . . . . . . . . . . . . . . . . . . . . . . . . . . . . . . . . . . . . . . . . . . . . . . . . . . . . . . . . . . . . . . . . . . . . . . ]

ΕΡΓΑΣΙΑ [ . . . . . . . . . . . . . . . . . . . . . . . . . . . . . . . . . . . . . . . . . . . . . . . . . . . . . . . . . . . . . . . . . . . . . . . . . . . . . . . . . . . . . . . . . . . . . . . . . . . . . . . . . . . . . . . . . . . . . . . . . . . . . . . . . . . . . . . . . . . . . . . . . . . . . . . . . . . . . . . . . . . . . . . . . . . . . . . . . . . . . . ]

ΠΕΡΙΟΔΙΚΟ: Μικρό Ευκλείδη D ΕΥΚΛΕΙΔΗ Γ D ΕΥΚΛΕΙΔΗ Α ' D ΜΑΘΗΜΑτΙΚΉ ΕΠΙΘΕΩΡΗΣΗ. D ΕΥΚΛΕΙΔΗ Β ' D ΔΕΛτΙΟ D

l"'===���������=��=��������;;,;,;;���������=�����o·.���o·J·

Page 82: Ευκλειδης Β 59

ΕΛΛΗΝΙΚΉ ΜΑΘΗΜΑΤΙΚΉ ΕΤΑΙΡΕΙΑ � • .

Πανεπιστημίου (Ελευθερίου Βενιζέλου) 34 - 1 06 79 ΑΘΗΝΑ Τηλ. 3 6 1 6532 - 3 6 1 7784 - Fax: 364 1 025

www.hms.gr - e-mail : [email protected] Ν Ε ΕΣ ΕΚΔΟΣΕΙΣ

Παίγνια και Λήψη Αποφάσεων Σελίδες 3 1 8 ISBN: 960-7341 -27-9. Σχήμα 1 9Χ23 Λ. Τ. 28 €

Το βιβλίο πραγματεύεται με ενιαίο τρόπο το αντικείμενο της Θεωρίας Παιγνίων και της κλασικής Θεωρίας Βελτιστοποίησης υπό το πρίσμα των εφαρμογών των θεωριών και των τεχνικών τους στα οικονομικά, τις πολιτικές επιστήμες και τη διοίκηση επιχειρήσεων. Αποφεύγοντας τη σχολαστικότητα, αλλά χωρίς να υστερεί σε μαθηματική αυστηρότητα, το βιβλίο απευθύνεται κυρίως σε προπτυχιακούς αλλά και μεταπτυχιακούς φοιτητές των οικονομικών, θετικών και τεχνολογικών σχολών, καθώς και σε όσους ενδιαφέρονται εν γένει για τη Θεωρία Παιγνίων και τη Θεωρία Αποφάσεων. Ο μεγάλος αριθμός των επεξεργασμένων παραδειγμάτων και των προς επίλυση ασκήσεων που καλύπτουν ένα ευρύτατο φάσμα εφαρμογών, εισάγουν βαθμιαία τον αναγνώστη στη μεθοδολογία και τη λογική της θεωρίας, και τον διδάσκουν πώς να αξιολογεί και να εφαρμόζει κατάλληλα τις σχετικές τεχνικές

ΕΛλΗΜIΚΗ tιΙΑ&ΗΜλτ1ΚΗ εΤΑΙΡΕΙΑ Ευκλείδης Γ ' Νο 63 Λ. Τ. 10 € Ετήσια συνδρομή Μελών 20 € (2 τεύχη)

ΕΙ\ΛΗΝΙΚΗ ΜΑΘΗΜΑτΙΚΗ €ΤΑΙΡΕΙΑ

Πιιlιι•ιιι ιιtιι

ΑιJφιι Ιntιφtί•εω• Χ. Δ. Ai'1tnpόvτn ΚQι S Κ. Chakrebarti

ΕΥΚΛΕΙΔΗΣ γ' ΠΣΡΙ()ΔΙΚΟ MAΘtHAA11K'ttt nA.1Δt:ΙAt Ο Ευκλείδης γ ' είναι το απαραίτητο βοήθημα του συναδέλφου Μαθηματικού. Αποτελεί το βήμα που

αποδέχεται τη συμβολή των συναδέλφων για μια καλύτερη μαθηματική παιδεία. Οι άξονες του Περιοδικού Ευκλείδης γ' είναι: � Στην κατεύθυνση της έρευνας με τη δημοσίευση ερευνητικών αποτελεσμάτων του πεδίου της

διδακτικής των Μαθηματικών και της Μαθηματικής Εκπαίδευσης. � Στην κατεύθυνση της επιμόρφωσης, με τη δημοσίευση άρθρων που συμβάλλουν στην ευρύτερη και

βαθύτερη κατάρτιση όσων διδάσκουν μαθηματικά � Στην κατεύθυνση της ενημέρωσης, με την ανακοίνωση και παρουσίαση συνεδρίων, ημερίδων,

μεταπτυχιακών προγραμμάτων και επιμορφωτικών σεμιναρίων που αναφέρονται στη Μαθηματική Εκπαίδευση.

ΠΕΡΙΕΧΟΜΕΝΑ

Ιωάννης Λαζαρίδης: Ο ρόλος των γεωμετρικών κατασκευών στη διδασκαλία της γεωμετρίας στην υποχρεωτική εκπαίδευση

Βένιος Αγγελόπουλος: Σχετικά με την εισαγωγή νέων εννοιών στα σχολικά μαθηματικά: Το παράδειγμα της παραγώγου

Δημήτριος Καραγεώργος - Σωτήριος Μπίνας: Σύγκριση δυο Μεθόδων Διδασκαλίας (Παραδοσιακής- Χρήση Η/Υ) στις Εγγεγραμμένες Γωνίες

Μπούτα Χαρά: Μια εναλλακτική προσέγγιση εκπαίδευσης δασκάλων σε θέματα Διδασκαλίας των Μαθηματικών

Αγγελική Μητρογιαννοπούλου : Μορφολογική Ψυχολογία & Γνωστικές Θεωρίες Μάθησης

Νικόλαος Τζίφας-Χρήστος Μινόπουλος - Αναστάσιος Ν. Μπαρκάτσας: Τα χαρακτηριστικά μιας δίκαιης και ισότιμης αξιολόγησης στα

Μαθηματική Επιθεώρηση Νο 62 Λ. Τ. 1 0 € Ετήσια συνδρομή Μελών 20 € (2 τεύχη)

Το περιοδικό ΜαθηματικιΊ Επιθεώρηση (Μ.Ε.) έ"..(ει στόχο να ε\•ημερώνει τους συναδέλφους μαθηματικούς της χώρας �ιας σε θέματα Μαθηματικιον, Ιστορίας και Φιλοσοφίας των Μαθηματικών, Εφαρμογών τω'' Μαθηματικών, Εκπαιδευτικής Έρευ,•ας, Ειδικών θεμάτων, Παιδαγωγικών θεμάτων κ.λπ. Η ανάπτυξη των παραπάνω κλάδων γίνεται με υψηλού επιπέδου άρθρα τα οποία δημοσιεύονται. Το υψηλό επίπεδο των εργασιών που δημοσιεύονται εξασφαλίζεται από τους κριτές που εποτελούνται από μέλη ΔΕΠ, Ερευνητές και άλλους επιστήμονες ειδικούς στις θεματικές ενότητες των επί κρίση εργασιών

ΠΕΡΙ ΕΧΟΜΕΝΑ

Ν. Μεταξάς - Σ. Μερκούρης: Κβαντικοί υπολογιστές

Ι. Η. Απλακίδης: Εύρεση των εστιών των κωνικών τομών από το περίγραμμα τους

Κ. Παπαδόπουλος: Ογδόντα χρόνια υπολογιστικών γωνιακών σχέσεων σε τρίγωνα και τετράπλευρα

ΜΑΘΗΜΑτΙΚΗ 6 � ΕΠΙΘΕΩΡΗΣΗ � .. : .. :": I Ο Υ � I Ο Σ

Περιοδικr'ι Εnιστημονική Έκδοση

Ελληνική Μαθηματικη Εταιρεfα

I. Γκουλιώνης: Η χρήση τού μοντέλου των μερικά παρατηρήσιμων Μαρκοβιανών αλυσίδων (P.O.M.D.P.S) σε διαδικασίες αξιολόγησης εκπαιδευτικών συστημάτων.

I. Μ. Ρούσσος: Μια Παρουσίαση Θεμάτων Προβλημάτων Μεταβολών στη Ρημάννια Γεωμετρία

Χρ. Δ. Στατεράς: Η πρώτη μεγάλη κρίση στην ιστορία των Μαθηματικών. Ο αριθμός ε ίναι άρρητος Δέκα διαφορετικές αποδείξεις

Συνάδελφοι Επισκεφθείτε την ιστοσελίδα της Ε."Μ.Ε. (www.hms.gr ) όπου υπάριουν όϊ..α τα τεύχη τοι) Ευ:κλέιδ11 Γ και τι1ς 1\fαθιιματικής Ε πιθειί)ρησης με τα περιεχόμενά τους :καθώς και όλες οι εκδόσεις της Ε.Μ.Ε.